AL7MA01ANPA0013-Corriges-des-exercices

publicité
Mathématiques
Terminales ES et L
Corrigés des exercices
Rédaction :
Isabelle Tenaud
Jean-Yves Hély
Sébastien Kernivinen
Coordination :
Sébastien Kernivinen
Ce cours est la propriété du Cned. Les images et textes intégrés à ce cours sont la propriété de leurs auteurs et/ou ayants droit
respectifs. Tous ces éléments font l’objet d’une protection par les dispositions du code français de la propriété intellectuelle ainsi que
par les conventions internationales en vigueur. Ces contenus ne peuvent être utilisés qu’à des fins strictement personnelles. Toute
reproduction, utilisation collective à quelque titre que ce soit, tout usage commercial, ou toute mise à disposition de tiers d’un cours
ou d’une œuvre intégrée à ceux-ci sont strictement interdits.
©Cned-2013
© Cned - Académie en ligne
C orrigé Séquence 1
Corrigé des activités du chapitre 2
Activité 1
Le jeu télévisé
Le premier mois Vincent touche 1 000 €.
Le versement augmente, sur le modèle des intérêts composés, de 3 % chaque
mois. Ainsi le 2e mois Vincent touchera 1 030 € (on obtient ce résultat en
calculant 1 000 × 1, 03 = 1 030 ).
Pour le 3e mois on calcule 1 030 × 1, 03 = 1 060, 90.
Le 3e mois Vincent touchera 1 060,90 €.
Chaque mois on multiplie la somme versée le mois précédent par 1,03.
La suite v est une suite géométrique de raison q = 1,03 et de premier terme
v 1 = 1 000.
On a donc v n = v 1 × q n −1 d’où v n = 1 000 × 1,03n −1.
La somme versée à Vincent le dernier mois est égale, en euros, à v .
12
La calculatrice nous donne v 12 = 1 000 × 1, 0311 = 1 384 , 233...
Le 12e mois Vincent touchera 1 384,23 €.
La somme totale gagnée par Vincent durant les 12 mois est
S = v 1 + v 2 + ...... + v 12.
Méthode 1 – Calculatrice
À l’aide de la touche rép sur TI 82 ou Ans sur Casio 25+ on obtient rapidement les 12 premières valeurs.
La somme des 12 valeurs correspond à un
gain total de 14 192,02 €.
Durant ces 12 mois Vincent aura gagné la
somme de 14 192 €.
On a choisi de ne garder que deux décimales.
▶ Sur TI
▶ Sur
82 à l’aide de la touche mode on fait
▾ ▸
▸
▸
entrer quitter
Casio 25+ on fait MENU (RUN) EXE SHIFT SET UP
Corrigé Séquence 1 – MA01
3
© Cned - Académie en ligne
À l’aide de la touche ▾ on
descend jusqu’à obtenir :
On fait alors F1 (Fix) 2 EXE
pour avoir comme écran :
Méthode 2 – Tableur « OpenOffice.org Calc »
A
B
C
Gain du n-ième mois Gain au bout de n mois
1
Rang
2
n
vn
Somme
3
1
1 000
1 000
4
2
1 030
2 030
5
3
1 060,90
3 090,90
6
4
1 092,73
4 183,63
7
5
1 125,51
5 309,14
8
6
1 159,27
6 468,41
9
7
1 194,05
7 662,46
10
8
1 229,87
8 892,33
11
9
1 266,77
10 159,10
12
10
1 304,77
11 463,87
13
11
1 343,92
12 807,79
14
12
1 384,23
14 192,02
Activité 2
B3 → 1 000
B4 → = B3*1,03
C3 → = B3
C4 → = arrondi(C3+B4 ;2)
On retrouve les mêmes
résultats.
Filon de minerai
Comme les quantités extraites diminuent chaque année de 1 % le coefficient
multiplicateur est égal à 0,99.
Ainsi T1 = T0 × 0, 99 = 20 000 × 0, 99 = 19 800. On retrouve bien le résultat
donné dans l’énoncé.
De même T2 = T1 × 0, 99 = 19 800 × 0, 99 = 19 602.
T3 = T2 × 0, 99 = 19 602 × 0, 99 = 19 405, 98
(soit 19 406 arrondi à la tonne).
D’où T1 = 19 800 ; T2 = 19 602 ; T = 19 406.
3
4
© Cned - Académie en ligne
Corrigé Séquence 1 – MA01
Le coefficient multiplicateur étant 0,99 on a Tn +1 = Tn × 0,99.
La suite (T ) est une suite géométrique de raison q = 0,99 et de premier
n
terme T0 = 20 000.
On a donc Tn = T0 × q n soit Tn = 20 000 × 0, 99n .
L’année 2012 correspond au rang n = 62. On calcule
T62 = 20 000 × 0, 9962 = 10 725, 36...
La quantité de minerai extraite en 2012 est environ égale à 10 725 tonnes.
Posons S = T + T + ...... + T .
n
1 2
n
D’après le Tableur « OpenOffice.org Calc » on trouve : S67 = 990 218
et S68 = 1 000 316.
1
A
B
C
D
Année
Rang
Quantité en tonnes
Somme en tonnes
n
Tn
Sn
2
3
1950
0
20 000
20 000
4
1951
1
19 800
39 800
5
1952
2
19 602
59 402
69
2016
66
10 303
980 018
70
2017
67
10 200
990 218
71
2018
68
10 098
1 000 316
C3 → 20 000
C4 → = arrondi(C3*0,99)
D3 → = C3
En 2017 le filon n’est pas totalement épuisé.
C’est, en théorie, dans le courant de l’année
2018 que le filon devrait être épuisé.
D4 → = D3+C4
Remarque
Dans le tableur les valeurs sont toutes arrondies à l’unité.
En arrondissant à 2 ou 3 décimales les résultats seraient
légèrement différents. Le filon serait tout de même épuisé
en 2018.
Corrigé Séquence 1 – MA01
5
© Cned - Académie en ligne
Corrigé des exercices
d’apprentissage du chapitre 2
Exercice 1
À chaque rebond, la balle remonte aux
ment.
P0
9
de la hauteur atteinte précédem10
Ainsi h1 = h0 × 0, 9 = 100 × 0, 9 d’où h1 = 90.
h2 = h1 × 0, 9 = 90 × 0, 9 d’où h2 = 81.
P1
P2
h1
h0
À l’issue du 1er rebond la balle remonte à 90 cm du sol et à
l’issue du 2e rebond elle remonte à 81 cm du sol.
h2
P9
h9
S
1er
2e
3e
9e
10e
REBONDS
Les trajets en pointillé sont virtuels
À chaque rebond la hauteur précédente est multipliée par
0,9.
Ainsi hn +1 = 0, 9 × hn .
La suite h est une suite géométrique de raison q = 0,9 et de
premier terme h0 = 100.
On sait que hn = h0 × q n d’où hn = 100 × 0, 9n .
n +1
On a h
− 100 × 0, 9n
n +1 − hn = 100 × 0, 9
n
hn +1 − hn = 100 × 0, 9 (0, 9 − 1)
hn +1 − hn = −10 × 0, 9n .
On en déduit hn +1 − hn < 0 et la suite h est décroissante.
La balle demeure à moins de 20 cm du sol dès que hn < 20.
On cherche donc le plus petit entier n tel que hn < 20.
La suite h étant décroissante il suffit de faire des essais à la calculatrice.
On trouve h15 = 20, 58... et h16 = 18, 53...
Pour tout n ≥ 16 on a donc hn < 20.
À partir du 16e rebond la balle restera à moins de 20 cm du sol.
La balle rebondit 10 fois sur le sol.
Appelons P0 le point d’où la balle est lâchée et S le point où elle touche le sol
avant de rebondir (voir figure).
Appelons Pi le sommet atteint après le i-ème rebond (pour 1 ≤ i ≤ 9).
La distance d parcourue par la balle depuis le lâcher jusqu’au moment où elle
touche le sol pour la dixième fois est donnée par
d = P0 S + S P1 + P1 S + S P2 + P2 S + S P3 + P3 S + ……+ S P9 + P9 S.
6
© Cned - Académie en ligne
Corrigé Séquence 1 – MA01
On a donc d = h0 + 2h1 + 2h2 + 2h3 + ...... + 2h9 .
d = h0 + 2(h1 + h2 + h3 + ...... + h9 ).
La somme h1 + h2 + h3 + ...... + h9 est la somme de 9 termes consécutifs d’une
suite géométrique.
1− q 9
1 − 0 , 99
h1 + h2 + h3 + ...... + h9 = h1 ×
= 900 (1− 0, 99 ).
= 90 ×
1− q
1− 0 , 9
Remarque
Pour comprendre les trajets de la
balle, des trajets virtuels ont été tracés « en décalé » sur la figure. Les
deux premiers rebonds ainsi que le
dernier sont représentés.
Exercice 2
D’où d = 100 + 1 800 (1− 0, 99 ) = 1 202, 6... (arrondi en cm à
1 203).
La balle aura parcouru une distance égale à 12,02 m environ
avant de rebondir pour la dixième fois.
Le premier mètre coûte 50 €. Chaque mètre supplémentaire creusé coûte 5 %
de plus que le précédent.
En appelant un le prix du n-ième mètre équipé on a la relation un +1 = 1, 05 × un
ce qui prouve que la suite u est une suite géométrique de raison q = 1,05 et
de premier terme u1 = 50.
D’où un = u1 q n −1 ce qui donne un = 50 × 1, 05n −1.
On pose Sn = u1 + u 2 + ...... + un . La somme Sn est donc la somme de n termes
consécutifs d’une suite géométrique.
1− q n
1− 1, 05n
On a Sn = u1 ×
= 50 ×
. D’où Sn = 1 000 (1, 05n − 1).
1− q
1− 1, 05
Le prix à payer pour équiper une falaise de 50 mètres de hauteur est égal
à S50 .
Calculons S50 = 1 000 (1, 0550 − 1) = 10 467, 399...
L’équipement d’une falaise de 50 m de hauteur coûte 10 467 €.
On cherche la hauteur maximale de la falaise qui peut être équipée, sachant
que le budget est de 120 000 €.
Comme chaque mètre supplémentaire a un coût, la suite S est obligatoirement croissante.
On cherche donc le plus grand entier n tel que Sn ≤ 120 000.
La suite S étant croissante il suffit de faire des essais à la calculatrice.
On obtient S98 = 118 275, 51... et S99 = 124 239, 29...
Pour n ≥ 99 on a Sn > 120 000.
Pour un budget de 120 000 € on pourra équiper une falaise de 98 m de
hauteur.
Corrigé Séquence 1 – MA01
7
© Cned - Académie en ligne
Exercice 3
Désignons par a la somme, exprimée en centimes d’euro, versée le n-ième
n
jour du mois de février (1 ≤ n ≤ 28).
L’énoncé nous dit que a1 = 1, a2 = 2, a3 = 4 , etc. La somme versée étant doublée d’un jour au jour suivant, la suite a est une suite géométrique de premier
terme a1 = 1 et de raison q = 2.
On a donc, pour 1 ≤ n ≤ 28, an = a1q n −1 d’où an = 2n −1.
Calculons a10 = 29 = 512.
Le 10 février le grand-père versera 512 centimes d’euro, soit 5,12 €.
La somme totale versée à la date du 10 février est donnée par
1− 210
S10 =
= 1 023.
1− 2
Le 10 février Énora disposera d’une somme de 1 023 centimes d’euro, soit
10,23 €.
Le séjour au ski coûtant 1 000 €, Énora ne peut pas partir au ski avec cette
somme.
En désignant par S la somme totale versée le n-ième jour du mois de février
n
1− 2n
on a Sn =
= 2n − 1.
1− 2
La somme totale augmente chaque jour, la suite S est donc une suite croissante.
La calculatrice nous donne S16 = 65 535 et S17 = 131 071.
Comme 1 310, 71 > 1 000, Énora pourrait partir au ski dès le 17 février.
Le grand-père avait promis de verser une somme d’argent jusqu’au 28 février.
Calculons donc S28 = 228 − 1 = 268 435 455.
Remarque
Voir sur Internet la
légende de l’échiquier.
Exercice 4
Cette somme, convertie en euros, s’élève à la coquette somme de :
2 684 354,55 €.
Le 28 février Énora serait donc millionnaire! Pas sûr que le grand-père ait
prévu cela…
Chaque année les remboursements augmentent de 2 %. Le premier rembour-
sement est b1 = 20 000.
On a b2 = b1 × 1, 02 = 20 000 × 1, 02 = 20 400.
b3 = b2 × 1, 02 = 20 400 × 1, 02 = 20 808.
Ainsi b2 = 20 400 et b3 = 20 808.
Le coefficient multiplicateur est égal à 1,02.
La suite (bn ) est donc une suite géométrique de premier terme b1 = 20 000
et de raison q = 1,02.
8
© Cned - Académie en ligne
Corrigé Séquence 1 – MA01
On sait que b = b q n −1 d’où b = 20 000 × 1, 02n −1.
n
1
n
n
−qn
−
1
1 1, 02
Sn = b1 ×
= 20 000 ×
d’où Sn = 1 000 000 (1, 02n − 1).
1− q
1− 1, 02
L’acquéreur rembourse le prêt en 7 versements. On calcule donc S .
7
On a S7 = 1 000 000 (1, 027 − 1) soit S7 = 148 685, 66... (arrondi à 148 686).
L’acquéreur remboursera au total la somme de 148 686 €.
Corrigé Séquence 1 – MA01
9
© Cned - Académie en ligne
Corrigé de l’activité du chapitre 3
Activité 3
Marché des télécommunications
On estime que, chaque année, 20 % de la clientèle de A change pour B alors
que 30 % de la clientèle de B change pour A. En 2010 la société A comptait 900
clients et la société B en comptait 100.
On note an le nombre de clients de la société A en (2010 + n).
Pour n = 1 on obtient a = 900 × 0, 8 + 100 × 0, 3 = 750. On a bien a = 750.
1
1
Calculons a2 = 750 × 0, 8 + (1 000 − 750 ) × 0, 3 = 675. On trouve a2 = 675.
a
a
a
a
750 5
675 9
On a 1 =
= et 2 =
= . D’où 2 ≠ 1 , ce qui prouve que la
a0 900 6
a1 750 10
a1 a0
suite (an ) n’est pas géométrique.
La suite (an ) n’est pas géométrique.
La société A récupère
tEFTBDMJFOUÒMFTPJU an × 0, 8 ;
tEFMBDMJFOUÒMFEFMBTPDJÏUÏ#TPJU (1 000 − an ) × 0, 3.
D’où an +1 = an × 0, 8 + (1 000 − an ) × 0, 3.
En développant on trouve an +1 = 0, 5an + 300.
Voici des captures d’écran de calculatrice.
Sur Casio 25+
on fait Ans au
lieu de rép .
On observe que la suite (an ) semble décroissante et que le terme an semble
se rapprocher du nombre 600.
Conjecture
suite (an ) semble décroissante ;
▶ le terme a semble se rapprocher du nombre
n
▶ la
= 600.
On peut penser qu’à moyen terme le nombre de clients va se stabiliser : 600
pour la société A et 400 pour la société B.
10
© Cned - Académie en ligne
Corrigé Séquence 1 – MA01
Corrigé des exercices
d’apprentissage du chapitre 3
Exercice 5
▶ On
donne un =
2n +1
3n
On peut écrire un =
D’où
▶ On
.
2n × 2
3n
n
n
 2
 2
2
= 2 ×   . Comme 0 < < 1, lim   = 0.
3
 3
n → + ∞ 3
lim un = 0.
n →+∞
22n
donne v n =
.
3n +1
(2 )
=
2
On peut écrire v n
De plus
n
n
n
 4
1  4
4
= ×   . Comme 1 < , lim   = + ∞ .
3 n →+∞ 3
3n × 3 3  3 
1
> 0. D’où lim v n = + ∞ .
3
n →+∞
n
n
 2
 3
▶ On donne w =
n  3  −  2  .
n
n
 2
 3
2
3
On sait que 0 < < 1 et 1 < . D’où lim   = 0; lim   = +∞
3
2
n → + ∞  3
n → + ∞  2
n
 3
et lim −   = − ∞ . D’où lim w n = − ∞ .
n → + ∞  2
n→+∞
Exercice 6
Soit (un ) une suite géométrique de raison q (avec q > 0) telle que : u1 = 8
et lim u1 + u 2 + ...... + un = 10.
n→+∞
On sait que si la limite de la somme de n termes consécutifs, d’une suite géométrique est finie alors la raison q vérifie 0 < q < 1.
u
D’après le cours on peut écrire lim u1 + u 2 + ...... + un = 1 .
1− q
n→+∞
8
Comme u1 = 8 on obtient
= 10 ce qui donne 8 = 10 (1 – q).
1− q
La raison q est solution de l’équation 8 = 10 (1 – q) d’où q = 0,2.
La raison de la suite géométrique (un ) est q = 0,2.
Corrigé Séquence 1 – MA01
11
© Cned - Académie en ligne
Exercice 7
On donne, pour n ≥ 0, un =
2n +1
et Sn = u 0 + u1 + ...... + un .
3n
Montrons que la suite (un ) est une suite géométrique.
n
 2
2n × 2
21
Calculons u 0 =
= 2×   .
= 2. On a aussi un =
 3
3n
30
2
La suite (un ) est géométrique de raison q = et de premier terme u 0 = 2.
3
4
8
Calculons u1 et u 2. D’après la définition de la suite u1 = et u 2 = .
3
9
4 10
4 8 38
Calculons S1 = u 0 + u1 = 2 + =
et S2 = u 0 + u1 + u2 = 2 + + = .
3 3
3 9 9
10
38
Ainsi S1 =
et S2 = .
3
9
u
2
Comme 0 < q < 1, la limite de la suite (Sn ) est finie et lim Sn = 0 =
= 6.
2
1− q
n→+∞
1−
D’où lim Sn = 6.
3
n →+∞
Exercice 8
3
3
Soit (u ) la suite définie, pour n ≥ 0, par u =
. Calculons u 0 = .
n
n
n +1
2
2
3
n
3
3  1
On peut écrire un =
=
d’où un = ×   .
n +1
n
2  2
2
2 ×2
1
La suite (un ) est une suite géométrique de raison q =
et de premier
2
3
terme u 0 = .
2
Comme 0 < q < 1,
lim un = 0.
n→+∞
Calculons la somme R = u 5 + u 6 + ...... + u 36 .
 1
1−  
32
 2
1− q
3
= ×
On a R = u 5 ×
1
1− q
26
1−
2
32 

 1
3
On peut écrire R = × 1−    .
32   2  


32
=
32 

 1
× 1−    .
25   2 
3
On pose S = u + u + ...... + u . Ainsi S est la somme de termes consécutifs
n
5
6
n
n
1
3
3
et de premier terme u 5 =
= .
6
2
64
2
3
u
3
2× 3
lim Sn = .
D’après le cours lim Sn = 5 = 64 =
d’où
32
1
1− q
64
n→+∞
n→+∞
1−
2
d’une suite géométrique de raison q =
12
© Cned - Académie en ligne
Corrigé Séquence 1 – MA01
Corrigé des activités du chapitre 4
Activité 4
Abonnement à un magazine
a. On trace la droite (D) d’équation y = 0,85x + 1,8 et la droite (d) d’équation
y = x dans un repère orthonormé.
y
12
(D) y = 0,85 x + 1,8
(d) y = x
(D)
(d)
1,8
1
O
1
u0= 8 u1 u2 u3
12 x
b. On place u 0 = 8 sur l’axe des abscisses. On obtient u1 comme ordonnée
du point de (D) d’abscisse u 0 = 8. La droite (d) permet de ramener u1 sur
l’axe des abscisses ; il suffit ensuite de réitérer la construction précédente avec
u1, u2 , etc. On peut aussi faire la construction sur une calculatrice.
c. Conjecture : la suite semble avoir pour limite le réel 12, abscisse du point d’intersection des deux droites.
Corrigé Séquence 1 – MA01
13
© Cned - Académie en ligne
On pose, pour tout entier naturel n, v = u − 12.
n
n
Ainsi v 0 = u 0 − 12 = 8 − 12 = −4.
a. Exprimons v n +1 en fonction de v n .
v n +1 = un +1 − 12 (par définition de la suite v)
v n +1 = 0, 85un + 1, 8 − 12 = 0, 85un − 10, 2
un +1
v n +1 = 0, 85 (v n + 12) − 10, 2
un
v n +1 = 0, 85v n .
La suite (v n ) est une suite géométrique de raison q = 0,85 et de premier
terme v 0 = −4.
b. On sait que, pour tout n ≥ 0, v n = v 0 × q n d’où v n = −4 × 0, 85n .
Par définition v n = un − 12 d’où un = v n + 12.
On a donc, pour tout entier naturel n, un = 12 − 4 × 0, 85n .
c. Comme 0 < 0,85 < 1, la suite géométrique de terme général 0, 85n est
décroissante.
Comme – 4 < 0, la suite géométrique de terme général v n = −4 × 0, 85n est
croissante. Le fait d’ajouter 12 au terme v n ne change rien au sens de variation : la suite (un ) a le même sens de variation que la suite (v n ).
Les suites (v n ) et (un ) sont croissantes.
Remarque
On peut aussi montrer que
un +1 − un = 12 − 4 × 0, 85n +1 − 12 + 4 × 0, 85n = 4 × 0, 85n (1− 0, 85) = 0, 6 × 0, 85n .
d. Comme 0 < 0,85 < 1, lim 0, 85n = 0 et lim − 4 × 0, 85n = 0.
n→+∞
n→+∞
On a un = v n + 12 et lim v n = 0. Ainsi lim un = 12.
n→+∞
n→ + ∞
Un magazine, vendu uniquement par abonnement, avait 8 milliers d’abonnés
en 2010.
D’une année sur l’autre 15 % des abonnés ne se réabonnent pas, donc
85 % se réabonnent. On sait d’autre part que chaque année il y a 1 800 nouveaux abonnés ce qui fait 1,8 millier de nouveaux abonnés par an.
14
© Cned - Académie en ligne
Corrigé Séquence 1 – MA01
a. On désigne par un le nombre de milliers d’abonnés en (2010 + n) .
Ainsi u 0 = 8 et un +1 = 0, 85 × un +
abonnements
renouvelés
1
,8
.
nouveaux
abonnés
Le nombre d’abonnements à ce magazine peut donc être modélisé par la
suite (un ).
b. L’année 2016 correspond au rang n = 6.
Calculons u 6 = 12 − 4 × 0, 856 = 10, 491 4...
On peut estimer à 10 491 le nombre d’abonnés à ce magazine en 2016.
Activité 5
Évolution de population
Au 1er janvier 2005, une ville avait une population de 100 000 habitants.
Pour tout entier naturel n, on note un le nombre d’habitants de cette ville au 1er
janvier de l’année 2005 + n.
Ainsi u 0 = 100 000.
On sait que ▶ le nombre d’habitants de la ville augmente chaque année de 5 %
du fait des naissances et des décès ;
▶ du fait des mouvements migratoires, 4 000 personnes supplémentaires viennent s’installer chaque année dans cette ville.
Calculons le nombre d’habitants en 2006 et en 2007.
On a u1 = 1, 05 × u 0 + 4 000 = 1, 05 × 100 000 + 4 000 = 109 000.
De même u 2 = 1, 05 × u1 + 4 000 = 1, 05 × 109 000 + 4 000 = 118 450.
Ainsi u1 = 109 000 et u 2 = 118 450.
On a u1 − u 0 = 9 000 et u 2 − u1 = 9 450.
Ainsi u1 − u 0 ≠ u 2 − u1, ce qui prouve que la suite u n’est pas arithmétique.
u1 109 000
u
118 450
=
= 1, 09 et 2 =
= 1, 086...
u 0 100 000
u1 109 000
u
u
Ainsi 1 ≠ 2 , ce qui prouve que la suite u n’est pas géométrique.
u 0 u1
On a
La population d’une année s’obtient en multipliant le nombre d’habitants de
l’année précédente par 1,05 et en ajoutant 4 000.
D’où un +1 =
1, 05 un
augmentation
naturelle
+ 4 000 .
nouveaux
habitants
Corrigé Séquence 1 – MA01
15
© Cned - Académie en ligne
a. On pose v n = un + 80 000. D’où un = v n − 80 000.
Calculons v 0 = u 0 + 80 000 = 100 000 + 80 000 soit v 0 = 180 000.
b. On peut écrire v n +1 = un +1 + 80 000
v n +1 = 1, 05 un + 4 000 + 80 000
un +1
v n +1 = 1, 05 un + 84 000
v n +1 = 1, 05 (v n − 80 000 ) + 84 000
v n +1 = 1, 05v n .
D’où
un
La suite v est géométrique de raison q =1,05 et de premier terme v 0 = 180 000.
c. On sait que v n = v 0q n d’où v n = 180 000 × 1, 05n .
On sait que un = v n – 80 000 .
D’où un = 180 000 × 1, 05n − 80 000.
a. L’année 2020 correspond au rang n = 15 .
Calculons u15 = 180 000 × 1, 0515 − 80 000 = 294 207, 07...
On peut estimer la population de la ville à 294 207 habitants en 2020.
b. On a un +1 − un = 180 000 × 1, 05n +1 − 80 000 − (180 000 × 1, 05n − 80 000 )
un +1 − un = 180 000 × 1, 05n × 1, 05 − 180 000 × 1, 05n − 80 000 + 80 000
un +1 − un = 180 000 × 1, 05n (1, 05 − 1)
un +1 − un = 9 000 × 1, 05n .
D’où un +1 − un > 0 et la suite u est croissante.
c. On cherche le plus petit entier n tel que un > 200 000.
Comme la suite u est croissante il suffit de faire des essais à la calculatrice .
On obtient u 9 = 199 239, 07... et u10 = 213 201, 03...
Pour tout n > 9 on a donc un > 200 000.
Dès l’année 2015 la population de cette ville devrait dépasser 200 000
habitants.
16
© Cned - Académie en ligne
Corrigé Séquence 1 – MA01
Corrigé des exercices
d’apprentissage du chapitre 4
Exercice 9
u = 4
On considère la suite (un ) définie par  0
.
un +1 = 1, 5un − 1
Calculons les termes u et u .
1
2
Pour n = 0 on obtient u1 = 1, 5u 0 − 1 = 1, 5 × 4 − 1 d’où u1 = 5.
Pour n = 1 on obtient u 2 = 1, 5u1 − 1 = 1, 5 × 5 − 1 d’où u 2 = 6, 5.
Ainsi u1 = 5 et u 2 = 6, 5.
Calculons u1 − u 0 = 5 − 4 = 1 et u 2 − u1 = 6, 5 − 5 = 1, 5.
u
u
5
6, 5
Calculons 1 = et 2 =
= 1, 3.
u0 4
u1
5
Comme u1 − u 0 ≠ u 2 − u1 la suite (un ) n’est pas arithmétique.
u
u
Comme 1 ≠ 2 la suite (un ) n’est pas géométrique.
u 0 u1
On définit la suite (v ) par v = u − 2. D’où u = 2 + v .
n
n
n
n
n
a. Calculons v 0 = u 0 − 2 = 4 − 2 = 2 et exprimons v n +1 en fonction de v n .
On peut écrire v n +1 = un +1 − 2
v n +1 = 1, 5un − 1− 2 = 1, 5un − 3
un +1
v n +1 = 1, 5(2 + v n ) − 3
v n +1 = 1, 5v n .
Ceci prouve que la suite (v n ) est géométrique de raison q = 1,5 et de premier
terme v 0 = 2.
b. On sait que v n = v 0 q n d’où v n = 2 × 1, 5n .
On sait que un = 2 + v n .
Ainsi un = 2 + 2 × 1, 5n .
c. On a 1 < 1,5 d’où lim 2 × 1, 5n = + ∞ .
n→+∞
De même
lim un = + ∞ .
n→+∞
Corrigé Séquence 1 – MA01
17
© Cned - Académie en ligne
Exercice 10
1
Soit (un ) la suite définie par u1 = 2 et, pour tout n ≥ 1, un +1 = un + 2.
2
La TI 82 affiche les valeurs de u , u et u (sur Casio 25+ faire Ans au lieu
2 3
4
de rép ).
← u1
← u2
← u3
← u4
On trouve u 2 = 3 ; u 3 = 3, 5 et u 4 = 3, 75.
D’après la représentation graphique des termes de la suite on conjecture que :
t MBTVJUFTFNCMFDSPJTTBOUF
t MBTVJUFTFNCMFBWPJSQPVSMJNJUFMFSÏFM ᐉ = 4.
On considère la suite (v ) définie, pour n ≥ 1, par v = 4 − u .
n
n
n
a. Calculons v 1 = 4 − u1 = 4 − 2 d’où v 1 = 2.
Exprimons v n +1 en fonction de v n .
1

On peut écrire v n +1 = 4 − un +1 = 4 −  un + 2
2

un +1
1
1
v n +1 = 2 − un = 2 − ( 4 − v n ) car v n = 4 − un d'où un = 4 − v n .
2 2
1
v n +1 = v n .
2
un
La suite (v n ) est géométrique de raison q = 0 ,5 et de premier terme v 1 = 2.
b. Le premier terme étant v 1 on a v n = v 1q n −1 = 2 × (0, 5)n −1.
 1
On peut encore écrire v n = 2 ×  
 2
n −1
1
soit v n =
.
2n − 2
1
Par définition v n = 4 − un d’où un = 4 − v n . On en déduit un = 4 −
.
n −2
2
c. On a 0 < 0,5 < 1 d’où lim v n = 0 ce qui implique
n→+∞
18
© Cned - Académie en ligne
Corrigé Séquence 1 – MA01
lim un = 4.
n→+∞
Exercice 11
Partie A – Étude de l’offre de la banque A
La banque A propose un placement à intérêts composés au taux annuel de
3,5 %. Le coefficient multiplicateur est donc égal à 1,035.
La suite a est une suite géométrique de raison q = 1,035 et de premier terme
a0 = 10 000.
On sait que an = a0 × q n d’où an = 10 000 × 1, 035n .
Le capital disponible au début de l’année 10 est donné par a .
10
Calculons a10 = 10 000 × 1, 03510 = 14 105, 987... (arrondi au centime d’euro
à 14 105,99).
Le capital disponible au début de l’année 10 est égal à 14 105,99 €.
Partie B – Étude de l’offre de la banque B
La banque B propose un placement à intérêts composés au taux annuel de 2 %
du capital. Les intérêts obtenus sont augmentés d’une prime annuelle de 170 €
intégrée au capital.
On a b0 = 10 000 et b1 = 10 370.
Calculons b2 = b1 × 1, 02 + 170 = 10 370 × 1, 02 + 170 d’où b2 = 10 747, 40.
On a b1 − b0 = 10 370 − 10 000 = 370 et
b2 − b1 = 10 747, 40 − 10 370 = 377, 40.
b 10 370
b
10 747, 40
On a 1 =
= 1, 037 et 2 =
= 1, 036 393...
b0 10 000
b1
10 370
Comme b1 − b0 ≠ b2 − b1 la suite b n’est pas arithmétique.
b b
Comme 1 ≠ 2 la suite b n’est pas géométrique.
b0 b1
On obtient le capital d’une année en multipliant le capital de l’année précé-
dente par 1,02 et en ajoutant la prime annuelle égale à 170 € .
.
D’où bn +1 = 1, 02 × bn + 170
capital
+
intérêts
prime
Pour tout entier naturel n on pose u = b + 8 500.
n
n
Corrigé Séquence 1 – MA01
19
© Cned - Académie en ligne
a. Calculons u 0 = b0 + 8 500 = 10 000 + 8 500 soit u 0 = 18 500.
b. Exprimons un +1 en fonction de un .
On a un +1 = bn +1 + 8 500
un +1 = 1, 02bn + 170 + 8 500 = 1, 02bn + 8 670
bn +1
un +1 = 1, 02(un − 8 500 ) + 8 670 car un = bn + 8 500
bn
d'où bn = un − 8 500
un +1 = 1, 02un .
La suite u est une suite géométrique de raison q = 1,02 et de premier terme
u0 = 18 500.
c. On sait que un = u 0 × q n d’où un = 18 500 × 1, 02n .
On en déduit bn = 18 500 × 1, 02n − 8 500.
Le capital disponible au début de l’année 10 est donné par b .
10
Calculons b10 = 18 500 × 1, 0210 − 8 500 = 14 051, 396... (arrondi au centime
d’euro à 14 051,40) .
Le capital disponible au début de l’année 10 est égal à 14 051,40 €.
On a a > b . L’offre de la banque A est donc (légèrement) plus intéres10
10
sante que celle de la banque B .
Sur 10 ans la différence n’est que de 54,59 € en faveur de la banque A.
Exercice 12
4
On donne v 0 = 100 et v n +1 = v n + 28.
5
On pose, pour n ≥ 0, u = v − α . Exprimons u
n
n
n +1 en fonction de un .
4
a. On peut écrire un +1 = v n +1 − α = v n + 28 − α (avec v n = un + α )
5
4
4
1
un +1 = (un + α ) + 28 − α = un + 28 − α .
5
5
5
4
Pour que la suite u soit géométrique de raison q =
il faut avoir
5
1
28 − α = 0, d’où α = 5 × 28 = 140.
5
20
© Cned - Académie en ligne
Corrigé Séquence 1 – MA01
Pour α = 140 la suite u est géométrique et on a la relation un = v n − 140.
Calculons u 0 = v 0 − 140 = 100 − 140 soit u 0 = −40.
b. On sait que un = u 0 × q
n
n
 4
d’où un = −40 ×   = −40 × (0, 8 )n .
 5
On en déduit v n = 140 + un soit v = 140 − 40 × (0, 8 )n .
n
750
5
L’indice d est donné par d =
− v .
n
n
7 7 n
750 5
750 5
250
On calcule d 0 =
− v0 =
− × 100, soit d 0 =
.
7 7
7 7
7
750 5 
750 5
On sait que d n =
− v n , d’où d n =
− × 140 − 40 × (0, 8 )n  .

7 7 
7 7
50 200
On obtient d n = +
× (0, 8 )n .
7
7
Les indices v et d sont des indices trimestriels. Pour calculer les valeurs
n
n
des deux indices au bout des trois ans, nous devons calculer v 12 et d 12.
La calculatrice nous donne
50 200
v 12 = 140 − 40 × (0, 8 )12 = 137, 251... et d 12 = +
× (0, 8 )12 = 9,106...
7
7
En arrondissant les deux indices à l’unité on obtient v 12 = 137 et d 12 = 9.
Remarque
On pouvait aussi calculer directement une valeur approchée
de d 12 connaissant v 12 ≈ 137, 251.
Corrigé Séquence 1 – MA01
21
© Cned - Académie en ligne
Corrigé des exercices de synthèse
de la séquence 1
Exercice I
En 2010 la forêt possède 50 milliers d’arbres. On a donc u = 50.
0
Comme 5 % des arbres sont abattus chaque année, le coefficient multiplicateur est égal à 0,95. On sait de plus que 3 milliers d’arbres sont replantés
chaque année.
On a donc la relation suivante, valable pour tout entier n :
un +1 = (1− 0, 05)un + 3
soit un +1 = 0, 95 un + 3.
abattage
de 5 %
arbres
replantés
(milliers )
Ainsi u 0 = 50 et, pour tout entier naturel n, un +1 = 0, 95 un + 3.
On considère la suite v définie pour tout entier naturel n par v = 60 − u .
n
n
D’où un = 60 − v n .
a. Exprimons v n +1 en fonction de v n .
On a v n +1 = 60 − un +1 = 60 − (0, 95 un + 3) = 57 − 0, 95 un
un +1
v n +1 = 57 − 0, 95(60 − v n ) = 57 − 57 + 0, 95 v n
v n +1 = 0, 95 v n .
un
La suite v est donc une suite géométrique de raison q = 0,95 et de premier
terme v 0 .
b. Calculons v 0 = 60 − u0 = 60 − 50 d’où v 0 = 10. On sait que v n = v 0 × q n
d’où v n = 10 × 0, 95n .
c. Par définition v n = 60 − un ce qui donne un = 60 − v n .
n
Ainsi, pour tout entier naturel n, un = 60 − 10 × (0, 95) .
L’année 2015 correspond au rang n = 5. Le nombre d’arbres de la forêt en
2015 est donné, en milliers, par u 5 .
La calculatrice donne u 5 = 52, 262 19...
On a 1 000 × u 5 = 52 262,19... (arrondi à l’unité à 52 262)
On estime à 52 262 le nombre d’arbres de la forêt en 2015.
22
© Cned - Académie en ligne
Corrigé Séquence 1 – MA01
(
a. On peut écrire un +1 − un = 60 − 10 × (0, 95)n +1 − 60 − 10 × (0, 95)n
un +1 − un = 10 × (0, 95)n (1− 0, 95)
)
un +1 − un = 0, 5 × (0, 95)n .
b. D’après ce qui précède on peut dire que, pour tout entier naturel n,
un +1 − un > 0.
La suite u est strictement croissante et donc strictement monotone.
Calculons 50 × 1,10 = 55. On cherche le plus petit entier n tel que u > 55.
n
La suite u étant strictement croissante on fait des essais à la calculatrice.
On a 2010 + 14 = 2024 .
Le nombre d’arbres de la forêt aura dépassé de 10 % le
nombre d’arbres de la forêt en 2010 à partir de 2024.
On a 0 < 0,95 < 1, d’où lim ( 0, 95)n = 0 et lim − 10 × ( 0, 95)n = 0.
n→+∞
n→+∞
Ainsi
lim un = 60.
n→+∞
0OTBJURVF t MBTVJUFu est strictement croissante ;
t lim un = 60 ;
n→+∞
t QPVSUPVUFOUJFSn, un < 60 car 60 − un = 10 × (0, 95)n .
Ainsi le nombre d’arbres devrait continuer à augmenter d’année en année et
se rapprocher de 60 000, sans jamais atteindre ce nombre (en théorie bien
entendu).
Exercice II
a. On désigne par C n le capital, exprimé en euros, disponible le 1er janvier de
l’année (2013 + n). Le capital disponible le 1er janvier 2014 est donc C1.
Le taux annuel du placement étant de 3,5 % le coefficient multiplicateur est
égal à 1,035. Le 31 décembre 2013 Monsieur Magot a placé 700 € supplémentaires .
Calculons 2 000 × 1, 035 + 700 = 2 770. Ainsi C1 = 2 770.
Le capital disponible le 1er janvier 2014 est égal à 2 770 € .
b. Le capital C n +1 est égal au capital C n multiplié par 1,035 auquel on ajoute
700 € .
D’où C n +1 = 1, 035 C n + 700.
Corrigé Séquence 1 – MA01
23
© Cned - Académie en ligne
Pour tout entier naturel n, on pose : u = C + 20 000.
n
n
a. Calculons u 0 = C 0 + 20 000 = 2 000 + 20 000 = 22 000.
Exprimons, pour tout entier n, un +1 en fonction de un .
On peut écrire
un +1 = C n +1 + 20 000 = 1, 035C n + 700 + 20 000 = 1, 035C n + 20 700
relation de
récurrence
C n +1
un +1 = 1, 035 × (un − 20 000 ) + 20 700
un +1 = 1, 035 × un .
La suite (un ) est une suite géométrique de raison q = 1,035 et de premier
terme u 0 = 22 000 .
b. On sait que un = u 0 × q n d’où un = 22 000 × (1, 035)n .
c. On connaît la relation un = C n + 20 000 d’où C n = un − 20 000.
Ainsi, pour tout entier naturel n, C n = 22 000 × (1, 035)n − 20 000.
d. Le capital disponible le 1er janvier 2018 est donné, en euros, par la valeur de
C 5 . La calculatrice nous donne C 5 = 6 129, 098... (arrondi à 6 129 à l’euro
près).
Le capital disponible le 1er janvier 2018 est égal à 6 129 €.
Désignons les 4 mensualités par m , m , m et m . Ces quatre mensualités
1 2 3
4
forment quatre termes consécutifs d’une suite arithmétique de raison r = 800.
On sait que m1 + m2 + m3 + m4 = 6 000.
On peut écrire
m1 + m1 + r + m1 + 2r + m1 + 3r = 4m1 + 6r = 4m1 + 6 × 800 = 6 000.
m2
m3
m4
On en déduit 4m1 = 1200 d’où m1 = 300.
Ainsi m1 = 300 ; m2 = 1100 ; m3 = 1900 ; m4 = 2 700.
Les quatre mensualités s’élèvent respectivement à
300 €, 1 100 €, 1 900 € et 2 700 €.
Exercice III
On peut indiquer dans un tableau les étapes montrant les soldes sur le compte
courant et sur le compte épargne.
24
© Cned - Académie en ligne
Corrigé Séquence 1 – MA01
Date
n
Solde du compte
courant
Sn
Solde du compte
épargne
en
28-01-12
1
500
250
28-02-12
2
500 + 250 =750
0, 5 × 750 = 375
28-03-12
3
500 + 375 = 875
0, 5 × 875 = 437, 5
28-04-12
4
?
n
500 + 437,5 = 937,5 0, 5 × 937, 5 = 468, 75
Sn = 500 + 0, 5 × Sn −1
Sn = 500 + en −1
en = 0 , 5 × Sn
Le 28 mars 2012 le solde S 3 est égal à 500 + 0, 5 × S2 = 875 et e 3 = 437, 5.
Le 28 avril 2012 le solde S 4 est égal à 500 + 0, 5 × S 3 = 937, 5 et e 4 = 468, 75.
D’après le tableau précédent on a : e
n +1 = 0, 5 × Sn +1 = 0, 5 × (500 + en ).
On a donc en +1 = 0, 5 × (en + 500 ).
a. Pour tout nombre entier naturel n non nul, on définit la suite v par
v n = 500 − en .
Calculons v 1 = 500 − e1 = 500 − 250 = 250.
Exprimons v n +1 en fonction de v n .
On a, pour tout n > 0, v n +1 = 500 − en +1 = 500 − 0, 5(en + 500 )
e
n +1
v n +1 = 500 − 250 − 0, 5en = 250 − 0, 5(500 − v n )
en
v n +1 = 0, 5v n .
La suite v est une suite géométrique de raison q = 0,5 et de premier terme
v 1 = 250.
b. On sait que, pour tout n ≥ 1, v n = v 1 × q n −1 d’où v n = 250 × (0, 5)n −1.
c. Calculons S = v 1 + v 2 + ...... + v 12.
La somme S est la somme de 12 termes consécutifs d’une suite géométrique.
1− q 12
1− (0, 5)12
= 250 ×
On calcule S = v 1 ×
. D’où S = 500 × 1− (0, 5)12  .


1− q
1− 0 , 5
Corrigé Séquence 1 – MA01
25
© Cned - Académie en ligne
a. On sait que en = 500 − v n , soit en = 500 − 250 × (0, 5)n −1.
b. Le montant C de la somme capitalisée sur le plan épargne est donné par
C = e1 + e2 + ...... + e12.
On a C = (500 − v 1) + (500 − v 2 ) + (500 − v 3 ) + ...... + (500 − v 12 )
C = 12 × 500 − (v 1 + v 2 + ...... + v 12 )
C = 6 000 − S = 6 000 − 500 × 1− (0, 5)12 


C = 5 500 + 500 (0, 5)12
C = 5 500,122...
La somme capitalisée sur le plan d’épargne le 29 décembre 2012 sera égale
à 5 500,12 €.
Exercice IV
À l’étape 2 on a colorié 8 nouveaux carrés ; ainsi c = 8.
2
Dans chacun des 8 carrés où l’on a colorié un carré à l’étape 2 on colorie 8
nouveaux carrés, ce qui fait 8 × 8 nouveaux carrés à l’étape 3 ; ainsi c 3 = 64.
On a donc c 2 = 8 et c 3 = 64.
On admet que la suite c est géométrique.
Son premier terme est c1 = 1 et sa raison q = 8.
On sait que c n = c1 × q n −1 d’où c n = 8n −1.
On désigne par S
n le nombre total de carrés coloriés à l’issue de l’étape n .
a. Ainsi S1 = 1; S2 = 1+ 8 ; S 3 = 1+ 8 + 64 ce qui donne S1 = 1; S2 = 9 ; S 3 = 73.
On a S2 = 9S1 mais S 3 ≠ 9S2 , ce qui prouve que la suite S n’est pas géométrique .
b. Sn est la somme de n termes consécutifs de la suite c qui est une suite géométrique .
1− q n
1− 8n
8n − 1
On a Sn = c1 ×
. Ainsi Sn =
d’où Sn =
.
1− q
1− 8
7
c. La suite S est une suite croissante car le nombre total de carrés coloriés augmente à chaque étape .
On cherche d’abord quel est le plus petit entier n tel que Sn ≥ 1000, puis
ensuite le plus petit entier n tel que Sn ≥ 1000 000.
26
© Cned - Académie en ligne
Corrigé Séquence 1 – MA01
À l’aide de la calculatrice on trouve :
Pour n ≥ 5 on a Sn ≥ 1000 et pour n ≥ 8 on a Sn ≥ 1000 000.
d. Comme 1 ≤ 8,
On a donc
lim 8n = + ∞ , lim (8n − 1) = + ∞ et
n→+∞
n→+∞
8n − 1
= + ∞ .
n→+∞ 7
lim
lim Sn = + ∞ .
n→+∞
Le nombre de carrés augmente très vite mais l’aire de chacun d’eux diminue
très vite elle aussi. On peut néanmoins penser qu’en coloriant de plus en plus
de carrés on va finir par colorier quasiment tout le carré initial .
Intuitivement on pense que lim an = aire du carré initial, d’où lim an = 1.
n→+∞
n→+∞
Remarque
n
 8
On peut démontrer que an = 1−   , d’où lim an = 1.
 9
n→+∞
Corrigé Séquence 1 – MA01
27
© Cned - Académie en ligne
C orrigé Séquence 2
Corrigé des activités du chapitre 2
Activité 1
Étude d’une fonction coût
Soit C la fonction définie sur [0 ; 20] par C ( x ) = 2x 3 − 54 x 2 + 500 x + 392.
La fonction C est une fonction polynôme dérivable sur [0 ; 20].
La fonction dérivée C ‘ est définie par C '( x ) = 6 x 2 − 108 x + 500.
Le trinôme 6 x 2 − 108 x + 500 a pour discriminant ∆ = −336. Comme ∆ < 0
le trinôme garde un signe constant sur [0 ; 20] ; ce signe étant le même que
celui du coefficient de x 2 , le trinôme est positif sur [0 ; 20].
La fonction C est croissante (strictement) sur [0 ; 20].
Dressons le tableau de variation de la fonction C.
x
0
C '( x )
20
+
4 792
C (x )
392
Soit A le point de (C ) d’abscisse x = 14.
Calculons C (14 ) = 2 296 et C '(14 ) = 164.
Une équation de la tangente à la courbe (C ) au point A est
y − C (14 ) = C '(14 ) × ( x − 14 ) ce qui donne ici y − 2 296 = 164 ( x − 14 ).
On obtient y = 164 x .
Une équation de la tangente en A est y = 164 x .
Cette tangente passe par l’origine du repère.
a. La recette R ( x ) est donnée par R ( x ) = 230 x . Le bénéfice B ( x ) est défini par
B ( x ) = R ( x ) − C ( x ) = 230 x − (2x 3 − 54 x 2 + 500 x + 392).
Ainsi B ( x ) = −2x 3 + 54 x 2 − 270 x − 392.
Corrigé Séquence 2 – MA01
29
© Cned - Académie en ligne
b. Étudions le sens de variation de la fonction B sur l’intervalle [5 ; 20].
La fonction B est une fonction polynôme dérivable sur [5 ; 20].
La fonction dérivée B ‘ est définie par B '( x ) = −6 x 2 + 108 x − 270.
Le trinôme −6 x 2 + 108 x − 270 a pour discriminant ∆ = 5 184. Les racines du
trinôme sont x 1 = 3 et x 2 = 15.
Comme x 1 ∉[5 ; 20] , seule la racine x 2 convient.
"JOTJ t B '( x ) = 0 pour x = 15
t B '( x ) < 0 pour x ∈ ]15 ; 20]
t B '( x ) > 0 pour x ∈[5 ; 15[.
La fonction B est donc croissante sur [5 ; 15] et décroissante sur [15 ; 20].
Dressons le tableau de variation de la fonction B.
x
5
B '( x )
15
+
0
20
–
958
B(x )
– 642
– 192
c. Le bénéfice est maximal lorsque l’entreprise fabrique 15 objets.
On a Bmax = 958 €.
a. On voit intuitivement que le bénéfice s’annule pour une valeur x dans [5 ; 15]
et pour une valeur x dans [15 ; 20].
La calculatrice donne B (8 ) = −120 et B (9 ) = 94 puis B (19 ) = 254 et
B (20 ) = −192.
L’entreprise reste bénéficiaire si elle fabrique un nombre x d’objets tel que
9 ≤ x ≤ 19.
b. L’entreprise veut assurer un bénéfice d’au moins 500 €. On cherche donc tous
les entiers x tels que B ( x ) ≥ 500.
La calculatrice donne B (10 ) = 308 et B (11) = 510 puis B (18 ) = 580 et
B (19 ) = 254.
Le bénéfice est au moins égal à 500 € pour les valeurs entières de x telles
que 11 ≤ x ≤ 18 , c’est-à-dire pour x ∈{11 ; 12 ; 13 ; 14 ; 15 ; 16 ; 17 ; 18}.
30
© Cned - Académie en ligne
Corrigé Séquence 2 – MA01
Activité 2
Fonction de demande et élasticité
Partie A – Étude d’une fonction
On considère la fonction f définie sur
par f ( x ) =
20 ( x + 1)
x 2 + 2x + 2
.
Pour que la fonction f soit définie il faut avoir x 2 + 2x + 2 ≠ 0.
On peut écrire x 2 + 2x + 2 = ( x 2 + 2x + 1) + 1 = ( x + 1)2 + 1 ce qui montre que,
pour tout x réel, x 2 + 2x + 2 > 0.
La fonction f est bien définie sur
.
a. La fonction f est le quotient de deux fonctions polynômes dérivables sur .
Comme le dénominateur ne s’annule pas sur , la fonction f est dérivable
sur .
u ( x ) = x + 1
u '( x ) = 1
d’où 
Posons 
2
v '( x ) = 2x + 2 = 2( x + 1)
v ( x ) = x + 2x + 2
On a f = 20 ×
u
u 'v − uv '
soit f ' = 20 ×
.
v
v2
D’où f '( x ) = 20 ×
= 20 ×
= 20 ×
Ainsi f '( x ) =
x 2 + 2x + 2 − ( x + 1) 2( x + 1)
( x 2 + 2x + 2)2
x 2 + 2x + 2 − 2x 2 − 4 x − 2
( x 2 + 2x + 2)2
− x 2 − 2x
( x 2 + 2x + 2)2
−20 x ( x + 2)
( x 2 + 2x + 2)2
.
.
La fonction dérivée a le même signe que le trinôme − x ( x + 2). Ce trinôme
s’annule pour x = −2 et pour x = 0.
t f '( x ) = 0 pour x ∈{ −2 ; 0}
t f '( x ) > 0 pour x ∈] − 2 ; 0[
t f '( x ) < 0 pour x ∈] − ∞ ; − 2[ ∪ ]0 ; + ∞ [.
b. La fonction f est croissante sur ] − 2 ; 0[, décroissante sur ] − ∞ ; − 2[ et sur
]0 ; + ∞ [.
Corrigé Séquence 2 – MA01
31
© Cned - Académie en ligne
Dressons le tableau de variation de la fonction f.
−∞
x
f '( x )
–2
–
+∞
0
0
+
0
–
10
f (x )
– 10
Partie B – Fonction de demande et élasticité
a. Le prix unitaire de 600 euros correspond à x = 6. On calcule donc f (6 ) = 2, 8.
Pour un prix unitaire de 600 euros,
le nombre d’objets demandés est égal à 2 800.
b. Lorsque le prix unitaire de 600 euros augmente de 1 % il est égal à 606 euros.
On calcule donc f (6, 06 ) = 2, 777 1...
Pour un prix unitaire de 606 euros,
le nombre d’objets demandés est égal à 2 777.
c. On calcule
2 777 − 2 800
× 100 = −0, 821...
2 800
Lorsque le prix passe de 600 € à 606 € la demande baisse de 0,82 %.
L’élasticité E ( x ) est définie par E ( x ) = x ×
a. E ( x ) = x ×
−20 x ( x + 2)
( x 2 + 2x + 2)2
D’où E ( x ) =
×
f '( x )
.
f (x )
x 2 + 2x + 2
− x 2 ( x + 2)
=
.
2
20( x + 1)
( x + 1)( x + 2x + 2)
− x 2 ( x + 2)
( x + 1)( x 2 + 2x + 2)
.
 − x 2 ( x + 2) ≤ 0
b. Pour x ≥ 0 on a 
d’où E ( x ) ≤ 0.
( x + 1)( x 2 + 2x + 2) > 0
32
© Cned - Académie en ligne
Corrigé Séquence 2 – MA01
Si x > 0 alors E ( x ) < 0, ce qui signifie que si un prix x non nul augmente de
1 % alors la demande f ( x ) diminue.
c. Sur l’écran de la calculatrice la courbe représentant la fonction E montre que
la fonction E semble décroissante sur [0 ; + ∞ [.
Les résultats affichés sur l’écran nous permettent de dire que pour un prix de
181 € environ l’élasticité est égale à – 0,5.
d. Lorsqu’un prix passe de 600 à 606 euros, ou de 100 à 101 euros, il augmente
dans les deux cas de 1 %. En calculant l’élasticité par rapport à ces deux prix
on obtient donc la variation de la demande.
Calculons E (6 ) = −0, 822 8... et E (1) = −0, 3.
Remarque
Lorsque le prix passe de 600 à 606 € la demande baisse de
On note que le réel E (6) est très
0,82 % ; lorsque le prix passe de 100 à 101 € la demande
proche du résultat obtenu en c.
baisse de 0,3 %.
Corrigé Séquence 2 – MA01
33
© Cned - Académie en ligne
Corrigé des exercices
d’apprentissage du chapitre 2
Exercice 1
C (q )
Le coût moyen est défini, pour q ≠ 0, par CM (q ) =
.
q
Le coût marginal de production est défini par C ma (q ) = C '(q ).
Dérivons la fonction « coût moyen ». On a C ' (q ) =
M
C '(q ) × q − C (q )
2
.
q
Si le coût moyen admet un minimum pour une valeur q 0 on a C 'M (q 0 ) = 0.
C (q 0 )
On a donc q 0C '(q 0 ) − C (q 0 ) = 0 d’où q =
0 C '(q ) .
0
C (q 0 ) C (q 0 )
Comparons CM (q 0 ) et C ma (q 0 ). On a CM (q 0 ) =
=
= C '(q 0 ).
C (q 0 )
q0
C '(q 0 )
Par définition du coût marginal C '(q 0 ) = C ma (q 0 ), ce qui implique
CM (q 0 ) = C ma (q 0 ).
Ainsi, pour la valeur q 0 telle que C 'M (q 0 ) = 0, on a bien CM (q 0 ) = C ma (q 0 ).
Soit A
0 le point d’abscisse q 0 situé sur la courbe représentant la fonction
« coût total ».
(
)
Le point A0 a pour coordonnées q 0 ; C (q 0 ) .
Le coefficient directeur de la tangente en A0 est m = C '(q 0 ).
L’équation de la tangente en A0 est de la forme y = C '(q 0 ) × q + b.
Déterminons la valeur de b en écrivant que A0 est sur la tangente.
Au point A0 on peut écrire C (q 0 ) = C '(q 0 ) × q 0 + b d’où b = 0.
L’équation de la tangente au point A0 d’abscisse q 0 est y = C '(q 0 ) × q .
La tangente en A0 passe par l’origine du repère.
Exercice 2
On considère la fonction « coût » définie sur l’intervalle [0 ; 20] par
C ( x ) = 2x 3 − 54 x 2 + 500 x + 392.
La fonction « coût moyen » est définie sur l’intervalle [5 ; 20] par
CM ( x ) = 2x 2 − 54 x + 500 +
34
© Cned - Académie en ligne
Corrigé Séquence 2 – MA01
392
.
x
a. Déterminons la dérivée de la fonction CM .
392
2x 3 − 27x 2 − 196
C 'M ( x ) = 4 x − 54 −
ce qui s’écrit aussi C 'M ( x ) = 2 ×
.
x2
x2
b. Comme x 2 > 0, C 'M ( x ) a le même signe que 2x 3 − 27x 2 − 196.
Développons
( x − 14 )(2x 2 + x + 14 ) = 2x 3 + x 2 + 14 x − 28 x 2 − 14 x − 196 = 2x 3 − 27x 2 − 196.
Ainsi C 'M ( x ) a le même signe que ( x − 14 )(2x 2 + x + 14 ).
c. Le trinôme 2x 2 + x + 14 a un discriminant négatif ( ∆ = −111). Pour tout x
réel il garde le signe du coefficient de x 2.
Ainsi, pour tout x de l’intervalle [5 ; 20], 2x 2 + x + 14 > 0.
La dérivée C 'M ( x ) a donc le même signe que x − 14.
t C 'M ( x ) = 0 pour x = 14
t C 'M ( x ) < 0 pour x ∈[5 ; 14[
t C 'M ( x ) > 0 pour x ∈ ]14 ; 20].
Dressons le tableau de variation de la fonction CM sur l’intervalle [5 ; 20].
x
5
C 'M ( x )
14
–
0
20
+
358,4
239,6
CM ( x )
164
a. La fonction « coût marginal » est définie sur l’intervalle [5 ; 20] par
C ma ( x ) = C '( x ) = 6 x 2 − 108 x + 500.
b. Déterminons la fonction dérivée de la fonction C ma .
On a C 'ma ( x ) = 12x − 108 = 12( x − 9 ).
t C 'ma ( x ) = 0 pour x = 9
t C 'ma ( x ) < 0 pour x ∈[5 ; 9[
t C 'ma ( x ) > 0 pour x ∈ ]9 ; 20].
La fonction C ma est décroissante sur [5 ; 9] et croissante sur [9 ; 20].
Corrigé Séquence 2 – MA01
35
© Cned - Académie en ligne
Dressons le tableau de variation de la fonction C ma sur l’intervalle [5 ; 20].
x
5
9
C 'ma ( x )
–
0
20
+
110
740
C ma ( x )
14
c. Le coût moyen est minimal pour x = 14. Comparons CM (14 ) et C ma (14 ).
CM (14 ) = 164 et C ma (14 ) = 164. On a donc CM (14 ) = C ma (14 ).
Les courbes représentatives des deux fonctions C et C
M
ma sont tracées sur
la figure 1.
Elles se coupent au point K (14 ;164 ).
y
Figure 1
700
y = CM(x) = 2x2 –54 x + 500 +
600
y = Cma(x) = 6x2 –108 x + 500
392
x
500
400
300
200
100
O
Exercice 3
K(14;164)
2
4
6
8
10
12
14
16
18
20
x
Partie A
On considère les fonctions C et B définies sur l’intervalle [0 ; 60] par
C ( x ) = 450 − 5x et B ( x ) = − x 2 + 55x − 450.
La fonction C est une fonction affine : elle est donc représentée par un seg-
ment sur [0 ; 60].
36
© Cned - Académie en ligne
Corrigé Séquence 2 – MA01
On calcule C (0 ) = 450 et C (60 ) = 150.
Le segment d est tracé sur la figure 2.
La dérivée de la fonction B est définie par B '( x ) = −2x + 55.
t B '( x ) = 0 pour x = 27, 5
t B '( x ) < 0 pour x ∈ ]27, 5 ; 60]
t B '( x ) > 0 pour x ∈[0 ; 27, 5[.
Dressons le tableau de variation de la fonction B sur l’intervalle [0 ; 60].
x
0
27,5
B '( x )
+
60
0
–
306,25
B(x )
– 450
– 750
La courbe (B ), qui est une partie de parabole, est tracée sur la figure 2.
Figure 2
y
d
400
K(30;300)
300
200
(B)
100
0
x
10
20
30
40
50
60
–100
–200
–300
–400
–500
d
–600
(B) y = B(x) = – x2 + 55x – 450
y = C(x) = 450 –5x
–700
–800
Corrigé Séquence 2 – MA01
37
© Cned - Académie en ligne
a. Résolvons l’inéquation B ( x ) ≥ 0, c’est-à-dire − x 2 + 55x − 450 ≥ 0.
Le trinôme − x 2 + 55x − 450 a pour discriminant ∆ = 1225 et pour
racines x 1 = 10 et x 2 = 45.
Comme le coefficient de x 2 est négatif (il est égal à – 1) le trinôme est positif
entre les racines.
L’ensemble des solutions de l’inéquation B ( x ) ≥ 0 est l’ensemble S tel que
S = [10 ; 45].
Graphiquement on vérifie que
t MBDPVSCFDPVQFMBYFEFTBCTDJTTFTQPVSx = 10 et pour x = 45 ;
t TVSMJOUFSWBMMF>ø<MBDPVSCFFTUTJUVÏFBVEFTTVTEFMBYFEFTBCTcisses.
b. Soit K le point de la courbe (B) d’abscisse x = 30. Ce point K a pour ordonnée
y = B ( 30 ) = 300.
Pour x = 30 on a 450 − 5x = 450 − 150 = 300, ce qui prouve que le point K
est sur le segment d.
Calculons B '( 30 ) = −60 + 55 = −5. Le nombre dérivé B '( 30 ) est égal au coefficient directeur du segment d , ce qui prouve que le segment d est tangent à
la courbe (B) au point K ( 30 ; 300 ).
Le segment d est tangent à la courbe (B) au point K ( 30 ; 300 ).
Partie B
La dépense, exprimée en euros, est égale à :
200 000 + 5 × N ( x ) = 200 000 + 5 × (50 000 − 1000 x ) = 450 000 − 5 000 x .
La dépense, exprimée en milliers d’euros, est donc égale à 450 − 5x = C ( x ).
La recette R(x), exprimée en euros, est égale à :
x × N ( x ) = x × (50 000 − 1 000 x ) = 1 000 x × (50 − x ).
La recette R(x), exprimée en milliers d’euros, est
R ( x ) = x (50 − x ) = 50 x − x 2.
Le bénéfice pour un spectacle est
B ( x ) = R ( x ) − C ( x ) = 50 x − x 2 − ( 450 − 5x ) = − x 2 + 55x − 450.
Le bénéfice, exprimé en milliers d’euros, est donc B ( x ) = − x 2 + 55x − 450.
38
© Cned - Académie en ligne
Corrigé Séquence 2 – MA01
a. D’après les variations de la fonction B le bénéfice sera maximal pour x = 27, 5.
Le bénéfice est maximal lorsque le prix du billet est fixé à 27,50 €.
b. Le bénéfice est positif ou nul si B ( x ) ≥ 0.
Le bénéfice est positif ou nul si le prix x du billet, exprimé en euros, est tel
que 10 ≤ x ≤ 45.
Exercice 4
La fréquence f (t ) est donnée par f (t ) =
300t
pour t ≥ 0.
3t + 2
300
= 60.
5
Le pourcentage de personnes qui connaissent le nom de ce remède au bout
d’une semaine est égal à 60 %.
Le pourcentage de personnes qui ignorent le nom de ce remède au bout
d’une semaine est donc égal à 40 %.
Calculons f (1) =
▶ Résolvons l’équation f (t ) = 75.
300t
= 75 ce qui donne
3t + 2
300t = 75( 3t + 2) = 225t + 150. D’où t = 2.
On résout l’équation
▶
Résolvons l’équation f (t ) = 93, 75.
300t
On résout l’équation
= 93, 75 ce qui donne
3t + 2
300t = 93, 75( 3t + 2) = 281, 25t + 187, 50. D’où t = 10.
0OFOEÏEVJU t "VCPVUEFEFVYTFNBJOFTEFTQFSTPOOFTDPOOBJTsent le nom de ce remède.
t Au bout de dix semaines 93,75 % des personnes
connaissent le nom de ce remède.
Les deux fonctions u : t 300t et v : t 3t + 2 sont définies et dérivables
sur [0 ; + ∞ [. De plus la fonction v ne s‘annule pas sur [0 ; + ∞ [.
u
est donc dérivable sur [0 ; + ∞ [
v
( 3t + 2) − 3t
2
f '(t ) = 300 ×
. D’où f '(t ) = 300 ×
.
2
( 3t + 2)
( 3t + 2)2
La fonction f =
et l’on a
Sur l’intervalle [0 ; + ∞ [ on a f '(t ) > 0. La fonction f est donc strictement
croissante sur [0 ; + ∞ [.
Corrigé Séquence 2 – MA01
39
© Cned - Académie en ligne
Dressons le tableau de variation de f sur l’intervalle [0 ; 18].
t
0
18
f '(t )
+
On a f (18 ) ≈ 96, 43.
675
7
f (t )
0
Le tracé de la courbe C, représentative de la fonction f, est sur la figure 3.
y
Figure 3
TA
100
90
C
80
70
60
50
A(1;60)
40
C
30
20
y = f(t) = 300 t
3t + 2
y = 24 t + 36
TA
10
0
2
4
6
8
10
12
14
16
18 t
Les coordonnées du point A sont A (1 ; f (1)) ce qui donne A (1 ; 60 ).
Pour t = 1 le nombre dérivé est f '(1) = 24.
Une équation de la tangente au point A est y = 24(t − 1) + 60 soit
y = 24t + 36.
Une équation de la tangente TA est y = 24t + 36.
Cette tangente TA est tracée sur la figure 3.
a. Le graphique nous montre que l’équation f (t ) = 90 admet une solution t
proche de 6. Calculons f (6 ) = 90.
Au bout de 6 semaines 90 % des personnes connaissent le nom de ce remède.
40
© Cned - Académie en ligne
Corrigé Séquence 2 – MA01
b. Le graphique nous montre que l’équation f (t ) = 95 admet une solution t
proche de 12.
Calculons f (12) = 94 , 736... et f (13) = 95,121...
Au bout de 13 semaines 95 % des personnes connaissent le nom de ce
remède.
Il faut donc 7 semaines pour passer de 90 % à 95 %.
c. On sait qu’au bout de 6 semaines 90 % des personnes connaissent le nom de
ce remède. Pour faire passer ce taux de 90 % à 95 % il faudrait sept semaines
supplémentaires de publicité.
La courbe nous montre aussi qu’à partir de t = 6 la croissance de la fonction
f est assez lente.
Il est donc raisonnable d’arrêter la campagne au bout de six semaines.
Exercice 5
Partie A
a. On lit sur le graphique f ( 4 ) = 100, ce qui montre qu’il y a saturation
pour x = 4.
b. La fonction f étant croissante sur l’intervalle [0 ; 4], sa dérivée est positive
sur cet intervalle.
La fonction f étant décroissante sur l’intervalle [4 ; 8], sa dérivée est négative
sur cet intervalle.
Il y a « envie » sur l’intervalle [0 ; 4] et « rejet » sur l’intervalle [4 ; 8].
a. Pour x = 4 la dérivée s’annule en changeant de signe, d’où v ( 4 ) = 0.
b. La fonction v est une fonction affine, d’où v ( x ) = ax + b.
On sait que v ( 4 ) = 0 et v (0 ) = 50.
t v (0 ) = 50 implique v (0 ) = b = 50
t v ( 4 ) = 4a + 50 = 0, d’où a = −12, 5.
La fonction v est définie par v ( x ) = −12, 5x + 50.
c. On a v '( x ) = −12, 5 ce qui montre que v est décroissante sur [0 ; 8].
La fonction f est définie par f ( x ) = ax 2 + bx + c .
On lit sur le graphique f (0 ) = f (8 ) = 0 et f ( 4 ) = 100.
Comme f (0 ) = 0 alors c = 0.
Corrigé Séquence 2 – MA01
41
© Cned - Académie en ligne
64a + 8b = 0
soit, après simplification,
D’où 
16a + 4b = 100
8a + b = 0 ← L1

 4a + b = 25 ← L2
L1 − L2 donne 4a = −25, d’où a = −6, 25.
2L2 − L1 donne b = 50.
La fonction f est définie sur [0 ; 8] par f ( x ) = −6, 25x 2 + 50 x .
Partie B
La fonction de satisfaction f
100 x
f (x ) =
.
x +1
On peut écrire 100 −
Ainsi f ( x ) = 100 −
est définie, pour tout x de [0 ; + ∞ [, par
100 100 ( x + 1) − 100 100 x
=
=
= f ( x ).
x +1
x +1
x +1
100
.
x +1
100
, d’où 100 − f ( x ) > 0. Ceci montre que, pour
x +1
tout x de [0 ; + ∞ [ , f ( x ) < 100.
On en déduit : 100 − f ( x ) =
Aucun réel x ne permet d’obtenir la saturation.
La fonction envie v est la dérivée de la fonction f.
u ( x ) = x
u '( x ) = 1
d’où 
.
Posons 
w ( x ) = x + 1
w '( x ) = 1
u
u 'w − uw '
On a f = 100 × , d’où f ' = 100 ×
.
w
w2
On obtient v ( x ) = f '( x ) = 100 ×
( x + 1) − x
2
( x + 1)
=
100
2
( x + 1)
. Ainsi v ( x ) =
100
( x + 1)2
.
La fonction v est strictement positive sur l’intervalle [0 ; + ∞ [.
Il y a « envie » sur tout l’intervalle [0 ; + ∞ [ et « rejet » sur aucun intervalle.
Partie C
La fonction de satisfaction f est définie sur l’intervalle [0 ; 30] par f ( x ) =
2 000 x
x 2 + 100
.
Les fonctions x 2 000 x et x x 2 + 100 sont dérivables sur [0 ; 30] et
x 2 + 100 > 0.
42
© Cned - Académie en ligne
Corrigé Séquence 2 – MA01
La fonction f est dérivable sur [0 ; 30].
u ( x ) = x
u '( x ) = 1
.
Posons 
d’où 
2
w '( x ) = 2x
w ( x ) = x + 100
u
u 'w − uw '
On a f = 2 000 × , d’où f ' = 2 000 ×
.
w
w2
On obtient f '( x ) = 2 000 ×
Ainsi f '( x ) = 2 000 ×
( x 2 + 100 ) − x (2x )
( x 2 + 100 )2
(10 − x )(10 + x )
( x 2 + 100 )2
= 2 000 ×
100 − x 2
( x 2 + 100 )2
.
.
a. Comme x ∈[0 ; 30] la fonction dérivée a le même signe que 10 − x .
t 10 − x = 0 pour x = 10
t f '( x ) = 0 pour x = 10
t 10 − x < 0 pour 10 < x ≤ 30 t f '( x ) < 0 sur ]10 ; 30]
t f '( x ) > 0 sur [0 ; 10[.
t 10 − x > 0 pour 0 ≤ x < 10.
Il y a « envie » sur l’intervalle [0 ; 10] et « rejet » sur l’intervalle [10 ; 30].
b. La fonction f est croissante sur [0 ; 10] et décroissante sur [10 ; 30].
Dressons le tableau de variation de f.
x
0
f '( x )
10
+
30
0
–
100
f (x )
0
60
Il y a saturation lorsque f ( x ) = 100.
La croisière doit durer 10 jours pour qu’il y ait saturation.
On cherche les réels x tels que f ( x ) ≥ 80.
D’après le tableau de variation de f on peut dire que l’équation f ( x ) = 80
admet deux solutions dans [0 ; 30]. L’une est inférieure à 10 et l’autre supérieure à 10.
La calculatrice donne f (5) = 80 et f (20 ) = 80.
Le niveau de satisfaction des clients est supérieur ou égal à 80 % de la saturation sur la période [5 ; 20], c’est-à-dire entre le 5e et le 20e jour.
Corrigé Séquence 2 – MA01
43
© Cned - Académie en ligne
Corrigé des activités du chapitre 3
Activitié 3
Fuite de gaz
Soit f la fonction définie sur l’intervalle 0 ; +∞  par
f (x ) =
 1
3
3
1 
−
= 3× 
−
.
2x + 1 (2x + 1)2
 2x + 1 (2x + 1)2 
Partie A
a. La fonction f est dérivable sur 0 ; +∞  car c’est la somme de deux fonctions
définies et dérivables sur 0 ; +∞  .
Cherchons la dérivée de la fonction x Elle est définie par x −
v '( x )
v 2( x )
1
(2x + 1)2
=
1
.
v (x )
.
Pour cela calculons d’abord la dérivée de la fonction v définie par
v ( x ) = (2x + 1)2.
Posons u ( x ) = 2x + 1 d’où u '( x ) = 2. Comme v = u 2 on a v ' = 2uu ' ce qui
donne v '( x ) = 2 × (
2x + 1) × 2 .
On obtient
u
u'
 −2
−2 × (2x + 1) × 2 
f '( x ) = 3 × 
−


 (2x + 1)2
(2x + 1)4
 −2 × (2x + 1)
4 
= 3× 
+

 (2x + 1)3
(2x + 1)3 
−4 x + 2
= 3×
.
(2x + 1)3
6(1− 2x )
.
Ainsi, pour x ≥ 0, f '( x ) =
(2x + 1)3
b. Sur [0 ; + ∞ [, la fonction dérivée a le même signe que 1− 2x .
t 1− 2x = 0 pour x = 0, 5
t 1− 2x > 0 pour 0 ≤ x < 0, 5
t 1− 2x < 0 pour x > 0, 5.
La fonction f est croissante sur [0 ; 0,5] et décroissante sur [0, 5 ; + ∞ [.
Dressons le tableau de variation de f sur [0 ; 15].
44
© Cned - Académie en ligne
Corrigé Séquence 2 – MA01
x
0
0,5
f '( x )
+
15
0
–
f (15) = 0, 093...
0,75
f (x )
90
961
0
c. Le taux de gaz est maximal au bout de 0,5 min c’est-à-dire au bout de 30
secondes.
Le taux maximal est égal à 0,75. (on peut aussi dire que le taux maximal est
de 75 %).
Le tracé de la courbe, pour x ∈[0 ; 15], est sur la figure 4.
Figure 4
y
0,8
0,7
Cf
0,6
3
y = f (x) =
2x + 1
Cf
0,5
3
(2x + 1)2
0,4
0,3
0,2
0,1
0
x
1
2
3
4
5
6
7
8
9
10
11
12
13
14
15
On voit d’après le graphique que le taux de gaz est inférieur ou égal à
0,10 ppm au bout de 14 minutes environ.
La calculatrice nous donne f (13, 98 ) = 0,100 014..., f (13, 99 ) = 0, 099 947... et
f (14 ) = 0, 099 881...
Le taux de gaz est devenu négligeable au bout de 14 min.
On cherche les réels x tels que f ( x ) > 0, 62.
D’après le graphique il existe deux valeurs de x telles que f ( x ) = 0, 62. Appelons α et β ces deux valeurs avec α < β.
On lit sur le graphique α ≈ 0, 25 et β ≈ 1, 25.
Corrigé Séquence 2 – MA01
45
© Cned - Académie en ligne
À l’aide de la calculatrice on obtient
f (0, 20 ) = 0, 612... et f (0, 21) = 0, 624... D’où 0, 20 < α < 0, 21.
f (1, 21) = 0, 620 7... et f (1, 22) = 0, 618... D’où 1, 21 < β < 1, 22.
 −0, 21 < −α < −0, 20
d’où 1 < β − α < 1, 02.
D’après ce qui précède 
 1, 21 < β < 1, 22
L’écart entre α et β est donc (légèrement) supérieur à 1.
On conclut que le personnel de l’usine a été affecté par la fuite de gaz.
Partie B
a. Sur [0 ; 0,5] la fonction f est strictement croissante et f (0 ) = 0.
Sur [0,5 ; 15] la fonction f est strictement décroissante et f (15) = 0, 09...
On a donc f (0 ) = 0 et f ( x ) > 0 pour x ∈[0, 5 ; 15].
b. Le nombre de solutions de l’équation f ( x ) = k est égal au nombre de points
d’intersection de la courbe Cf avec la droite horizontale d’équation y = k .
La courbe obtenue dans la partie A nous permet de remplir le tableau suivant.
k
0
0,5
0,75
0,80
Nombre de solutions de
l’équation f ( x ) = k
1
2
1
0
t 4VS MFT JOUFSWBMMFT < > < > FU < > MB GPODUJPO f est strictement
monotone. Il suffit donc de calculer les images des extrémités des intervalles.
t 4VS MFT JOUFSWBMMFT < > FU < > MB GPODUJPO f n’est pas monotone.
Dans ce cas il ne suffit plus de connaître les images des extrémités : on calcule
f (0, 3) = 0, 703..., f (0, 7) = 0, 729..., f (1) = 0, 666 et fmax = f (0, 5) = 0, 75.
On sait que sur [0,3 ; 0,7] et sur [0,3 ; 1] le maximum de la fonction f est
fmax = f (0, 5).
Comme f (0, 3) < f (0, 7) l’image de l’intervalle [0,3 ; 0,7] est l’intervalle
[f (0, 3) ; f (0, 5)].
Comme f (0, 3) > f (1) l’image de l’intervalle [0,3 ; 1] est l’intervalle [f (1) ; f (0, 5)].
Donnons tous les résultats dans un tableau.
Intervalle
Intervalle image
46
© Cned - Académie en ligne
[0 ; 0,3]
[0,3 ; 0,5]
[0,3 ; 0,7]
[0,3 ; 1]
[f (0 ) ; f (0, 3)] [f (0, 3) ; f (0, 5)] [f (0, 3) ; f (0, 5)] [f (1) ; f (0, 5)]
Corrigé Séquence 2 – MA01
[1 ; 5]
[f (5) ; f (1)]
Activité 4
Tarifs postaux
Les tarifs donnés dans l’énoncé nous permettent d’obtenir le tableau suivant :
x
10
18
20
f ( x ) 0,60 0,60 0,60
23
46
50
1
1
1
56
100
123
250
1,45 1,45 2,40 2,40
Le tracé de la courbe représentant la fonction f « tarifs postaux », sur l’inter-
valle ]0 ; 250[, est sur la figure 5.
y
Figure 5
Les 4 segments sont :
▶ ouverts à gauche ;
▶ fermés à droite.
2,5
On ne peut pas tracer la
courbe « sans lever le
crayon ».
1,5
2
Fonction
“tarifs postaux”
1
0,5
0
20
50
100
150
200
250 x
D’après la représentation graphique on peut dire que la fonction f est
constante sur chacun des intervalles ]0 ; 20], ]20 ; 50], ]50 ; 100] et ]100 ; 250].
On dit que f est une fonction « constante par intervalle ».
Pour cette fonction f qui est « constante par intervalle » les images sont des
valeurs isolées.
Ainsi, par exemple, tous les réels de l’intervalle ]50 ; 100] ont la même image
qui est 1,45.
Cela nous permet d’obtenir les images des six intervalles dans le tableau
suivant :
Intervalle
Intervalle image
]0 ; 20]
]0 ; 50]
{0, 6} {0, 6 ; 1}
]50 ; 250]
*
{1, 45 ; 2, 4}
[100 ; 200] [150 ; 200]
*
{1, 45 ; 2, 4}
*
{2, 4}
[100 ; 500]
{1, 45 ; 2, 4 ; 3, 25}
*
Remarque
Cet exemple nous montre que l’image d’un intervalle par une fonction
n’est pas toujours un intervalle.
Les ensembles { 0, 6} et { 2, 4} sont des intervalles réduits à un point.
Les ensembles repérés par une étoile (*) ne sont pas des intervalles.
Corrigé Séquence 2 – MA01
47
© Cned - Académie en ligne
Corrigé des exercices
d’apprentissage du chapitre 3
Exercice 6
La fonction f est définie pour x ≠ 1 par f ( x ) =
x 3 − 2x 2 − 5x + 6
.
x −1
La fonction f n’est pas définie pour x = 1; elle ne peut donc pas être continue
sur .
L’écran de la calculatrice semble pourtant montrer une courbe tracée « sans
lever le crayon » mais c’est impossible car f (1) n’existe pas. C’est donc qu’il
y a un « trou » dans la courbe, au niveau du point de coordonnées (1 ; − 6 ).
Soit g la fonction définie sur
par g ( x ) = x 2 − x − 6 .
Comme g est définie sur et que f (1) n’existe pas, les deux fonctions ne
sont pas égales. Ainsi f ≠ g .
La fonction g est une fonction trinôme : elle est continue sur
.
Quand on trace la courbe (C ) représentative de la fonction f et la courbe (P)
représentative de la fonction g elles semblent se « superposer ». Pourquoi ?
Développons ( x − 1)( x 2 − x − 6 ) = x 3 − x 2 − 6 x − x 2 + x + 6
= x 3 − 2x 2 − 5x + 6.
D’où f ( x ) =
x 3 − 2x 2 − 5x + 6 ( x − 1)( x 2 − x − 6 )
=
.
x −1
x −1
Pour x ≠ 1 on obtient f ( x ) = x 2 − x − 6 = g ( x ). Mais pour x = 1 on a
g (1) = −6 alors que f (1) n’existe pas.
Remarque
Sur la figure 18 de l’énoncé on peut
observer que pour x = 1 on a y = ce qui montre que 1 n’a pas d’image par f.
Exercice 7
Effectivement il est « normal » d’observer deux courbes
superposées ; la seule différence c’est que la courbe
(P) est une parabole complète alors que la courbe (C )
est une « parabole trouée ».
Le côté des petits carrés verts ne peut pas dépasser la moitié de la largeur
du rectangle.
D’où 0 ≤ x ≤ 5.
Le volume de la boîte est égal à l’aire du rectangle de base multipliée par la
hauteur de la boîte, c’est-à-dire la longueur x du côté des petits carrés.
48
© Cned - Académie en ligne
Corrigé Séquence 2 – MA01
Ainsi V ( x ) = x (
16
− 2x
)(10
−2
x ) = x ( 4 x 2 − 52x + 160 )
aire du rectangle
de base
soit V ( x ) = 4 x 3 − 52x 2 + 160 x .
a. Une fonction polynôme est toujours dérivable.
La dérivée de V est définie par V '( x ) = 12x 2 − 104 x + 160 = 4( 3x 2 − 26 x + 40 ).
Le trinôme 3x 2 − 26 x + 40 a pour discriminant ∆ = 196 et pour racines
20
x 1 = 2 et x 2 = .
3
Comme x ∈[0 ; 5], seule la solution x 1 = 2 convient.
Le trinôme 3x 2 − 26 x + 40 est positif à l’extérieur de ses racines et négatif
entre ses racines.
Pour le volume V, défini sur [0 ; 5], cela implique que la dérivée est positive
sur [0 ; 2] et négative sur [2 ; 5].
b. Dressons le tableau de variation de la fonction V.
x
0
V '( x )
2
+
0
5
–
144
V (x )
Courbe de la fonction V
0
0
Le volume maximal est égal à 144 pour x = 2 . D’où Vmax = 144 cm3 .
On cherche un intervalle [a ; b ] tel que, pour tout x de [a ; b ], V ( x ) ≥ 108.
fonction V est continue et strictement croissante sur [0 ; 2].
On a V ([0 ; 2] = [0 ; 144 ]. Comme 108 ∈[0 ; 144 ], il existe un réel α unique
tel que : α ∈[0 ; 2] et V (α ) = 108.
La calculatrice donne V (0, 94 ) = 107, 775... et V (0, 95) = 108, 499...
On a V (0, 94 ) ≤ V (α ) ≤ V (0, 95). La fonction V est croissante sur [0 ; 2] d’où
0, 94 ≤ α ≤ 0, 95.
▶ La
)
fonction V est continue et strictement décroissante sur [2 ; 5].
On a V ([2 ; 5] = [0 ; 144 ]. Comme 108 ∈[0 ; 144 ], il existe un réel β unique
tel que : β ∈[2 ; 5] et V ( β ) = 108.
▶ La
)
Corrigé Séquence 2 – MA01
49
© Cned - Académie en ligne
La calculatrice donne V ( 3, 25) = 108, 062... et V ( 3, 26 ) = 107, 548...
On a V ( 3, 26 ) ≤ V ( β ) ≤ V ( 3, 25). La fonction V est décroissante sur [2 ; 5]
d’où 3, 25 ≤ β ≤ 3, 26.
Le volume est supérieur ou égal à 144 cm3 sur l’intervalle [0,95 ; 3,25].
Exercice 8
On considère la fonction f définie sur
Conjecture
par f ( x ) = x + 1+
2
.
x +1
2
semble croissante sur l’intervalle I = [ −2, 35 ; 2, 35].
(elle semble même constante sur un petit intervalle...)
▶ la fonction f
x
–2
–1
0
f (x )
–0,6
1
3
0,3
0,5
0,75
1
2
valeur 3,1
arrondie
3,03
3
3,4
3,13*
La conjecture précédente est fausse car 0, 5 < 1 et f (0, 5) > f (1).
La courbe obtenue sur l’écran de la calculatrice coupe l’axe des abscisses en
un seul point.
L’équation f ( x ) = 0 admet une solution et une seule. On désigne par α cette
solution.
La fonction f est continue et strictement croissante sur [ −2 ; 0] et
f ([ −2 ; 0]) = [ −0, 6 ; 3].
Comme 0 ∈[ −0, 6 ; 3] il existe un réel unique α tel que α ∈ −2 ; 0 
et f (α ) = 0.
La calculatrice donne f ( −1, 58 ) = −0, 007... et f ( −1, 57) = 0, 007...
On a f ( −1, 58 ) ≤ f (α ) ≤ f ( −1, 57). La fonction f est croissante sur [ −2 ; 0] d’où
−1, 58 ≤ α ≤ −1, 57.
La courbe obtenue sur l’écran de la calculatrice coupe la droite d’équa-
tion y = 1, 5 en un seul point.
L’équation f ( x ) = 1, 5 admet une solution et une seule. On désigne par β
cette solution.
La fonction f est continue et strictement croissante sur [ −2 ; 0] et
f ([ −2 ; 0]) = [ −0, 6 ; 3].
Comme 1, 5 ∈[ −0, 6 ; 3] il existe un réel unique β tel que β ∈[ −2 ; 0] et
f ( β ) = 1, 5.
50
© Cned - Académie en ligne
Corrigé Séquence 2 – MA01
La calculatrice donne f ( −0, 77) = 1, 485... et f ( −0, 76 ) = 1, 507...
On a f ( −0, 77) ≤ f ( β ) ≤ f ( −0, 76 ). La fonction f est croissante sur [ −2 ; 0]
d’où −0, 77 ≤ β ≤ −0, 76.
Les coordonnées du point A sont ( 0 ; 3).
La fonction dérivée de f est définie par f '( x ) = 1−
Une équation de la tangente en A est y = x + 3.
4x
( x 2 + 1)2
d’où f '(0 ) = 1.
Pour étudier les positions relatives de C et de TA on va étudier le signe de la
différence y − f ( x ).
2
2
2x 2
On a y − f ( x ) = x + 3 − x − 1−
= 2−
=
.
x2 +1
x2 +1 x2 +1
Cette différence est toujours positive ou nulle.
t TJ x ≠ 0 alors C est située en dessous de TA ;
t TJ x = 0 alors C est tangente à TA .
Ceci confirme ce que l’on peut observer sur la figure de l’énoncé.
La droite (D ) passe par les points de coordonnées ( −1 ; 0 ) et ( 0 ; 1).
La droite (D ) a pour équation y = x + 1.
2
2
Par définition f ( x ) = x + 1+
d’où f ( x ) − ( x + 1) =
.
x2 +1
x2 +1
Comme x 2 + 1 > 0 on a, pour tout réel x , f ( x ) − ( x + 1) > 0.
Ainsi, pour tout réel x, la courbe C est située au-dessus de la droite (D ).
On en déduit que, pour tout réel x, x + 1 < f ( x ) ≤ x + 3.
Corrigé Séquence 2 – MA01
51
© Cned - Académie en ligne
Corrigé des exercices de synthèse
de la séquence 2
Exercice I
Partie A – Recherche d’un minimum

5
On donne K ( 0 ; 2, 5) soit encore K  0 ;  . Les coordonnées du point A
 2
 a2 
sont  a ;  .
2

On sait aussi que KA = ( x A − x K )2 + ( y A − y K )2 .
Les différentes valeurs de la longueur KA sont données dans un tableau.
a
0
1
2
(x A ;y A )
(0 ; 0)
 1
 1 ; 2 
(2 ; 2)
KA
5
2
5

5
 5 ; 2 
1
17
=
4
2
1+ 4 = 5
4+
5 = 2, 236...
17
= 2, 061...
2
5
5 = 2, 236...
t 4VS  0 ; 5 
Remarque


D’après la figure donnée dans l’énoncé et les valeurs trouvées on peut
conjecturer que la longueur KA diminue lorsque a ∈[0 ; a0 ] et augmente
lorsque a ∈[a0 ; 5 ], avec 1 ≤ a0 ≤ 2.
Pour a = 0 les points K et A ont la même
abscisse ; la droite (KA ) est verticale.
Pour a = 5 les points K et A ont la même
ordonnée ; la droite (KA ) est horizontale.
t 4VS  5 ; + ∞ 


La figure de l’énoncé nous montre que la longueur KA augmente lorsque a augmente.
KA = ( x − x )2 + ( y − y )2
A
K
A
K
2
)
 a2 5
2
1
1
d’où KA = a 2 +  −  = a 2 + a 2 − 5 = a 2 + a 4 − 10a 2 + 255
4
4
 2 2
(
a 4 6 2 25
1 4
KA =
− a +
a − 6a 2 + 25.
soit KA =
4 4
4
2
52
© Cned - Académie en ligne
Corrigé Séquence 2 – MA01
(
)
On pose, pour x ≥ 0, d ( x ) = x 4 − 6 x 2 + 25.
a. La dérivée de la fonction d est définie sur [0 ; + ∞ [ par
d '( x ) = 4 x 3 − 12x = 4 x ( x 2 − 3) = 4 x ( x − 3 )( x + 3 ).
Cette dérivée s’annule pour x = 0 et pour x = 3 (la valeur − 3 ne convient
pas car − 3 < 0) .
Ainsi d '( x ) a le même signe que le trinôme x ( x − 3 ) qui est négatif entre
les racines et positif à l’extérieur.
On obtient le tableau de variation suivant :
x
0
d '( x )
0
La fonction d est :
t EÏDSPJTTBOUFTVS 0; 3 
+∞
3
–
0
+
25
t DSPJTTBOUFTVS  3; +∞
d (x )
16
1 4
1
a − 6a 2 + 25 = d (a ).
2
2
La fonction « racine carrée » est croissante sur [0 ; +∞ [ d’où la longueur KA
varie comme d (a ).
b. On a KA =
Pour a = 3 la distance KA est minimale.
c. La distance minimale est égale à
1
16 = 2. On a KAmin = 2.
2
Partie B – Recherche d’un maximum
x2
est symétrique par rapport à l’axe des
2
ordonnées (la fonction f est paire).
Le point B, symétrique du point A par rapport à l’axe des ordonnées, se trouve
donc sur la parabole ᏼ .
La parabole
ᏼ d’équation y =
Soit H le milieu du segment [ AB ] (le point H est sur l’axe des ordonnées).
1
On a aire( ABK ) = AB × HK .
2
a
0
1
2
5
AB = 2a
0
2
4
2 5
HK
aire( ABK )
5
2
0
2
2
1
2
1
0
0
Corrigé Séquence 2 – MA01
53
© Cned - Académie en ligne
t 4VS 0 ; 5 


D’après la figure donnée dans l’énoncé et les valeurs trouvées on peut conjecturer que l’aire augmente lorsque a ∈[0 ; a1] et diminue lorsque a ∈[a1 ; 5 ],
avec 0 ≤ a1 ≤ 2.
t 4VS  5 ; + ∞ 


La figure de l’énoncé nous montre que l’aire augmente lorsque a augmente.
On a aire( ABK ) =
1
AB × HK .
2
t 4VS 0 ; 5 


Lorsque 0 ≤ a ≤ 5 le point H est en dessous du point K.
5 a2
Ainsi HK = y K − y H = − .
2 2
 5 a2  a
1
On a alors aire( ABK ) = × 2a ×  −  = ( 5 − a 2 ).
2
2 2  2
t 4VS  5 ; + ∞ 


Lorsque a ≥ 5 le point H est au-dessus du point K.
a2 5
Ainsi HK = y H − y K = − .
2 2
 a2 5 a
1
On a alors aire( ABK ) = × 2a ×  −  = (a 2 − 5).
2
 2 2 2
a
i (5 − a 2 ) si 0 ≤ a ≤ 5
D’où aire( ABK ) = 2
.
a 2
i (a − 5) si a ≥ 5
2
On s’intéresse maintenant au cas où 0 ≤ a ≤ 5 et on pose g (a ) = aire ( ABK ).
a
a. D’après ce qui précède on a alors g (a ) = (5 − a 2 ).
2
54
© Cned - Académie en ligne
Corrigé Séquence 2 – MA01
b. Le tableur « OpenOffice.org Calc » nous donne les valeurs de g (a ) en prenant
pour les valeurs de a un pas de 0,1.
On en déduit que, sur l’intervalle 0 ; 5  , la fonction « aire » est d’abord


croissante puis décroissante. Le maximum semble être atteint pour a ≈ 1, 3
et ce maximum vaut environ 2,2.
x
c. Soit g ( x ) = (5 − x 2 ) pour 0 ≤ x ≤ 5.
2
La fonction dérivée de g est définie sur l’intervalle 0 ; 5  par


1
1
2
2

g '( x ) = (5 − x ) + x ( −2x ) = (5 − 3x ).
 2
2
  5

 3  5
3 5
− x ×
+ x
On peut écrire g '( x ) = ×  − x 2  = × 
2 3
 2  3
  3

  15

3  15
g '( x ) = × 
− x × 
+ x.
2  3
  3

La fonction dérivée s’annule pour x =
15
.
3
Elle est strictement positive pour 0 ≤ x <
pour
15
< x ≤ 5.
3
15
et strictement négative
3
Corrigé Séquence 2 – MA01
55
© Cned - Académie en ligne
Le maximum est atteint pour x = 15 . (La calculatrice nous
3
15
donne
= 1, 290... )
3
 15 
15  15 
5− .
d. L’aire maximale du triangle ABK est égale à g 
=
6 
9
 3 
5 15
On a airemax =
.
9
La calculatrice donne
5 15
= 2,151...
9
L’aire maximale est égale à 2,15 au centième près.
Ces résultats sont cohérents avec ceux obtenus à l’aide du tableur.
Exercice II
La courbe C a pour équation y = f ( x ) = ax 3 + bx 2.
On sait que : f ( 0 ) = 0 ; f ( 4 ) = 1 ; f '( 0 ) = 0 ; f '( 4 ) = 0.
La dérivée de la fonction f est définie sur [0 ; 4] par f '( x ) = 3ax 2 + 2bx .
On obtient f ( 4 ) = 64a + 16b = 1 et f '( 4 ) = 48a + 8b = 0.
On simplifie la seconde équation et le couple (a , b ) est solution du système
64a + 16b = 1
.

 6a + b = 0
64a + 16b = 1
On résout 
6a + b = 0
1
−16
3
−32
1
3
. De même 16b = 3 d’où b = .
32
16
1
3
 1 3
On trouve (a , b ) =  − ,  d’où f ( x ) = − x 3 + x 2.
32
16
 32 16 
On obtient −32a = 1 d’où a = −
a. La fonction polynôme f est dérivable sur [0 ; 4 ].
3
3
3
On a f '( x ) = − x 2 + x = x ( 4 − x .
32
8
32
)
On retrouve que f '(0 ) = 0 et f '( 4 ) = 0.
x ≥ 0
Sur [0 ; 4] on a 
d’où f '( x ) ≥ 0.
4
−
x
≥
0

56
© Cned - Académie en ligne
Corrigé Séquence 2 – MA01
La fonction f est croissante sur [0 ; 4]. Dressons le tableau de variations de f
sur [0 ; 4].
x
0
f '( x )
0
4
+
0
1
f (x )
0
3
b. Le point K a pour coordonnées K (2 ; 0, 5) et on a f '(2) = .
8
3
1
Une équation de la tangente TK est donnée par y = ( x − 2) + .
8
2
3
1
Une équation de la tangente TK est y = x − .
8
4
3
c. La pente de la tangente TK est p = ce qui, exprimé en pourcentage,
8
donne p = 37, 50 %.
La pente de la tangente TK est de 37, 50 %.
Le tracé de la courbe C est sur la figure 6.
Les tangentes à la courbe C aux points (0 ; 0 ) et ( 4 ; 1) sont horizontales car
f '(0 ) = f '( 4 ) = 0.
Il semble que la courbe C traverse sa tangente au point K.
On conjecture
i C semble être au-dessus de TK sur l'intervalle [0 ; 2[;
i C semble être en dessous de TK sur l'intervalle ]2 ; 4 ].
(voir exercice d’apprentissage n° 2 – Séquence 8)
Figure 6
y
C
1
TK
1 3
3 2
x +
x
32
16
y = f(x) =
y=
3
x
8
1
4
C
0,5
0
TK
K (2 ; 0,5)
1
2
3
4 x
Corrigé Séquence 2 – MA01
57
© Cned - Académie en ligne
Exercice III
Partie A – Étude de la demande
Soit g la fonction définie sur [0 ; 5] par g ( x ) =
50
.
x2 + x +1
a. La fonction g est une fonction rationnelle définie sur [0 ; 5] ; elle est donc
dérivable sur [0 ; 5].
On a g '( x ) =
−50(2x + 1)
2
2
( x + x + 1)
.
b. Sur [0 ; 5] on a 2x + 1 > 0. Ainsi, pour tout x de [0 ; 5], g '( x ) < 0.
La fonction g est strictement décroissante sur [0 ; 5]. Dressons son tableau
de variation.
x
0
5
g '( x )
–
50
50
31
g(x )
Le point A d’abscisse 0 situé sur la courbe C a pour ordonnée 50.
g
Calculons g '(0 ) = −50.
La tangente en A(0 ; 50 ) a pour équation y = −50 x + 50 = −50 ( x − 1).
La courbe C et la tangente (T ) sont tracées sur la figure 7.
g
y
Figure 7
A(0 ; 50)
50
40
y = g(x) =
Cf
y = f(x) = 1,25x2 + 3
(T)
y = –50(x – 1)
10
K (q0 ; p0)
(T)
© Cned - Académie en ligne
x +x+1
Cg
20
58
2
Cf
30
0
50
Cg
1
Corrigé Séquence 2 – MA01
2
3
4
5 x
Partie B – Étude de l’offre
5
Soit f la fonction définie sur [0 ; 5] par f ( x ) = x 2 + 3.
4
La dérivée de f est définie sur [0 ; 5] par f '( x ) =
5
x . D’où f '(0 ) = 0 et
2
f '( x ) > 0 pour x ∈]0 ; 5].
La fonction f est strictement croissante sur [0 ; 5]. On a f (0 ) = 3 et
f (5) = 34 , 25.
La courbe C est tracée sur la figure 7.
f
Partie C – Recherche du prix d’équilibre
Soit K (q ; p ) le point d’intersection des deux courbes C et C .
0 0
g
f
Le prix d’équilibre p0 est compris entre 7 et 8 alors que la quantité q 0 associée est comprise entre 1 et 2.
D’où 7 ≤ p0 ≤ 8 et 1 ≤ q 0 ≤ 2.
On pose, pour tout x de
[0 ; 5] h( x ) = f ( x ) − g ( x ) .
a. En dérivant les deux membres de cette égalité on obtient h '( x ) = f
'( x ) − g
'( x ).
>0
D’où h '( x ) > 0.
<0
La fonction h est strictement croissante sur [0 ; 5].
b. La calculatrice nous donne h (1) = f (1) − g (1) = −12, 416...
et h (2) = f (2) − g (2) = 0, 857...
La fonction h est continue et strictement croissante sur [1 ; 2] et de plus
h (1) × h (2) < 0.
L’équation h ( x ) = 0 admet donc une solution unique, notée q 0 , sur l’intervalle [1 ; 2].
c. À l’aide de la calculatrice on trouve : h (1, 916 ) = −0, 001... et h (1, 917) = 0, 008...
Ainsi h (1, 916 ) ≤ h (q 0 ) ≤ h (1, 917).
La fonction h est strictement croissante sur [1 ; 2] d’où 1, 916 ≤ q 0 ≤ 1, 917.
Une valeur arrondie à 10 −2 de q 0 est q 0 = 1, 92.
Le nombre p est l’image de q par la fonction f et aussi l’image de q par
0
0
0
la fonction g.
On a donc p0 = f (q 0 ) = g (q 0 ).
Corrigé Séquence 2 – MA01
59
© Cned - Académie en ligne
 g (1, 917) ≤ g (q 0 ) ≤ g (1, 916 )
Comme 1, 916 ≤ q 0 ≤ 1, 917 on a 
f (1, 916 ) ≤ f (q 0 ) ≤ f (1, 917)
7, 585 ≤ p0 ≤ 7, 591
.
d’où 
7, 588 ≤ p0 ≤ 7, 594
On en déduit 7, 588 ≤ p0 ≤ 7, 591.
Une valeur arrondie à 10 −2 du prix d’équilibre p0 est p0 = 7, 59.
Exercice IV
Partie A
Soit g la fonction définie sur [5 ; 80] par g ( x ) = x 3 − 1200 x − 100.
La fonction g est une fonction polynôme dérivable sur [5 ; 80].
On a g '( x ) = 3x 2 − 1200 = 3( x 2 − 400 ) = 3( x − 20 )( x + 20 ).
Cette dérivée a le même signe que ( x − 20 ).
t g '( x ) = 0 pour x = 20
t g '( x ) < 0 pour 5 ≤ x < 20
t g '( x ) > 0 pour 20 < x ≤ 80
La fonction g est décroissante sur [5 ; 20] et croissante sur [20 ; 80].
Le tableau de variation de g est le suivant :
La calculatrice donne :
g (20 ) = −16 100 ;
g ( 40 ) = 15 900 ;
g ( 34 ) = −1596 ;
g ( 35) = 775 ;
g ( 34 , 5) = −436, 375.
x
20
5
g '( x )
–
0
–5 975
80
+
415 900
g(x )
–16 100
Sur l’intervalle [20 ; 40] la fonction g est continue et strictement croissante.
On a g (20 ) × g ( 40 ) < 0, d’où l’équation g ( x ) = 0 admet une solution
unique α sur l’intervalle [20 ; 40].
Comme g ( 34 , 5) < 0 et g ( 35) > 0, on peut dire qu’une valeur arrondie de α à
l’unité près est α = 35.
60
© Cned - Académie en ligne
Corrigé Séquence 2 – MA01
Donnons le signe de g ( x ) dans un tableau.
x
α
5
g(x )
–
80
0
+
Partie B
Soit f la fonction définie sur [5 ; 80] par f ( x ) = x + 50 +
1200 x + 50
x2
.
a. La fonction rationnelle f est définie sur [5 ; 80] et dérivable sur ce même
intervalle.
1200 x + 50
Dérivons la fonction h définie par h ( x ) =
.
x2
u ( x ) = 1200 x + 50
u '( x ) = 1200
.
d’où 
Posons 
2
v '( x ) = 2x
v ( x ) = x
On trouve h '( x ) =
1200 x 2 − 2x (1200 x + 50 )
x
4
=
−1200 x − 100
x3
.
La dérivée de la fonction f est telle que
f '( x ) = 1+ h '( x ) = 1−
1200 x + 100
x3
=
x 3 − 1200 x − 100
x3
Ainsi, pour x ∈[5 ; 80], on obtient f '( x ) =
g(x )
x3
.
.
b. La fonction dérivée f ‘ a le même signe que la fonction g.
La fonction f est décroissante sur [5 ; α ] et croissante sur [α ; 80].
Dressons le tableau de variation de f.
On a :
t f ( 80 ) ≈ 145.
t f (α ) ≈ f ( 35)
et f ( 35) = 119, 32...
x
α
5
–
f '( x )
0
297
80
+
f (80 )
f (x )
f (α )
Corrigé Séquence 2 – MA01
61
© Cned - Académie en ligne
Le tracé de la courbe (C ) est sur la figure 8.
y
Figure 8
220
(C)
200
180
(C)
y = f(x) = x + 50 +
(D)
y = 130
1200x + 50
x2
160
140
120
(D)
100
80
60
40
20
0
5
10
15
20
25
30
35
40
45
50
55
60
65
70
75
80 x
Les solutions de l’équation f ( x ) = 130 sont les abscisses des points d’inter-
section de la courbe (C ) et de la droite (D) d’équation y = 130.
D’après le graphique l’équation f ( x ) = 130 admet deux solutions x 1 et x 2.
On lit x 1 ≈ 20 et x 2 ≈ 60.
Partie C
Le coût total de fabrication est défini sur [5 ; 80] par
x 3 + 50 x 2 + 1200 x + 50
C (x ) =
.
x
Le coût moyen de fabrication par centaine d’objets est défini par
CM ( x ) =
C ( x ) x 3 + 50 x 2 + 1200 x + 50
=
.
x
x2
Ainsi CM ( x ) = x + 50 +
1200 x + 50
x2
d’où CM ( x ) = f ( x ).
D’après l’étude de f on sait que f est minimale pour x = α et que α ≈ 35.
Le coût moyen est minimal pour une fabrication de 35 centaines d’objets.
62
© Cned - Académie en ligne
Corrigé Séquence 2 – MA01
Le prix de vente d’une centaine d’objets est égal à 130 milliers d’euros. Le
prix de vente de x centaines d’objets est donc égal à 130 x milliers d’euros.
Le coût total de fabrication de x centaines d’objets est égal à C ( x ), exprimé
en milliers d’euros.
Pour que l’entreprise soit rentable il faut avoir 130 x > C ( x ). On peut diviser
les deux membres par x car x > 0.
C (x )
On obtient 130 >
, ce qui s’écrit aussi 130 > CM ( x ) ou encore 130 > f ( x ).
x
La calculatrice donne f (20, 06 ) = 130, 004... et f (20, 07) = 129, 984...
f (59, 97) = 129, 993... et f (59, 98 ) = 130, 000 56...
On a donc f ( x ) < 130 lorsque 20, 07 ≤ x ≤ 59, 97.
Pour être rentable l’entreprise doit fabriquer
au minimum 2 007 objets et au maximum 5 997 objets.
Exercice V
Partie A – Variation d’une fonction
Soit f la fonction définie pour tout x ∈[0 ; 100] par f ( x ) = x 3 + 4 x 2 + 6 x − 1.
La fonction f est une fonction polynôme dérivable sur [0 ; 100].
La dérivée est définie par f '( x ) = 3x 2 + 8 x + 6.
Le trinôme 3x 2 + 8 x + 6 a pour discriminant ∆ = −8. Comme ∆ < 0 le trinôme garde toujours le même signe ; le coefficient de x 2 est positif donc le
trinôme est toujours positif.
Pour tout x ∈[0 ; 100], f '( x ) > 0.
La fonction f est strictement croissante sur [0 ; 100].
a. Calculons f (0 ) = −1 ; f (0, 5) = 3,125 ; f (1) = 10.
b. La fonction f est continue et strictement croissante sur [0 ; 1]. On a
f (0 ) × f (1) < 0, d’où l’équation f ( x ) = 0 admet une solution unique α sur
l’intervalle [0 ; 1].
c. On calcule f (0,150 ) = −0, 006... et f (0,151) = 0, 000 6...
Ainsi f (0,150 ) < f (α ) < f (0,151).
La fonction f est strictement croissante sur [0 ; 1] d’où 0,150 < α < 0,151.
On en déduit le signe de f ( x ) sur l’intervalle [0 ; 100].
x
f (x )
α
0
–
0
100
+
Corrigé Séquence 2 – MA01
63
© Cned - Académie en ligne
Partie B – Taux d’intérêt
Développons
(
2
(1+ X )4 = (1+ X )2  = 1+ 2X + X 2


) = 1+ 4X + 6X
2
2
+ 4X 3 + X 4 .
D’où (1+ X )4 = 1+ 4 X + 6 X 2 + 4 X 3 + X 4 .
a. Calculons les intérêts successifs pour le second type de placement.
1re
Année
t
2
t
S
2
Taux
Intérêt
2e
t
tS
3e
3
t
2
3
tS
2
4e
2t
2t S
La somme T4 obtenue à la fin des quatre années est :

 t
t
3
3
T4 = S + S + t S + t S + 2t S = S × 1+ + t + t + 2t  d’où T4 = (5t + 1)S .
2
2
2

 2
b. On sait que S 4 = S (1+ t )4 ,
d’où S 4 − T4 = S (1+ t )4 − (1+ 5t )S = S (1+ t )4 − (1+ 5t ) .


4
D’après la première question on a (1+ t ) = 1+ 4t + 6t 2 + 4t 3 + t 4 ce qui
donne :
S 4 − T4 = S (1+ 4t + 6t 2 + 4t 3 + t 4 − 1− 5t )
= S (t 4 + 4t 3 + 6t 2 − t )
= S × t × (t 3 + 4t 2 + 6t − 1).
S 4 − T4 = S × t × f (t ).
c. Le deuxième placement est préférable au placement à intérêt composé au
taux d’intérêt t lorsque S4 – T4 < 0, c’est-à-dire lorsque f (t ) < 0.
D’après la partie A on sait que f (t ) < 0 lorsque t < α .
On sait aussi que 0,150 < α < 0,151, d’où S 4 − T4 < 0 lorsque t ≤ 0,150.
Si t ≤ 0,150 alors le deuxième placement est préférable au placement à
intérêt composé au taux d’intérêt t.
On peut aussi dire que, pour un taux inférieur ou égal à 15 %, le deuxième
placement est préférable au placement à intérêt composé au taux d’intérêt t.
64
© Cned - Académie en ligne
Corrigé Séquence 2 – MA01
C orrigé Séquence 3
Corrigé des activités du chapitre 2
Activité 1
Tri sélectif
Une enquête portant sur le tri sélectif a été réalisée et 2000 personnes ont été
interrogées.
On leur a posé la question : « Triez-vous le verre et le papier ? ».
Voici les résultats pour les effectifs :
Tri
Age
Oui
Non
Total par ligne
Moins de 40 ans : J
700
400
1100
40 ans et plus : J
500
400
900
Total par colonne
1200
800
2000 : Total général
On choisit au hasard une personne parmi les 2000 personnes interrogées, on
utilise donc la loi équirépartie, l’univers Ω étant formé par l’ensemble des
2000 personnes interrogées.
P (T ) =
card (T ) 1200
=
= 0, 6
card ( Ω ) 2000
P (J ) =
card ( J ) 1100
=
= 0, 55
card ( Ω ) 2000
P (T ∩ J ) =
700
= 0, 35.
2000
On choisit maintenant au hasard une personne parmi celles faisant du tri sélec-
tif. On utilise toujours une loi équirépartie, mais il faut tenir compte que l’univers a changé, c’est maintenant l’événement T pour lequel card ( T ) = 1200.
La probabilité qu’une personne ait moins de 40 ans sachant
card ( J ∩T ) 700
7
qu’elle fait du tri sélectif est donc égale à
=
= .
card (T )
1200 12
Corrigé Séquence 3 – MA01
65
© Cned - Académie en ligne
On remarque que cette probabilité est différente de la valeur de P ( J ) déterminée à la question précédente.
En observant les valeurs des probabilités de la question , on obtient que
700
card ( J ∩T ) 700 2000 P ( J ∩T )
=
=
=
.
card (T )
1200 1200
P (T )
2000
De la même façon, si on choisit une personne au hasard parmi les personnes
ayant moins de 40 ans, l’univers est J pour lequel card ( J ) = 1100.
La probabilité qu’une personne fasse du tri sélectif sachant
card ( J ∩T ) 700 7
qu’elle a moins de 40 ans est donc égale à
=
=
et
card ( J )
1100 11
card ( J ∩T ) P ( J ∩T )
on a aussi
=
.
card ( J )
P (J )
Plus généralement, et même s’il n’y a pas équiprobabilité, on utilisera des
quotients analogues pour étudier ce type de probabilité liée à une condition.
Activité 2
Dans l’initialisation, 30 dates d’anniversaire sont fixées aléatoirement entre 1
et 365 (on suppose qu’il n’y a pas d’année bissextile).
Lors du traitement, chaque date (numérotée k) parmi les 29 premières dates
d’anniversaire (le compteur k va de 1 à 29) est comparée avec les dates qui
la suivent (ce sont les dates numérotées de k + 1 à 30). Si deux dates coïncident, le booléen « trouvé », qui a été initialisé avec la valeur « faux » prend
la valeur « vrai ».
À la sortie, la valeur du booléen « trouvé » est affichée : si aucune coïncidence n’a été trouvée, « faux » est affiché, et, si une coïncidence a été trouvée, c’est « vrai » qui est affiché.
Pour obtenir un algorithme qui donne la fréquence des groupes où il existe
des coïncidences d’anniversaires dans 1000 groupes de 30 personnes, on
insère l’algorithme précédent, dans une boucle.
Variables
dates : tableau des trente jours d’anniversaire ;
trouvé : un booléen qui indique si deux dates coïncident ;
i, k, p : trois compteurs de boucles ;
N : un entier naturel
f : un nombre réel compris entre 0 et 1.
Initialisation
i =0
N =0
66
© Cned - Académie en ligne
Corrigé Séquence 3 – MA01
Traitement
Pour i de 0 à 1000
Pour k de 1 à 30
dates[k] prend une valeur entière aléatoire comprise
entre 1 et 365 inclus
trouvé prend la valeur faux
Pour k de 1 à 29
Pour p de k+1 à 30
Si dates[k] = dates[p] alors
trouvé prend la valeur vrai
Si trouvé = vrai N + 1→ N
N / 1000 → f
Sortie
Affiche f
En faisant fonctionner cet algorithme, on trouve, par exemple, f = 0, 839.
Cette fréquence peut paraître surprenante car on s’attend généralement à
une fréquence plus faible.
Les probabilités conditionnelles, qui seront définies dans cette séquence, permettront de calculer la probabilité que deux anniversaires coïncident dans un groupe
de 30 personnes et de comprendre les fréquences observées lors des simulations.
Corrigé Séquence 3 – MA01
67
© Cned - Académie en ligne
Corrigé des exercices
d’apprentissage du chapitre 2
Exercice 1
Par lecture de l’arbre, P (B ) = 0,8⫻0,3 + 0,2⫻0,6 (probabilités des deux che-
mins en gras) soit P (B ) = 0,36.
0,8
0,2
0,3
B
La bonne réponse est la b).
0,7
B
(Il faut tenir compte de P (A ) et P ( A ) donc cela ne peut être la a))
0,6
B
0,4
B
A
P ( A ∩ B ) = 0,2 × 0,6 = 0,12 donc la bonne réponse est la c). (ne
A
pas confondre avec P A (B ) ).
PA (B ) = 0,3 donc la bonne réponse est la a).
C’est la pondération de la branche : A ––––––– B.
Exercice 2
1
2
P ( A ) = , P (B ) =
1
1
et P ( A ∩ B ) = .
4
10
1
P ( A ∩ B ) 10 1
1
=
= ×2= .
On utilise la formule : PA (B ) =
1 10
P (A)
5
2
1
P ( A ∩ B ) 10 1
2
=
= ×4 = .
De même, PB ( A ) =
1 10
P (B )
5
4
1
2
P ( A ) = , P (B ) =
1
2
et P ( A ∪ B ) = .
3
3
On utilise la formule P ( A ∪ B ) = P ( A ) + P (B ) − P ( A ∩ B ).
1 1 2 1 1 1
D’où P ( A ∩ B ) = + − = − = .
2 3 3 2 3 6
1
P (A ∩B ) 6 1
= = .
On utilise la formule suivante : PA (B ) =
1 3
P (A)
2
1
P (A ∩B ) 6 1
De même PB ( A ) =
= = .
1 2
P (B )
3
68
© Cned - Académie en ligne
Corrigé Séquence 3 – MA01
Exercice 3
Les données de l’énoncé permettent de déterminer le nombre de personnes qui
ont les yeux bleus et qui ne fument pas, le nombre de celles qui fument et qui
n’ont pas les yeux bleus et enfin le nombre de celles qui n’ont pas les yeux bleus
et qui ne fument pas.
F
F
Total
B
4
16
20
B
36
24
60
Total
40
40
80
On utilise ensuite la loi équirépartie sur Ω ou des lois équiréparties sur les univers utilisés pour les probabilités conditionnelles.
Ω
Ω
0,75 = P(B)
P(B) = 0,25
0,5 = P(F)
P(F) = 0,5
B
P (F) = 4 = 0,2
B
20
F
Exercice 4
B
36
P (F) = = 0,6
60
B
F
0,8 = P (F)
B
0,4 = P (F)
B
F
F
P (B) = 4 = 0,1
F
40
F
F
P (B) =16 = 0,4
40
F
0,6 =P (B)
F
0,9 = P (B)
F
B
B
B
B
« Lorsque M1 est en panne, la probabilité que M2 tombe en panne est 0,5 »
traduit une probabilité conditionnelle : la probabilité que M2 tombe en panne
sachant que M1 est en panne est de 0,5 soit : PM (M2 ) = 0,5.
1
Comme on nous donne PM (M2 ) , on construit l’arbre avec M1, M1 d’abord :
1
re
2e
machine
1
machine
0,50
0,004
0,996
M1
M2
M2
P(M1) = 1 – P(M1) = 1 – 0,004 = 0,996.
M2
M1
M2
On utilise la formule : PM (M1)=
2
P (M1 ∩ M2 )
P (M2 )
avec P (M2 ) = 0, 006 (donné dans l’énoncé) et d’après l’arbre,
Corrigé Séquence 3 – MA01
69
© Cned - Académie en ligne
P (M1 ∩ M2 ) = P (M1) × PM (M2 ) = 0, 004 × 0, 5 = 0, 002
1
0, 002 1
= .
d’où PM (M1) =
2
0, 006 3
Remarque
On pourrait à l’aide de P( M2 ) calculer les autres probabilités de l’arbre du .
On a par exemple :
P (M2 ) = 0, 004 × 0, 5 + 0, 996 × PM (M2 ) = 0, 006
1
d'où PM (M2 ) =
1
Exercice 5
0, 006 − 0, 004 × 0, 5
= 0, 0040 à 10−4 près .
0, 996
On introduit les événements A : « la pièce est acceptée » et D : « la pièce est
défectueuse ».
3
= 0, 03 .
100
98 % des pièces bonnes sont acceptées donc la probabilité que la pièce soit
98
acceptée sachant qu’elle est bonne (c’est-à-dire non défectueuse) est de
100
soit de 0,98.
3 % des pièces sont défectueuses donc P (D ) =
Donc PD ( A ) = 0, 98 .
De même 97 % des pièces défectueuses sont refusées (donc non acceptées)
se traduit par : PD ( A ) = 0, 97 .
On commence donc l’arbre par D, D car on a PD (A ) et PD ( A ) soit :
0,03
0,97
0,03
A
0,97
A
0,98
A
0,02
A
D
P (D ) = 1− 0, 03 = 0, 97
D’après la loi des nœuds, PD ( A ) = 1− 0, 97 = 0, 03
D
et PD ( A ) = 1− 0, 98 = 0, 02
La probabilité qu’une pièce soit bonne mais refusée est P (D ∩ A ) .
D’après l’arbre ci-dessus : P (D ∩ A ) = 0, 97 × 0, 02 = 0,0194.
Une erreur de contrôle se produit si :
▶
▶
soit la pièce est défectueuse mais acceptée
soit la pièce est bonne mais refusée
donc la probabilité qu’il y ait une erreur de contrôle est P (D ∩ A) + P (D ∩ A ) .
70
© Cned - Académie en ligne
Corrigé Séquence 3 – MA01
Or d’après l’arbre, P (D ∩ A) = 0,03 × 0,03 = 0,0009 et P (D ∩ A ) = 0,0194.
La probabilité qu’il y ait une erreur de contrôle est donc de 0,0009 + 0,0194
soit de 0,0203.
La probabilité qu’une pièce acceptée soit mauvaise est la probabilité que la
pièce soit mauvaise (c’est-à-dire défectueuse) sachant qu’elle est acceptée à
savoir PA (D ).
P (D ∩ A )
P (A)
avec P ( A ) = P (D ∩ A ) + P (D ∩ A ) = 0, 00099 + 0, 97 × 0, 98 = 0, 9515.
PA (D ) =
Or
0, 0009
≈ 0, 000946. La probabilité qu’une pièce acceptée soit
0, 9515
mauvaise est d’environ 0,000946.
Donc PA (D ) =
Exercice 6
Résumons la situation par un arbre de probabilité. D’après l’énoncé :
1
1
4
p1 = P ( A1) = ; PA ( A2 ) = ; PB (B2 ) =
1
1
2
3
5
1
2
1
2
A1
1
3
A1 ∩ A2
D’après la formule des probabilités totales :
B1
2
3
A2
Les événements A1 et B1 forment une partition de ⍀.
1
5
4
5
B2
A2
A1 ∩ B2
B1 ∩ A2
P ( A2 ) = P ( A2 ∩ A1) + P ( A2 ∩ B1)
P ( A2 ) = P ( A1) × PA ( A2 ) + P (B1) × PB ( A2 )
1
1
1 1 1 1 1 1 8
4
P ( A2 ) = × + × = + = = B2
2 3 2 5 6 10 30 15
B1 ∩ B2 soit p = 4 .
2 15
Les événements A
n −1 et Bn −1 forment une partition de Ω.
pn –1
1 – pn –1
An –1
An
An –1 ∩ An
P ( An ) = P ( An ∩ An −1) + P ( An ∩ Bn −1)
Bn –1
1
3
pn = P ( An ) = P ( An −1) × PA
n −1
1
5
Bn
An
Bn –1 ∩ An
Donc
Bn
( An ) + P (Bn −1) × PB
n −1
( An )
1
1
pn = P ( An ) = pn −1 × + (1− pn −1) ×
3
5
 1 1 1 2
1
pn = pn −1  −  + = pn −1 +
5
 3 5  5 15
Corrigé Séquence 3 – MA01
71
© Cned - Académie en ligne
( )
Pour montrer que la suite u n
est une suite géométrique on peut exprimer
un +1 en fonction de un en utilisant la définition de la suite un et la relation prouvée à la question précédente.
Pour tout entier n, on a :
3
2
1 3
2
2
2
3 2
un +1 = pn +1 − = pn + − = pn − =  pn −  = un .
13 15
5 13 15
65 15 
13  15
( )
( )
2
Ainsi, pour tout entier n non nul, u n +1 = u n : la suite u n
15
2
3
géométrique de raison
et de premier terme u1 = p1 − =
15
13
 2
On en déduit u = u ×
n
1  15 
 2
Comme lim  
n →+∞  15 
n −1
7  2
= × 
26  15 
n −1
= 0 car 0 <
est la suite
1 3
7
− = .
2 13 26
n −1
.
2
< 1,
15
n −1
3 7 2
3
3
et lim pn = .
pn = u n + = ×   +
13 26  15 
13
13
n →+∞
Exercice 7
Soit A (resp. B, C, D ) l’événement : « l’élève choisit l’itinéraire A (resp. B, C, D ) ».
Soit R l’événement : « l’élève arrive en retard ».
P (D ) = 1− P ( A ) − P (B ) − P (C ) car A , B , C et D forment une partition de
l’univers.
1 1 1 1
Donc P (D ) = 1− − − = .
3 4 12 3
On cherche P (C ).
R
PR (C ) =
P (R ∩ C )
P (R )
Pour déterminer P (R ), on peut utiliser la formule des probabilités totales :
P (R ) = P (R ∩ A ) + P (R ∩ B ) + P (R ∩ C ) + P (R ∩ D )
P (R ) = P ( A ) × PA (R ) + P (B ) × PB (R ) + P (C ) × PC (R ) + P (D ) × PD (R )
1 1 1 1 1 1 1
P (R ) = × + × + × + × 0
3 20 4 10 12 5 3
P (R ) =
7
120
1 1
×
P (R ∩ C ) 12 5 2
=
PR (C ) =
=
7
P (R )
7
120
72
© Cned - Académie en ligne
Corrigé Séquence 3 – MA01
Remarque
On peut s’aider d’un arbre pondéré pour calculer P ( R ).
1
3
1
3
A
B
C
D
1
20
19
20
1
10
9
10
1
5
4
5
R
R
R
R
R
R
R∩A
Exercice 8
1
12
1
4
R∩B
R∩C
0
1
R
R
R∩D
Désignons par E l’événement « la personne est en état d’ébriété » ;
T l’événement « l’alcootest se révèle positif ».
L’énoncé nous donne les résultats suivants :
P (E ) = 0, 02 ; PE (T ) = 0, 96 ; PE (T ) = 0, 01.
On veut calculer PT (E ).
P (E ∩T )
On sait que : PT (E ) =
.
P (T )
▶ Calcul
de P (E ∩T ).
On a P (E ∩T ) = PE (T ) × P (E )
P (E ∩T ) = 0, 01× 0, 98
P (E ∩T ) = 0, 009 8..
▶ Calcul
de P (T ).
On a T = (T ∩ E ) ∪ (T ∩ E ).
Les événements T ∩ E et T ∩ E sont incompatibles donc
P (T ) = P (T ∩ E ) + P (T ∩ E ).
Soit P (T ) = PE (T ) × P (E ) + PE (T ) × P (E )
P (T ) = 0, 96 × 0, 02 + 0, 009 8
P (T ) = 0, 029.
On a donc PT (E ) =
0, 009 8
= 0, 337 9...
0, 029
La probabilité qu’une personne dont l’alcootest est positif ne soit pas en
état d’ébriété est environ égale à 0,338.
Corrigé Séquence 3 – MA01
73
© Cned - Académie en ligne
La probabilité trouvée peut paraître surprenante car il y a environ une chance
sur trois qu’une personne contrôlée positive ne soit pas en état d’ébriété !
Ce résultat est dû au faible taux (2 %) de conducteurs en état d’ébriété.
Un raisonnement identique nous donne :
PT (E ) =
P (E ∩T )
P (E ∩T )
=
P (T )
P (T ∩ E ) + P (T ∩ E )
PT (E ) =
0, 95 × 0, 01
0, 96 × 0, 05 + 0, 01× 0, 95
PT (E ) = 0,165 2... .
La probabilité des faux
positifs est PT (E ) = 0,165.
On fait de même pour les faux négatifs.
PT (E ) =
P (E ∩T )
P (E ∩T )
=
P (T )
P (T ∩ E ) + P (T ∩ E )
PT (E ) =
0, 05 × 0, 04
0, 05 × 0, 04 + 0, 95 × 0, 99
PT (E ) = 0, 002 1... .
La probabilité des faux
négatifs est PT (E ) = 0, 002.
La probabilité pour une personne dont l’alcootest est positif de ne pas être
en état d’ébriété reste encore assez forte, bien qu’elle ait diminué de moitié
environ (de 33 % à 16,5 %).
Par contre la probabilité pour une personne dont l’alcootest est négatif d’être
en état d’ébriété est assez faible.
On observe que le test est plus fiable dans celui des deux cas où la probabilité
p = P(E) est la plus grande.
Exercice 9
Lorsque le logiciel de programmation utilisé ou la calculatrice le permet, on peut
utiliser une seule boucle en faisant trier la liste des dates d’anniversaire. Il suffit
alors de comparer chaque date à la suivante.
Variables
dates : tableau des trente jours d’anniversaire
trouvé : un booléen qui indique si deux dates coïncident.
k : un compteur de boucles.
Initialisation
Pour k de 1 à 30
dates[k] prend une valeur entière aléatoire comprise entre 1 et 365 inclus
Trier la liste du plus petit au plus grand
trouvé prend la valeur faux
74
© Cned - Académie en ligne
Corrigé Séquence 3 – MA01
Traitement
Pour k de 1 à 29
Si dates[k] = dates[k+1] alors
trouvé prend la valeur vrai
Sortie
Affiche trouvé
Le logiciel Algobox ne permet pas de trier de listes. Ci-dessous les programmes
correspondant à l’algorithme précédent.
Casio
Exercice 10
Texas Instruments
L’expérience aléatoire est formée à partir de trois expériences aléatoires indépendantes. Elles ne sont pas identiques comme celles qui ont été utilisées en
Première ES, mais on procède de même : on utilise donc la loi de probabilité telle
que la probabilité d’une liste de résultats soit le produit des probabilités des
résultats partiels qui la constituent.
On note UC l’événement « on obtient 1 avec le dé cubique », UO l’événe-
ment « on obtient 1 avec le dé octaédrique », U D l’événement « on obtient 1
avec le dé dodécaédrique ».
1 1 1
1
On a P (U ) = P (UC ) × P (U O ) × P (U D ) = × × =
.
6 8 12
576
Avec des notations analogues on a :
2 4 8 64 1
P (Q ) = P (QC ) × P (QO ) × P (QD ) = × × =
= .
6 8 12 576 9
Soit X la variable aléatoire égale au gain algébrique d’un jeu.
La loi de probabilité de X est :
xi
P (X = xi )
E (X ) = 3×
3
1
576
−1
64
576
0,01 x n
1
64 511
1−
−
=
576 576 576
1
64
511 5,11 n − 61
− 1×
+ 0,01× n ×
=
.
576
576
576
576
Corrigé Séquence 3 – MA01
75
© Cned - Académie en ligne
Le jeu est favorable au joueur lorsque E ( X ) > 0 ce qui équivaut à
61
61
. Comme
5,11 n − 61 > 0 c’est-à-dire n >
≈ 11, 93 la plus petite
5,11
5,11
valeur de n est égale à 12.
Exercice 11
Le candidat répète 10 fois la même épreuve à 2 issues possibles :
1
S « le candidat répond correctement à la question posée » ; P (S ) = p =
3
puisque le candidat répond au hasard.
2
S « le candidat répond mal » ; P (S ) = 1− P (S ) = 1− p = .
3
Les réponses à chaque question sont indépendantes.
On est donc en présence d’une suite de 10 épreuves de Bernoulli.
Soit X le nombre de réponses exactes. X suit la loi binomiale de paramètres 10
1
et .
3
Pour être reçu, il faut répondre au moins à 8 questions. La probabilité d’être reçu
est donc :
P ( X = 8 ) + P ( X = 9 ) + P ( X = 10 )
10   1 8  2 2 10   1 9  2 1 10   1 10  2 0
=  ×   ×   +  ×   ×   +  ×   ×  
 3
 8   3   3   9   3   3  10   3 
 1 8 
4
1 2 1
67
=   ×  45 × + 10 × × +  =
≈ 0,003.
 3 
9
3 3 9  19 683
76
© Cned - Académie en ligne
Corrigé Séquence 3 – MA01
Corrigé des exercices de synthèse
de la séquence 3
Exercice I
Une fabrique artisanale de jouets en bois vérifie la qualité de sa production avant
sa commercialisation.
Chaque jouet produit par l’entreprise est soumis à deux contrôles : d’une part
l’aspect du jouet est examiné afin de vérifier qu’il ne présente pas de défaut de
finition, d’autre part sa solidité est testée. Il s’avère, à la suite d’un grand nombre
de vérifications, que :
▶ 92 % des jouets sont sans défaut de finition ;
▶ parmi les jouets qui sont sans défaut de finition, 95 % réussissent le test de
solidité ;
▶ 2 % des jouets ne satisfont à aucun des deux contrôles.
On prend au hasard un jouet parmi les jouets produits. On note :
▶ F l’événement : « le jouet est sans défaut de finition » ;
▶ S l’événement : « le jouet réussit le test de solidité ».
Construction d’un arbre pondéré associé à cette situation.
)
1
b. On sait P (F ∩ S ) = P (F ) × P (S ) , soit 0 , 02 = 0 , 08 × PF (S ) , donc PF (S ) = .
F
4
c. On a aussi PF (S ) = 1− PF (S ) = 0 , 75 et on peut construire l’arbre pondéré :
(
a. Les données sont P ( F ) = 0 , 92, PF ( S ) = 0 , 95 et P F ∩ S = 0 , 02.
PF(S) = 0,95
S
PF(S) = 0,05
S
PF(S) = 0,75
S
PF(S) = 0,25
S
F
P(F) = 0,92
P(F) = 0,08
F
( ) et , comme F et F forment une partition de l’uniP (S ) = P (F ∩ S ) + P (F ∩ S )
= P (F ) × P (S ) + P (F ) × P (S )
a. On a S = (F ∩ S ) ∪ F ∩ S
vers, on a :
F
F
= 0, 92 × 0,, 95 + 0, 08 × 0, 75
P (S ) = 0, 934.
Corrigé Séquence 3 – MA01
77
© Cned - Académie en ligne
b. On cherche PS (F ).
On a PS (F ) =
P (F ∩ S ) P (F ) × PF (S ) 0, 92 × 0, 95
=
=
≈ 0, 936.
P (S )
P (S )
0, 934
a. La variable aléatoire B prend trois valeurs : 10 ; 5 et 0.
P (B = 10 ) = P (F ∩ S ) = P (F ) × PF (S ) = 0, 92 × 0, 95 = 0, 874 ;
()
P (B = 0 ) = P S = 1− P (S ) = 0, 066 ;
P (B = 5) = 1− P (B = 10 ) − P (B = 0 ) = 0, 06.
bi
10
5
0
P (B = bi )
0,874
0,06
0,066
b. E(B) = 10 ⫻ 0,874 + 5 ⫻ 0,06 + 0 ⫻ 0,066
E(B) = 9,04
Comme la quantité fabriquée est suffisamment importante pour que la
constitution du lot de 10 jouets puisse être assimilée à un tirage avec remise,
la variable aléatoire X suit la loi binomiale Ꮾ(10 ; 0,934 .
)
On cherche P ( X ≥ 8 ) et on a :
P ( X ≥ 8) = P ( X = 8) + P ( X = 9) + P ( X = 10)
10 
10 
10 
=   0,934 8 × 0,0662 +   0,934 9 × 0,066 +   0,93410
8 
9 
10 
P ( X ≥ 8) ≈ 0,9757.
Exercice II
Les données nous permettent de construire l’arbre pondéré.
0,96
T
P(T ∩ M) = 0,96p
0,04
T
P(T ∩ M) = 0,04p
0,01
T
P(T ∩ M) = 0,01 (1–p)
0,99
T
P(T ∩ M) = 0,99 (1–p)
M
p
1– p
M
Déterminons P (M ) =
T
P (T ∩ M )
.
P (T )
On a P (T ) = P (T ∩ M ) + P (T ∩ M )
P (T ) = 0, 96p + 0, 011
( − p)
P (T ) = 0, 95p + 0, 01.
78
© Cned - Académie en ligne
Corrigé Séquence 3 – MA01
D’où PT (M ) =
0, 96p
96p
=
.
0, 95p + 0, 01 95p + 1
▶ Posons
f (p ) =
96p
p
= 96 ×
.
95p + 1
95p + 1
Déterminons la dérivée.
(95p + 1) − 95p
96
f '( p ) = 96 ×
=
.
(95p + 1)2
(95p + 1)2
La fonction f est croissante sur [0 ; 1] car f '( p ) > 0.
p
0
1
f (p )
▶ Tableau
1
0
de valeurs.
p
f (p )
0,001 0,005
0,01
0,02
0,05
0,10
0,087 7 0,325 4 0,492 3 0,662 1 0,834 8 0,914 3
▶ Déterminons
PT (M ) =
0,20
0,96
P (T ∩ M )
.
P (T )
On a P (T ) = 1− P (T ) = 1− (0, 95p + 0, 01)
P (T ) = −0, 95p + 0, 99.
D’où PT (M ) =
▶ Posons
g (p ) =
0, 99(1− p )
99(1− p )
=
.
−0, 95p + 0, 99 99 − 95p
99(1− p )
.
99 − 95p
Déterminons la dérivée.
99( −99 + 95p + 95 − 95p )
−4 × 99
g '( p ) =
=
.
(99 − 95p )2
(99 − 95p )2
La fonction g est décroissante sur [0 ; 1] car g '( p ) < 0.
p
0
g (p )
▶ Tableau
p
g (p )
1
1
0
de valeurs.
0,001 0,005
0,01
0,02
0,05
0,10
0,999 9 0,999 8 0,999 6 0,999 2 0,997 9 0,995 5
0,20
0,99
Corrigé Séquence 3 – MA01
79
© Cned - Académie en ligne
On sait que P (M ) + P (M ) = 1, d'où P (M ) = 1− P (M ).
T
T
T
T
Posons h ( p ) = PT (M ).
D’où h ( p ) = 1− f ( p ).
Comme la fonction f est croissante sur [0 ; 1], h est décroissante sur [0 ; 1].
p
0
1
h (p )
▶ Tableau
1
0
de valeurs.
p
h (p )
0,001 0,005
0,01
0,02
0,05
0,10
0,912 3 0,674 6 0,507 7 0,337 9 0,165 2 0,085 7
0,20
0,04
On sait que P (M ) + P (M ) = 1, d'où P (M ) = 1− P (M ).
T
T
T
T
Posons k ( p ) = PT (M ).
D’où k ( p ) = 1− g ( p ).
Comme la fonction g est décroissante sur [0 ; 1], k est croissante sur [0 ; 1].
p
0
1
k (p )
▶ Tableau
1
0
de valeurs.
p
k (p )
0,001 0,005
0,01
0,02
0,05
0,10
0,000 04 0,000 2 0,000 4 0,000 8 0,002 1 0,004 5
0,20
0,01
Quelques commentaires
Probabilité
conditionnelle
PT (M )
Fonction
f
Variations de
la fonction
Valeurs prises
80
© Cned - Académie en ligne
( )
P M
T
g
( )
PT M
P (M )
(faux positifs)
(faux négatifs)
h = 1− f
k = 1− g
T
croissante sur
I = 0 ; 0, 2
de 0 à 0,96
Corrigé Séquence 3 – MA01
décroissante sur I décroissante sur I
toujours proches
de 1
de 1 à 0,04
croissante sur I
toujours proches
de 0
On remarque tout d’abord que la valeur diagnostique du test n’est pas une
notion intrinsèque au test lui-même : elle varie fortement suivant la probabilité p qui dépend de la population ciblée.
Si la population est une population à risque, p n’est pas faible et il n’y a pas
trop de « faux positifs » ; la positivité du test sera donc un élément important du diagnostic. Par contre, pour une maladie rare, un test de dépistage
systématique de toute une population aura l’inconvénient majeur de fournir
beaucoup de faux positifs. Le nombre des personnes non malades dont le test
est positif et, pour la société, le prix des tests de dépistage systématique, sont
des problèmes éthiques et économiques liés à la mise en place de tels tests.
Autres remarques
( )
probabilités conditionnelles PT (M ) et PT M varient en sens contraires.
▶ P M
est toujours proche de 1, ce qui est rassurant.
T
▶ Les
( )
probabilité qu’une personne soit malade alors que le test est négatif, PT (M ),
est donc toujours proche de 0.
▶ La
Exercice III
Le mathou n’ayant aucune mémoire, les essais successifs sont modélisés par
la succession d’épreuves répétées et indépendantes, et pour chacune d’elles
on utilise la loi équirépartie. On a donc :
1
a. P ( X = 1) = ,
6
b. P ( X = 2) =
5 1 5
× = ,
6 6 36
5
5


c. P ( X = 6 ) = 5  × 1  = 5 ≈ 0, 067.
 6   6 
66
6

d. P ( X ≤ 6 ) = 1− P ( X > 6 ) = 1− 5  ≈ 0, 665 car P ( X > 6 ) est la probabilité
 6 
que le mathou pousse une mauvaise porte à chacun des six premiers essais.
On note Tn l’événement « le mathou trouve le fromage au n-ième essai ».
Comme le mathou a une mémoire parfaite, s’il ne trouve pas le fromage le
nombre de portes entre lesquelles il va choisir diminue de 1 à chaque essai.
On obtient l’arbre suivant :
1er essai
1/6
T1
5/6
T1
2e essai
1/5
T2
4/5
T2
3e essai
1/4
3/4
T3
........
6e essai
........
T3
Corrigé Séquence 3 – MA01
81
© Cned - Académie en ligne
Les valeurs prises par la variable aléatoire Y sont donc 1, 2, 3, 4, 5 et 6.
Avant même de trouver la loi de Y, on peut donc savoir que P (Y ≤ 6 ) = 1,
effet le mathou fait six essais au maximum.
1
5 1 1
5 4 1
On trouve P (Y = 1) = , P (Y = 2) = × = , P (Y = 3) = × × =
6
6 5 6
6 5 4
5 4 3 2 1 1 1
…, P (Y = 6 ) = × × × × × = .
6 5 4 3 2 1 6
en
1
,
6
La loi de la variable aléatoire Y est donc la loi équirépartie.
Pour tester l’hypothèse selon laquelle les mathoux ont une mémoire, on peut
faire faire un très grand nombre d’essais à plusieurs mathoux et calculer
la fréquence de découverte du fromage en au plus six essais. Si cette fréquence est très proche de 0,67, on en déduira que les mathoux n’ont aucune
mémoire, si cette fréquence est proche de 1, on en déduira que ces animaux
ont une excellente mémoire.
Exercice IV
On rencontre une personne par hasard, on utilise donc la loi équirépartie sur
l’univers formé par l’ensemble des dates d’une année. L’événement A « avoir
la même date d’anniversaire que vous » contient une seule éventualité et
1
donc P ( A ) =
≈ 2, 7 × 10 −3.
365
Si les personnes sont plus nombreuses que les jours de l’année, il y a néces-
sairement plusieurs personnes dont les dates d’anniversaire coïncident. En
tenant compte du 29 février, on trouve qu’à partir de 367 personnes on est sûr
qu’il y a plusieurs anniversaires à la même date, la probabilité qu’au moins
deux personnes aient leur anniversaire le même jour est donc égale à 1.
Nommons ces 30 enfants e1, e2 ,... ,e 30 .
Soit D1et 2 l’événement « les enfants e1 et e2 ont des dates d’anniversaire
364
≈ 0, 997 car 2011 n’est pas une
différentes ». On a P (D1et 2 ) = 1− P ( A ) =
365
année bissextile.
Soit D1et 2et 3 l’événement « les enfants e1, e2 et e 3 ont des dates d’anniversaire différentes ».
D’où P (D1et 2et 3 ) = P (D1et 2 ) ×
363
car la probabilité que la date d’anniversaire
365
du troisième enfant soit différente des deux premières sachant que les deux
363
premières sont différentes est égale à
car les deux dates d’anniversaire
365
des deux premiers enfants sont exclues.
82
© Cned - Académie en ligne
Corrigé Séquence 3 – MA01
)
On a donc P (D1et 2et 3 =
364 363
×
.
365 365
)
On obtient de même que P (D1et 2et 3et 4 =
364 363 362
×
×
.
365 365 365
En appelant D l’événement « les anniversaires des 30 enfants sont tous à
des dates différentes » on obtient
30 −1= 29 fracttions
364 363 362
336
.
P D =
×
×
× ... ×
365 365 365
365
( )
Calcul de P (C )
L’événement C « deux anniversaires au moins coïncident » est l’événement
contraire de l’événement D
()
donc P (C ) = 1− P D = 1−
364 363 362
336
×
×
× ... ×
.
365 365 365
365
Pour calculer cette probabilité et déterminer l’entier N, on peut utiliser les
algorithmes suivants :
Calcul de P (C )
Détermination de N
Variables
k : compteur de boucles ;
n : un entier naturel ;
p : un nombre réel compris entre
0 et 1.
Initialisation
p =1
n = nombre de personnes du
groupe
Traitement
Pour k de 1 à n − 1
365 − k
mettre p ×
dans p
365
Sortie
Afficher 1− p
Variables
k : compteur de boucles ;
N : un entier naturel ;
p : un nombre réel compris entre
0 et 1.
Initialisation
p =1
Traitement
Pour k de 1 à 367
Tant que p < 0, 5
365 − k
mettre p ×
dans p
365
N =k
Sortie
Afficher N
On peut utiliser un tableur de la façon suivante.
On a rempli les colonnes A, B et C ; dans la cellule D2, on a recopié C2, c’est364
à-dire la fraction
≈ 0 , 997, dans la cellule D3 on a rentré la formule
365
364 363
D2*C3 c’est-à-dire
, et on l’a recopiée ce qui permet d’obtenir
×
365 365
364 363 362
336
dans la cellule D30. La colonne E donne donc les
×
×
× ... ×
365 365 365
365
probabilités de rencontrer au moins une coïncidence de dates d’anniversaire
en fonction du nombre de personnes du groupe.
Corrigé Séquence 3 – MA01
83
© Cned - Académie en ligne
Pour 30 enfants, on trouve P (C ) 0,706.
La probabilité P (C ) dépasse 0,5 à partir de N =23.
84
© Cned - Académie en ligne
Corrigé Séquence 3 – MA01
C orrigé Séquence 4
Corrigé des activités du chapitre 2
Activité 1
La moyenne d’accord, mais laquelle ?
La moyenne arithmétique de 5 et 15 est 10. La moyenne géométrique de 5 et
15 est 5 3 soit environ 8,7.
La moyenne arithmétique est la plus avantageuse pour Céleste.
Appelons a et b les notes d’un quelconque des élèves. La moyenne arithmé-
tique de cet élève est :
Conclusion
La moyenne arithmétique est toujours plus
grande ou égale à la moyenne géométrique,
donc c’est toujours la plus avantageuse,
indépendamment des notes de l’élève.
Activité 2
n
un
0
1
2
3
4
5
6
7
1
2
4
8
16
32
64
128
a +b
et la moyenne géométrique est G = a × b .
2
a +b
1
− ab = ( a − b )2 ≥ 0
Comme A − G =
2
2
puisqu’un carré est toujours positif, on a A ≥ G .
A=
Propriétés des suites
géométriques de premier terme 1 et de raison
q strictement positive
M3
Représentation graphique
de la suite (un )
M2
M1
1
0
M0
1
Corrigé Séquence 4 – MA01
85
© Cned - Académie en ligne
Propriétés de la suite (u )
n
a. u1+ 2 = u 3 = 8 et u1 × u 2 = 2 × 4 = 8.
b. u 3+ 4 = 128 = u 3 × u 4 .
Explication : um + n = 2m + n = 2m × 2n = um × un .
c. Le résultat du b) s’écrit f (m + n ) = f (m ) × f (n ).
du point C est égale à q fois l’ordonnée du point B.
▶ La moyenne arithmétique des abscisses des points A et C est égale à l’abscisse
du point B.
▶ La moyenne géométrique des ordonnées des points A et C est égale à l’ordonnée du point B.
▶ L’ordonnée
Activité 3
Prolongement à Z
a.
p
–3
–2
–1
M3
up = f (p )
2−3 =
1
1
= = 0,125
2 8
3
1
= 0, 25
4
1
2−1 = = 0, 5
2
2−2 =
0
20 = 1
1
21 = 2
2
4
3
8
M2
M1
M–3
M–2
M0
M–1
O
1
1 1 1
1
× = puis f ( −2 − 1) = f ( −3) = .
4 2 8
8
1
c. On calcule f ( 3) × f ( −2) = 8 × = 2 puis f ( 3 − 2) = f (1) = 2.
4
On obtient le même résultat.
b. On calcule f ( −2) × f ( −1) =
d. Calculons par exemple f ( −7) × f ( 3) et f ( −7 + 3).
1
1
1
f ( −7) × f ( 3) =
×8 =
et f ( −7 + 3) = f ( −4 ) = .
128
16
16
Le résultat est encore le même.
86
© Cned - Académie en ligne
Corrigé Séquence 4 – MA01
Pour cette question, on revient à q > 0.
Pour justifier que pour tous entiers relatifs m et n, on a f (m + n ) = f (m ) × f (n ),
on commence par fixer deux entiers relatifs m et n quelconques.
Puis on calcule séparément f (m + n ) et f (m ) × f (n ) , si on trouve le même
résultat c’est gagné !
D’après les règles connues sur les puissances entières (éventuellement négatives), on sait que :
f (m + n ) = 2m + n = 2m × 2n (ici, m ou/et n peuvent être négatifs).
Comme nous avons prolongé f à de sorte que la définition f (k ) = 2k
reste vraie pour k négatif, on a encore f (m ) = 2m et f (n ) = 2n ; donc
2m × 2n = f (m ) × f (n ).
Par conséquent, f (m + n ) = f (m ) × f (n ).
Puisque les entiers relatifs m et n ont été choisis quelconques, on peut
conclure que : pour tous entiers relatifs m et n, on a f (m + n ) = f (m ) × f (n ).
Justifions que f est strictement positive ; autrement dit, que pour tout entier
relatif p , f ( p ) > 0.
On commence par remarquer que si m est un entier positif alors f (m ) = 2m
est strictement positif, puis que si m est un entier négatif alors −m est un
1
entier strictement positif, donc f (m ) = 2m =
est strictement positif.
−m
2
On conclut que, dans tous les cas, pour tout entier relatif p , f ( p ) > 0.
Activité 4
Prolongement à Première étape
La fonction f :
RaR
p a 2p
est alors définie en … ; –3 ; –2,5 ;
–2 ; –1,5 ; –1 ; –0,5 ; 0 ; 0,5 ; 1 ;
1,5…
Autrement dit, f est définie pour
m
tous les réels du type
avec
2
m ∈Z.
1
O
1
Corrigé Séquence 4 – MA01
87
© Cned - Académie en ligne
Le tableau précédent peut ainsi s’écrire :
Abscisse
m
−3 =
2
Ordonnée
 m
f
 2
−6
2
−2, 5 =
−5
2
−2 =
1
1
1
8
32
4
−4
2
−1, 5 =
−3
2
−1 =
−2
−0 , 5 = −
2
2
8
0=
2
1
1
1
1
0
2
0, 5 =
1
1
2
1=
2
2
1, 5 =
2
2
3
2
2=
4
2
2, 5 =
4
8
5
2
3=
32
6
2
8
2
Ecrivons différemment les nombres du tableau précédent… pour y voir plus
clair.
5
−5
1
On calcule 2 = 32 puis 2
=
.
32
Ces valeurs sont dans le tableau précédent.
( )
( )
On en déduit le tableau :
Abscisse
m
2
Ordonnée
f
 m
 2
−6
−5
−4
−3
−2
−1
0
1
2
3
4
5
6
2
2
2
2
2
2
2
2
2
2
2
2
2
( 2) ( 2) ( 2) ( 2) ( 2) ( 2) ( 2) ( 2) ( 2) ( 2) ( 2) ( 2) ( 2)
−6
−5
−4
−3
−2
−1
0
1
2
3
4
5
6
× 2 × 2 × 2 × 2 × 2 × 2 × 2 × 2 × 2 × 2 × 2 × 2
Dans le cas présent (où q = 2), à la lecture de la 2e ligne du tableau précédent, une règle semble se dégager. L’égalité suivante résume cette règle :
m
f =
 2
( 2)
m
.
Le point M 5 dont l’abscisse est la moyenne (arith-
M3
2
métique) des abscisses des points M2 et M 3 a
pour ordonnée
32 alors que le milieu du seg-
ment [M 2M 3 ] a pour ordonnée la moyenne (arith-
I
M5/2
M2
métique) des ordonnées de ces points c’est-à-dire
4+8
= 6. On vérifie que 32 < 6 (la moyenne
2
géométrique est plus petite que la moyenne arithmétique). Le milieu (appelé I) du segment [M2M 3 ]
est donc situé au-dessus du point M 5 .
2
1
◀
O
88
© Cned - Académie en ligne
1
Corrigé Séquence 4 – MA01
Plaçons ces points sur le graphique.
Deuxième étape
En réitérant le processus de dichotomie on complète le tableau suivant :
Point
abscisse
M −3
–3
M 11 M 5 M 9
−
−
−
4
11
−
4
2
5
−
2
ordonnée
0,125
0,15
Point
M1
M1
M3
4
2
4
abscisse
ordonnée
−
0,18
1
1
3
4
2
4
1,19
1,41
1,68
M −2 M 7
−
4
9
−
–2
4
0,21
M1
1
2
0,25
M
−
4
7
−
4
0,30
3
M 5
2
4
−
3
−
2
0,35
M −1 M 3 M 1
−
5
–1
4
0,42
M5
M3
M7
4
2
4
5
3
7
4
2
4
2,38
2,83
3,36
0,5
M2
2
4
−
−
4
3
4
0,59
−
2
1
2
0,71
M9
M5
4
2
M 1
−
−
M0
4
1
0
4
0,84
1
M 11
M3
4
9
5
11
4
2
4
4,76
5,66
6,73
3
8
On place ensuite les nouveaux points du nuage sur un
nouveau graphique.
Ceci peut s’obtenir directement avec Geogebra à l’aide
de la seule commande :
L = Séquence[(i,2^i),i,-3,3,0.25]
Dans cette expression, il est demandé au logiciel de
construire une liste (appelée L) de points de coordoni
nées (i , 2 ) pour toutes les valeurs de i comprises
entre −3 et 3 distantes de 0,25.
1
O
1
Corrigé Séquence 4 – MA01
89
© Cned - Académie en ligne
Corrigé des exercices
d’apprentissage du chapitre 2
Exercice 1
Ecrivons sous la forme a b ou a b c d avec a ≠ c , les expressions suivantes :
a. 23,7 × 25 = 23,7+ 5 = 28,7.
b. 0, 4 3 × 0, 4 −2,5 = 0, 4 3 − 2,5 = 0, 40,5.
3, 9 × 3, 9 −2 = 3, 90,5 × 3, 9 −2 = 3, 90,5 − 2 = 3, 9 −1,5.
c.
3−8,1 × 22
d.
2
3
Exercice 2
× 2−2,1 = 3−8,1 × 3−2 × 22 × 2−2,1 = 3−10,1 × 2−0,11.
f est une fonction exponentielle, donc il existe un nombre réel q > 0 tel que pour
tout réel x, f ( x ) = q x .
Comme f (2) = 1, 2 le réel q doit vérifier q 2 = 1, 2. Comme en plus q > 0 , nécesx
sairement q = 1, 2.
 1
x
2
Finalement, la fonction f vérifie pour tout réel x, f ( x ) = 1, 2 =  1, 2 


x


2
soit f ( x ) = 1, 2 .
( )
Autrement dit, f est la fonction exponentielle de base
1, 2.
Exercice 3
5,5
5
4,5
4
3,5
x
x–0,8
3
2,5
2
1,5
1
x
0,8
x
0,5
0
90
© Cned - Académie en ligne
1
2
Corrigé Séquence 4 – MA01
3
4
5
6
7
8
9
10
11
12
13
14
15
On observe que les deux fonctions sont décroissantes mais que la fonction exponentielle de base 0,8 décroit (beaucoup) plus vite et ceci est d’autant plus vrai
que l’intervalle d’observation contient de grandes valeurs de x.
Exercice 4
a. Notons f la fonction cube. L’équation x 3 = 1 équivaut à f ( x ) = 1, ou encore
f ( x ) = f (1) puisque f (1) = 13 = 1.
Comme la fonction « cube » est continue et strictement croissante sur , il
ne peut exister, au plus qu’un nombre réel x tel que f ( x ) = f (1). Comme, de
manière évidente, le réel x = 1 convient, c’est que c’est le seul. Conclusion :
L’ensemble des solutions de l’équation x 3 = 1 est ᏿ = 1 .
{}
b. De la même façon, l’équation x 3 = 2 équivaut à f ( x ) = 2, qui n’admet elle
aussi qu’une solution réelle.
Conclusion
Pour obtenir une valeur approL’ensemble des solutions de l’équation
chée de cette solution vous pou 1
vez, à l’aide d’une calculatrice,
 
3
x
=
2
est ᏿ = 2 3 .
calculer 21/3 (cette écriture est
 
justifiée à la question ).
c. Des arguments analogues aux précédents montrent que l’équation x 3 = −8
admet une unique solution réelle. Comme −2 convient, on conclut que l’ensemble des solutions de l’équation x 3 = −8 est ᏿ = −2 .
 1


3
d. De même, l’ensemble des solutions de l’équation x = −5 est ᏿ = −5 3 .


Pour déterminer le réel strictement positif x tel que x −1,5 = 2 , nous allons
{ }
( )
utiliser la formule x a
soit b = −
1
.
1, 5
( )
Ainsi, x −1,5
L’équation x
− 1
15
,
−1,5
b
= x ab avec a = −1, 5 et b choisi de sorte que ab = 1
= x 1.
( )
= 2 devient donc x
−1,5
− 1
15
,
= ( 2)
− 1
15
,
soit x = ( 2 )
−1
15
,
.
Conclusion
On peut simplifier cette écriture :
( 2)
− 1
1 −2
15
, = (22 ) 3
=2
1×( − 2 )
2
3
−1
= 2 3.
L’ensemble des solutions de l’équation
 − 1 
x −1,5 = 2 est ᏿ = 2 3  .


Corrigé Séquence 4 – MA01
91
© Cned - Académie en ligne
Exercice 5
Il ne peut s’agir que de la courbe
(Ꮿ 3 )
puisque sa fonction f associée ne
vérifie pas f (0 ) = 1 et que c’est la seule.
Parmi les courbes restant à attribuer, la courbe
d’une fonction décroissante.
(Ꮿ 2 ) est la seule à être celle
Comme une fonction exponentielle de base q > 0 est décroissante seulement lorsque q < 1 , la courbe Ꮿ 2 est celle de la fonction exponentielle
2
de base .
3
( )
(Ꮿ 1) a une ordonnée d’environ 2,5.
Le point d’abscisse 1 de la courbe (Ꮿ 4 ) a une ordonnée d’environ 3.
Par conséquent, la fonction g associée à (Ꮿ 1) vérifie g (1) = 2, 5. Comme g
Le point d’abscisse 1 de la courbe
est une fonction exponentielle (notons q, sa base) pour tout réel x on sait que
g ( x ) = q x . En particulier, g (1) = q 1 = q . Donc q = 2, 5.
(Ꮿ 1) est donc associée à la fonction exponentielle de base 2,5.
La courbe (Ꮿ 4 ) est donc associée à la fonction exponentielle de base 3.
Remarquons que la courbe (Ꮿ 4 ) passe par le point de coordonnées (1 ; 3)
La courbe
donc sa fonction associée (notons la h) vérifie h (1) = 3 ce qui est bien le cas
puisque h ( x ) = 3x .
92
© Cned - Académie en ligne
Corrigé Séquence 4 – MA01
Corrigé de l’activité du chapitre 3
Activité 5
Laquelle choisir ?
( )
Appelons A le point de coordonnées 0 ; 1 . Pour toute base q > 0, la foncx
tion exponentielle de base q est définie par f ( x ) = q . Comme, pour toute
base q > 0, f (0 ) = q 0 = 1, la courbe de f passe par A.
0.5^x 0.3^x
16
3^x 2^x
1.5^x
14
12
Conclusion
Toutes les courbes des fonctions
exponentielles passent par le
point A.
10
8
6
4
0.9^x
1.1^x
2
1.1^x
2^x
1.5^x 3^x
–10
–8
–6
–4
Une équation de la droite
soit y = x + 1.
0 A
2
4
6
8
10
Ᏸ de pente 1 passant par A est : y − 1 = 1× ( x − 0)
9
8
a. Pour tracer la droite Ᏸ on peut entrer
simplement dans le champ de saisie
y = x + 1 et pour la fonction exponentielle on peut
entrer y = e^x.
7
6
5
4
3
2
q = 2.7
1
–3 –2 –1 0
–2
1
2
3
4
5
6
7
b. En faisant varier q à l’aide du curseur, on peut faire
en sorte que la tangente à la courbe de la fonction
exponentielle de base q se superpose avec la droite
Ᏸ ; cette superposition étant obtenue lorsque q
est environ égal à 2,7.
Corrigé Séquence 4 – MA01
93
© Cned - Académie en ligne
c. Puisque, par définition, f ′(a ) est le coefficient directeur de la tangente à la
courbe de la fonction f au point d’abscisse a, on a : f ′(0 ) ≈ 2, 7.
d. On peut lire f ′(0 ) sur l’équation de la tangente en x = 0 à la courbe de la
fonction exponentielle (cette équation apparait dans la fenêtre algèbre), et
f ′(0 ) est le coefficient de « x » dans l’équation de la tangente. Par exemple,
pour q = 0,1 on lit f ′(0 ) ≈ −2, 3 comme on peut le voir sur le graphique cidessous.
10
9
8
7
6
5
4
3
2
q = 0.1
1
–4 –3
f '(0 )
–2
–1 0
−2,30 −0, 69 −0,11
1
2
3
4
5
6
7
0, 26
0,41
0, 59
0, 69
0,79
0,83
1,3
1,5
1,8
2
2,2
2,3
q
0,1
0,5
f '(0 )
0,88
0,91
0,95
0,99
1,00
1,03
1,06
1,10
1,5
2,01
q
2,4
2,5
2,6
2,7
2,72
2,8
2,9
3
4,5
7,5
0,9
e. « La fonction exponentielle f de base q ≈ 2, 7 vérifie f ′(0 ) = 1 ».
94
© Cned - Académie en ligne
8
Corrigé Séquence 4 – MA01
9
Corrigé des exercices
d’apprentissage du chapitre 3
Exercice 6
A = e2 × e −1,5 × e = e2 × e −1,5 × e0,5 = e2−1,5+ 0,5 = e1 = e.
B=
e −1 × e5,5
e×e
0 ,8
= e −1+ 5,5 − (1+ 0,8 ) = e2,7 .
2
2
 1
C =   = e −3 = e( −3) × 2 = e −6 .
 e3 
( )
D=
ex
e
−2x
× e2 = e x − ( −2x )+ 2 = e 3x + 2.
E = e x × (e − x − 1) = e x × e − x − e x × 1 = e x − x − e x = e0 − e x = 1− e x .
Exercice 7
a. Soit x un réel fixé.
On calcule
( )
(e x + e − x )2 − (e x − e − x )2 = e x
2
( ) ( )
− (e − 2 + e
)= 4
( )
2
2 
2
+ 2 × ex × e− x + e− x −  ex − 2 × ex × e− x + e− x 


= e2x + 2 + e −2x
−2x
2x
Conclusion
Pour tout réel x, (e x + e − x )2 − (e x − e − x )2 = 4.
b. On calcule
ex
ex + e− x
=
ex
e x (1+ e − x e − x )
=
1
1+ e −2x
.
Conclusion
Pour tout réel x,
ex
ex + e− x
=
1
1+ e −2x
.
Corrigé Séquence 4 – MA01
95
© Cned - Académie en ligne
Corrigé des activités du chapitre 4
Activité 6
Le clone de sa dérivée
En faisant varier q, la valeur de q pour laquelle les deux courbes se super-
posent est environ 2,7.
(on retrouve environ la même valeur que celle trouvée à l’activité 5 lorsque
nous cherchions la base q pour laquelle la fonction exponentielle de base q
vérifiait f ′(0 ) = 1).
Le fait qu’il soit possible de superposer la courbe d’une fonction et la courbe
de sa fonction dérivée est déjà un résultat remarquable en soi. Nous remarquons aussi (graphiquement) que ceci est possible pour une fonction exponentielle, et il semble que ça ne le soit que pour une seule de celles-ci.
Ceci signifie que exp ′( x ) = exp( x ) pour tout réel x.
Activité 7
Stoïque par dérivation
exp(0 + h ) − exp(0 )
.
h
Comme toutes les fonctions exponentielles (et parce qu’elles prolongent les
suites géométriques de 1er terme u 0 = 1) la fonction exp vérifie exp(0 ) = 1 .
Mais la fonction exp vérifie en plus exp ′(0 ) = 1.
Par définition du nombre dérivé, exp ′( 0 ) = lim
h →0
eh − 1
La limite précédente s’écrit donc 1 = lim
.
h →0 h
Nous avons vu que exp(0 ) = 1 et que exp ′(0 ) = 1 , on en déduit bien que :
exp ′(0 ) = exp(0 ).
La relation fonctionnelle des fonctions exponentielles s’écrit :
e x + y = e x × e y , pour tous réels x et y.
Donc e 4 ,7+ h − e 4 ,7 = e 4 ,7 × eh − e 4 ,7 = e 4 ,7 × (eh − 1).
exp( 4 , 7 + h ) − exp( 4 , 7)
Par définition du nombre dérivé : exp ′( 4 , 7) = lim
.
h
h →0
D’après la factorisation précédente :
exp( 4 , 7 + h ) − exp( 4 , 7)
e 4 ,7 × (eh − 1)
= lim
h
h
h →0
h →0
exp ′( 4 , 7) = lim
h


4 ,7 ( e − 1)
C’est-à-dire exp ′( 4 , 7) = lim  e ×
.
h 
h →0 
96
© Cned - Académie en ligne
Corrigé Séquence 4 – MA01
Comme e 4 ,7 ne dépend pas de h, on peut écrire
 (eh − 1) 
exp ′( 4 , 7) = e 4 ,7 × lim 
.
h →0  h 
On reconnaît la limite du .
Donc exp ′( 4 , 7) = e 4 ,7 × 1 ; autrement dit : exp ′( 4 , 7) = exp(4,7).
On peut penser que le résultat précédent se généralise pour un réel a quel-
conque.
On conjecture que pour tout réel a, exp ′(a ) = exp(a ).
Activité 8
Comportements asymptotiques (limites)
a. La suite (en ) est géométrique de raison q = e et de 1er terme 1. Comme
e >1 , lim en = +∞ .
n →+∞
b. Pour tout n ∈ , on note An le point d’abscisse n de la courbe de la fonction
exponentielle.
Les coordonnées de A 100 sont environ (100 ; 2,69 × 1043 ) ; celles de A 200
sont environ (200 ; 7,23 × 1086 ).
Avec le logiciel Geogebra, on observe que l’ordonnée en du point An grandit très vite (bien plus vite que n) lorsque n prend de grandes valeurs. Ceci est
cohérent avec lim en = +∞ .
n →+∞
1
et de 1er terme 1.
e
n
n
 1
1
Comme < 1 , lim e −n = lim e −1 = lim   = 0.
e
n →+∞
n →+∞
n →+∞  e 
–1
a. La suite (e −n ) est géométrique de raison q = e =
( )
b. Pour tout n ∈ , on note Bn le point d’abscisse −n de la courbe de la
fonction exponentielle.
Les coordonnées de B100 sont environ ( −100 ; 3, 72 × 10 −44 ) ; celles de B200
sont environ ( −200 ;1, 38 × 10 −87 ).
Avec le logiciel Geogebra, on observe que l’ordonnée e −n du point Bn diminue et se rapproche très vite de zéro en lui restant supérieure (bien plus
« vite » que n ne grandit) lorsque n prend des valeurs de plus en plus grandes.
On pense donc que lim e −n = 0.
n →+∞
Corrigé Séquence 4 – MA01
97
© Cned - Académie en ligne
Corrigé des exercices
d’apprentissage du chapitre 4
Exercice 8
Une équation de la tangente (T) à la courbe de la fonction exponentielle au point
d’abscisse 1 est :
y = exp ′(1) × ( x − 1) + exp(1) ; soit encore y = e( x − 1) + e ; soit enfin y = e × x .
La droite (T) passe par l’origine O du repère.
Exercice 9
a. On a successivement e 3x +1 = e x équivaut à e 3x +1 × e − x = e x × e − x équi-
vaut à e 3x +1− x = e x − x équivaut à e2x +1 = e0 équivaut à 2x+1= 0 équivaut à x = − 1 .
2
L’ensemble des solutions de l’équation e 3x +1 = e x est donc
(e )
b. On transforme
2
x
e
2
2
ex
=
= e x −1 .
e
(e )
=
x
2
D’où, l’équation e x
{ }
᏿ = − 21 .
2
2
2
équivaut à e x −1 = e x soit encore
e
e
= 1 , soit finalement e −1 = 1 ce qui est toujours faux (quelque
soit x). Donc ᏿ = ∅.
x 2 −1− x 2
2
c. e x =
1
2
2
équivaut à e x × e x = 1 soit encore e x + x = 1 .
ex
2
Comme 1 = e0 , l’équation équivaut encore à e x + x = e0 ou bien à :
x 2 + x = 0 soit finalement à : x ( x + 1) = 0. Donc
᏿ = {−1 ; 0}.
( ) ( )
2
d. L’équation e2x − 2e x + 1 = 0 équivaut à e x − 2 e x + 1 = 0. Ceci signifie que le nombre e x est une solution de l’équation du second degré
t 2 − 2t + 1 = 0. Or, cette dernière équation équivaut à (t − 1)2 = 0 ; elle
n’admet donc qu’une seule solution qui est t = 1. Par conséquent, l’équation e2x − 2e x + 1 = 0 est équivalente à e x = 1 soit aussi e x = e0 , soit
enfin x = 0. Conclusion : ᏿ = 0 .
{}
e. L’équation
x
e =e
− x +1
ou enfin à 1 x = − x + 1.
2
2
Soit finalement x = . Conclusion :
3
98
© Cned - Académie en ligne
Corrigé Séquence 4 – MA01
( )
1
équivaut à e x 2 = e − x +1 ou encore à
2
3
᏿ =  .
1x
2
e = e − x +1 ,
Exercice 10
a. L’inéquation e5x + 2 ≤ e7x − 4 équivaut à 5x + 2 ≤ 7x − 4 puisque la fonction
exponentielle conserve l’ordre des nombres réels. Comme 5x + 2 ≤ 7x − 4
équivaut à x ≥ 3 , l’ensemble des solutions de l’inéquation est :
᏿ =  3 ; + ∞  .
( ) − e1 ≥ 0 équivaut à e
3
1
soit encore à e 3x ≥ e −1 .
e
Puisque la fonction exponentielle conserve l’ordre, cette dernière inéquation
équivaut à 3x ≥ −1 soit finalement x ≥ − 1 .
b. L’inéquation e x
3x
≥
3
L’ensemble des solutions de l’inéquation est donc
᏿
=  − 1 ; + ∞  .
 3

c. L’inéquation e2x − e1+ x < 0 équivaut à e2x < e1+ x . Puisque la fonction exponentielle conserve l’ordre, cette dernière inéquation équivaut à
2x < 1+ x soit x < 1.
L’ensemble des solutions de l’inéquation
᏿ =  −∞ ; 1 .
e2x − e1+ x < 0
est donc
d. Pour résoudre l’inéquation e x + e − x > 2 nous commençons par chercher à
ne faire apparaitre que des exposants qui soient des entiers positifs de e x .
Pour cela, on factorise e x + e − x ainsi e x + e − x = e − x (e2x + 1) de sorte
que l’inéquation e x + e − x > 2 devienne e − x (e2x + 1) > 2 ; puis en multipliant chacun des deux membres de cette inéquation par le même nombre
strictement positif e x on obtient l’inéquation équivalente suivante :
e x × e − x (e2x + 1) > e x × 2
soit encore e2x + 1 > 2e x
( ) − 2(e ) + 1> 0.
soit enfin e x
2
x
Ceci signifie que e x est solution de l’inéquation t 2 − 2t + 1 > 0. Résolvons
d’abord cette dernière inéquation. Comme t 2 − 2t + 1 = (t − 1)2 qui est un
carré toujours positif ou nul, l’inéquation (t − 1)2 > 0 ne sera pas vérifiée si
(t − 1)2 = 0 , c’est-à-dire si t = 1. Donc l’ensemble des solutions de l’inéquation t 2 − 2t + 1 > 0 est ᏿ = ] − ∞ ; 1[∪]1 ; + ∞ [. Par conséquent, e x est solution de l’inéquation t 2 − 2t + 1 > 0 si et seulement si e x ∈] − ∞ ; 1[∪]1 ; + ∞ [,
autrement dit, si et seulement si e x ≠ 1.
Comme e x = 1 ⇔ x = 0, l’ensemble des solutions de l’inéquation
e x + e − x > 2 est ᏿ = ] − ∞ ; 0[∪]0 ; + ∞ [.
Corrigé Séquence 4 – MA01
99
© Cned - Académie en ligne
Exercice 11
a. On a : h ( x ) = eu ( x ) avec u ( x ) = x 2 + 1 x − 3.
2
u qui est une fonction polynôme est dérivable sur et u ′( x ) = 2x + 1 .
2
Le théorème 3 s’applique : h est donc une fonction dérivable sur
h ′( x ) = u ′( x )eu ( x ) = (2x +
2 1
1) × ex + 2 x − 3 ,
2
et
pour tout réel x.
b. On a : k ( x ) = eu ( x ) avec u ( x ) = 7x 3 + 5x 2 − 3x .
u qui est une fonction polynôme est dérivable sur et u ′( x ) = 21x 2 + 10 x − 3.
Le théorème 3 s’applique : k est donc une fonction dérivable sur et
3
2
k ′( x ) = u ′( x )eu ( x ) = (21x 2 + 10 x − 3) × e7x + 5x − 3x , pour tout réel x.
Exercice 12
a. Calculons la dérivée de la fonction f définie pour tout x ∈[0 ; + ∞ [ par
f (x ) = e x .
f est une fonction dérivable sur ]0 ; + ∞ [ (pas en zéro, a priori) et pour
tout x ∈ ]0 ; + ∞ [ , on peut écrire f ( x ) = eu ( x ) où u ( x ) = x . On sait que
1
1
f ′( x ) = u ′( x )eu ( x ) avec u ′( x ) =
; donc f ′( x ) =
e x.
2 x
2 x
b. Calculons la dérivée de la fonction g définie pour tout x ∈ ]0 ; + ∞ [ par
1
g(x ) = e x .
g est une fonction dérivable sur ]0 ; + ∞ [ et pour tout x ∈ ]0 ; + ∞ [ , on peut
1
écrire g ( x ) = eu ( x ) où u ( x ) = .
x
−1
−1 x1
u(x )
On sait que g ′( x ) = u ′( x )e
avec u ′( x ) =
; donc g ′( x ) = e ou
1
x2
x2
ex
encore g ′( x ) = − .
x2
2
c. Calculons la dérivée de la fonction h définie pour tout réel x par h ( x ) = e −9 x .
h est une fonction dérivable sur et pour tout réel x, on peut écrire
h ( x ) = eu ( x ) où u ( x ) = −9 x 2.
On sait que h '( x ) = u ′( x )eu ( x ) avec u ′( x ) = −18 x ;
2
donc h '( x ) = −18 x e−9 x .
d. Calculons la dérivée de la fonction k définie pour tout réel x
à l’aide du logiciel XCAS.
100
© Cned - Académie en ligne
Corrigé Séquence 4 – MA01
−3x + 2
par k ( x ) = e 7x + 5
Exercice 13
La fonction f est dérivable sur
et pour tout x réel, f ′( x ) = e x + 1.
Comme pour tout réel x, on sait que e x > 0 a fortiori, e x + 1 > 0
soit pour tout réel x, f ′( x ) > 0.
D’après un théorème du cours, f est une fonction strictement croissante sur
.
Exercice 14
La fonction f est dérivable sur
et pour tout x réel, f ′( x ) = e x + xe x .
Pour étudier le signe de f ′( x ) on factorise : f ′( x ) = e x ( x + 1).
Comme pour tout réel x, e x > 0, le signe de f ′( x ) est celui de x + 1.
Le tableau suivant en découle :
x
f '( x )
−∞
−1
–
f (x )
Exercice 15
La fonction f définie sur
0
+∞
+
−1
e
par f ( x ) =
20 x
− 1 est dérivable et pour tout réel x,
e 3x
20e 3x − 20 x × 3 × e 3x 20e 3x (1− 3x ) 20(1− 3x )
f ′( x ) =
.
=
=
(e 3x )2
(e 3x )2
e 3x
Corrigé Séquence 4 – MA01
101
© Cned - Académie en ligne
Le signe de f ′( x ) est donc le même que celui de 1− 3x .
Le tableau suivant en découle :
x
1
3
−∞
+
f '( x )
+∞
0
–
20
−1
3e
f (x )
A l’aide du logiciel Geogebra, on entre au clavier dans le champ de saisie :
20x/exp(3x)-1
1,6
1,4
1,2
1
0,8
0,6
0,4
0,2
0
–0,4 –0,2 0 0,1
–0,2
0,3
0,5
0,7
0,9
1,1
1,3
1,5
1,7
1,9
2,1
2,3
2,5
2,7
2,9
–0,4
–0,6
–0,8
–1
La courbe de f coupe l’axe des abscisses en deux points d’abscisses
x 1 et x 2 avec x 1 < x 2.
Par lecture sur le graphique précédent, on trouve x 2 ≈ 1 environ.
102
© Cned - Académie en ligne
Corrigé Séquence 4 – MA01
Lorsqu’on demande au logiciel de calculer f (1) , la valeur a = −0, 004 apparaît dans la fenêtre Algèbre ; après avoir, si besoin, ajusté le nombre de décimales à 3 (Menu Option > Arrondi). Il semble donc que f (1) < 0. Comme la
fonction f est décroissante sur l’intervalle [ 1 ; + ∞ [, l’ordre de f (1) et f ( x 2 )
3
est l’ordre contraire de celui de 1 et x 2. Puisque f ( x 2 ) = 0, on a f (1) < f ( x 2 )
donc, nécessairement 1 > x 2 c’est-à-dire, x 2 < 1.
Exercice 16
(d’après bac ES – Polynésie – juin 2011)
VRAIE. On peut même affirmer que la solution de f ( x ) = 0 est située dans
l’intervalle ] − ∞ ; 1[.
FAUX. Par exemple, si x appartient à l’intervalle ]1 ; 6[⊂ ]1 ; + ∞ [, f ′( x ) < 0 ;
puisque f est décroissante sur cet intervalle.
VRAIE. On calcule g ( 6 ) = ef ( 6 ) = e 3 , puisqu’on lit dans le tableau f ( 6 ) = 3.
FAUX. Comme g ’(3) = f ’(3)ef (3) et f ’(3) < 0, g ’(3) < 0.
Corrigé Séquence 4 – MA01
103
© Cned - Académie en ligne
Corrigé des exercices de synthèse
de la séquence 4
Exercice I
La fonction f est dérivable sur . On commence par calculer sa dérivée :
f ′( x ) = e x − 1 dans le but d’étudier son signe. Comme f ′( x ) est la somme de e x
qui est toujours positif et de −1 qui est toujours négatif on ne peut conclure directement. On ne peut pas non plus factoriser f ′( x ). On résout donc les inéquations :
f ′( x ) ≥ 0 ⇔ e x − 1≥ 0
f ′( x ) ≤ 0 ⇔ e x − 1 ≤ 0
⇔ e x ≥ 1
⇔ e x ≤ 1
⇔ e x ≥ e0
⇔ x ≥ 0
⇔ e x ≤ e0
⇔ x ≤ 0
(Justification pour la dernière ligne : exp est croissante sur donc exp ne change
pas l‘ordre sur ).
On en déduit que f est strictement croissante sur ]0 ; + ∞ [ et strictement décroissante sur ] − ∞ ; 0[.
Exercice II
t
Pour tout t ≥ 0, q (t ) = Ce 8 est la quantité en cm3 de la substance à l’instant
−
t
en minutes.
Pour déterminer C, on se sert de la donnée de la quantité initiale présente
c’est-à-dire quant t = 0.
On a q (0 ) = C × e 0 = 5 d’après l’énoncé, soit : C = 5 car e 0 = 1.
t
t
−
8
On a donc q (t ) = Ce
= 5×e 8.
−
Remarque
Ici, la variable s’appelle t (et non x)
mais les calculs de dérivées s’effectuent de même. Ne pas se laisser
perturber par les notations.
q est dérivable sur [0 ; +∞ [ et q (t ) = 5e u (t )
t
1
1
avec u (t ) = − = − × t et u '(t ) = − .
8
8
8
t
t
−1 − 8
5 −8
u (t )
= 5× ×e = − e .
On a donc : q '(t ) = 5 × u '(t ) × e
8
8
On étudie le signe de q '(t ) sur [0 ; +∞ [ :
t
−
5
On a − < 0 et e 8 > 0 pour tout t ∈[0 ; +∞ [ donc pour tout t ≥ 0, q '(t ) < 0.
8
La fonction q est donc strictement décroissante sur [0 ; +∞ [.
Remarque
Ceci est cohérent puisque la substance est éliminée progressivement. Pour un tel
exercice, il faut s’efforcer de faire attention à la cohérence des résultats trouvés.
Cela peut permettre de détecter une erreur de dérivée par exemple (comme celle
u (t )
d’oublier u ’ en dérivant t e ).
104
© Cned - Académie en ligne
Corrigé Séquence 4 – MA01
(
La suite q (n )
q (n ) = 5e
−n
8
)n ∈ est définie pour tout entier naturel n par
n
 −1
= 5 e 8  .


On reconnaît la suite géométrique de premier terme 5 et de raison e
−1
8.
Comme la fonction exponentielle est strictement croissante sur l’intervalle ] − ∞ ; 0] , que − 1 et 0 appartiennent à cet intervalle, e
e
−1
8
8
−1
8
< e0 soit
< 1.
(q (n ))n ∈ est donc une suite de raison (bien sûr, strictement positive) strictement inférieure à 1.
Donc lim q (n ) = 0.
n →+∞
t
0
2
4
6
8
10
q (t )
5
3,894
3,033
2,362
1,840
1,433
y
y = q(t)
0,5
0
t
1
Corrigé Séquence 4 – MA01
105
© Cned - Académie en ligne
50 % de 5 cm3 est égal à 2, 5 cm3 .
On cherche à résoudre l’inéquation q (t ) ≤ 2, 5 graphiquement.
On peut s’aider du graphique précédent en traçant la droite d’équation
y = 2, 5 ou de la calculatrice (on peut utiliser un tableau de valeurs de
q (t ) de pas 0,1 par exemple). Au bout de 5,6 minutes, soit 5 minutes et 36
secondes, il reste moins de 50 % de la substance injectée dans le sang.
Remarque
Les 5 premières minutes, il y a plus de 50 % de la substance dans
le sang et après t = 6 minutes, il y a moins de 50 % de la substance
On pourra résoudre exactement l’inéquation q (t ) ≤ 2, 5
dans le sang.
plus tard avec la fonction ln.
Exercice III
2
La fonction f définie sur par f ( x ) = e − x est dérivable et pour tout réel x,
2
f ′( x ) = −2x e − x .
2
Le signe de f ′( x ) est celui de −2x puisque pour tout réel x, on a e − x > 0
(une exponentielle est toujours strictement positive).
x
f '( x )
−∞
+∞
0
+
0
–
1
f (x )
Voici les écrans obtenus avec :
106
© Cned - Académie en ligne
Casio
Texas Instruments
Réglages VWindow
Xmin = -10 ; Xmax = 10 ; Ymin = -0,5 ; Ymax= 1,2.
Réglages Window
Xmin = -10 ; Xmax = 10 ; Xgrad = 1 ;
Ymin = -0,5 ; Ymax= 1,2 ; Ygrad = 1.
Corrigé Séquence 4 – MA01
La courbe semble confondue avec l’axe des abscisses à partir d’une certaine
valeur mais ce n’est pas le cas. Il suffit pour le vérifier de changer la fenêtre
d’observation par exemple :
conduit à
puis
conduit à
Ainsi, dans une fenêtre adaptée (unités suffisamment petites sur l’axe (Oy)
par rapport à l’axe (Ox)) on observe que l’écart entre la courbe et l’axe (Ox)
n’est jamais nul (mais pour x = 3 il est déjà très faible et il devient vite
imperceptible à la calculatrice dès que x prend des valeurs plus grandes que
5). Ceci s’explique par un « écrasement très rapide » de la courbe de la fonction exponentielle sur l’axe des abscisses, accentué ici pour la fonction f par
le fait que x 2 grandit encore plus vite que x lui-même.
Exercice IV
(d’après bac ES – Métropole / La Réunion – septembre 2009)
On considère les fonctions f, g et h définies et dérivables pour tout nombre réel
x de l’intervalle [4 ; 6] par :
f ( x ) = 100(e x − 45), g ( x ) = 106 e− x et h ( x ) = g ( x ) − f ( x ).
a. La fonction h est dérivable sur l’intervalle [ 4 ; 6] et pour tout réel x ∈[ 4 ; 6],
h ′( x ) = g ′( x ) − f ′( x ) = 106 × ( −1) × e − x − 100e x = −100(e x + 104 e − x ).
Comme pour tout réel x, on sait que e x > 0 et e − x > 0 , à coup sûr,
e x + 104 e − x > 0.
D’où, pour tout réel x ∈[ 4 ; 6] , h ′( x ) < 0.
Conclusion
h est une fonction strictement décroissante sur [ 4 ; 6].
Corrigé Séquence 4 – MA01
107
© Cned - Académie en ligne
b. Le tableau de variations de la fonction h en découle :
x
4
6
h '( x )
–
h( x )
Conclusion
c. Pour justifier que l’équation
h ( x ) = 0 admet une solution
L’équation h ( x ) = 0 admet
unique α sur l’intervalle [ 4 ; 6],
une solution unique α sur l’innous allons montrer que h ( 4 ) > 0
tervalle [ 4 ; 6].
et h (6 ) < 0; ce qui permettra de
conclure, puisque la fonction h est
continue et strictement décroissante sur l’intervalle [ 4 ; 6].
On calcule h ( 4 ) = 106 e −4 − 100e 4 + 4500 ≈ 17 355, 8 puis
h (6 ) = 106 e −6 − 100e6 + 4500 ≈ −33 364 ,1.
a. Recopier puis compléter le tableau de valeurs suivant (les résultats seront
arrondis à la centaine la plus proche) :
x
4
4,2
4,4
4,6
4,8
5
h(x)
17400
12800
8600
4600
600
–3600
5,2
5,4
5,6
5,8
6
–8100 –13100 –18800 –25500 –33400
b. Sur la figure suivante, la courbe représentative
Ꮿ h de la fonction h :
28000
24000
20000
12000
8000
4000
α 4,82
0
3,6
3,8
4
–4000
4,2
4,4
4,6
–8000
–12000
–16000
–20000
–24000
–28000
–32000
108
© Cned - Académie en ligne
Corrigé Séquence 4 – MA01
4,8
5,0
5,2
5,4
5,6
5,8
6,0
6,2
6,4
c. Par lecture sur le graphique précédent on trouve 4 , 8 < α < 4 , 9.
Exercice V
Partie I
On a :
g '( x ) = e x − 1. Ainsi :
g '( x ) > 0 ⇔ e x − 1> 0 ⇔ e x > 1 ⇔ e x > e0 ⇔ x > 0 ;
g '( x ) < 0 ⇔ x < 0 et g '( x ) = 0 ⇔ x = 0.
On déduit de la précédente étude, le tableau de variations de g.
−∞
x
Signe de g ’
+∞
0
–
0
+
g
0
g (0 ) = e0 − 0 − 1 = 1− 1 = 0.
On déduit du tableau de variations précédent que le minimum de g sur
est 0 donc, pour tout x de , g ( x ) ≥ 0 soit e x − x − 1 ≥ 0 ou encore :
e x ≥ x + 1.
Partie II
La tangente
Ᏸ à Ꮿ au point d’abscisse 0 a pour équation :
Ᏸ : y = f '(0)( x − 0) + f (0)
soit Ᏸ : y = e0 ( x − 0 ) + e0 (la dérivée de la fonction exponentielle est la
fonction exponentielle).
Ainsi Ᏸ a pour équation : y = x + 1.
(déjà vu dans le cours sur la fonction exponentielle).
: e x ≥ x + 1 d’après la partie I. Cela signifie que Ꮿ est
toujours au-dessus de Ᏸ.
Pour tout x de
Corrigé Séquence 4 – MA01
109
© Cned - Académie en ligne
y
Ᏸ
Ꮿ
1
0
Exercice VI
(d’après bac ES – Amérique du Sud – novembre 2010)
y
1
x
1
On considère la fonction numérique f définie et dérivable sur
telle que, pour tout réel x, on ait :
f (x ) =
1
x
O
x2
− x 2e x −1.
2
On note f ′ sa fonction dérivée sur
.
Le graphique ci-contre est la courbe représentative de cette
fonction telle que l’affiche une calculatrice dans un repère
orthonormé.
On peut conjecturer que, sur l’intervalle [ −3 ; 2], la fonction f est décrois-
sante.
On calcule
(
)
(
)
2x
− 2x e x −1 + x 2e x −1 = x − x e x −1(2 + x ) = x 1− ( x + 2)e x −1 = xg ( x )
2
où g ( x ) = 1− ( x + 2)e x −1 pour tout x de .
f ′( x ) =
Étude du signe de g ( x ) suivant les valeurs de x.
a. Pour tout entier naturel n, on a g (n ) = −(n + 2)en −1 + 1 = −e −1(n + 2)en + 1.
110
© Cned - Académie en ligne
Corrigé Séquence 4 – MA01
1
Comme −e −1 = − < 0 et que lim (n + 2) = +∞ et lim en = +∞ , on a
e
n →+∞
n →+∞
lim −e −1(n + 2)en = −∞ ; par conséquent, lim g (n ) = −∞ .
n →+∞
n →+∞
b. On calcule g ′( x ) = −e x −1 − ( x + 2)e x −1 = −e x −1( x + 3).
Comme pour tout réel x, on sait que e x −1 > 0 , le signe de g ′( x ) est le
contraire de celui de x + 3.
c. Le tableau de variations suivant en découle :
x
g ′( x )
g(x )
−∞
+∞
–3
+
0
–
1+ e −4
2
d. Comme g (1) = −2, g (1) < 0. Comme g (0 ) = 1− ≈ 0, 26 ; g (0 ) > 0. Comme
e
g est une fonction continue et strictement décroissante sur l’intervalle [0 ; 1],
d’après la propriété des valeurs intermédiaires, l’équation g ( x ) = 0 possède
donc une unique solution dans 0 ; 1 . On note α cette solution.
La calculatrice indique g (0, 2) ≈ 0,115 et g (0, 21) ≈ −0, 003 ce qui justifie,
d’après le tableau de variations précédent que 0, 20 < α < 0, 21.
e. On a les équivalences x ∈] − ∞ ; − 2] ⇔ x ≤ −2 ⇔ x + 2 ≤ 0 ⇔ − ( x + 2) ≥ 0.
On sait aussi que pour tout réel x (a fortiori, pour tout x ∈] − ∞ ; − 2])
e x −1 > 0.
Donc pour tout x ∈] − ∞ ; − 2], −( x + 2)e x −1 ≥ 0.
Par conséquent pour tout x ∈] − ∞ ; − 2], 1− ( x + 2)e x −1 ≥ 1.
Autrement dit, pour tout x ∈] − ∞ ; − 2], g ( x ) ≥ 1.
f. Comme ] − ∞ ; − 3] ⊂ ] − ∞ ; − 2], d’après la question précédente, pour tout
x ∈] − ∞ ; − 3], g ( x ) ≥ 1.
g est donc de signe positif sur l’intervalle ] −∞ ; − 3].
Par ailleurs, la lecture du tableau de variations de g nous indique que
▶
▶
sur l’intervalle [ −3 ; α ] la fonction g décroit de 1+ e −4 à 0 donc pour
x ∈[ −3 ; α ], g ( x ) ≥ 0.
sur l’intervalle [α ; + ∞ [ la fonction g décroit de 0 en prenant des valeurs
négatives (ce qui est cohérent avec le résultat lim g (n ) = −∞ ) donc
n →+∞
pour x ∈[α ; + ∞ [, g ( x ) ≤ 0.
Corrigé Séquence 4 – MA01
111
© Cned - Académie en ligne
On peut résumer ces résultats dans le tableau suivant :
x
g(x )
α
−∞
+
+∞
−
0
Sens de variations de la fonction f
a. Puisque pour tout réel x, f ′( x ) = xg ( x ), du tableau de signes de la question
f. précédente découle le tableau de signes de f ′( x ) :
x
−∞
−
x
g(x )
+
−
f ′( x )
α
0
0
0
+
+∞
+
0
+
−
+
0
−
b. Puis, d’après un théorème du cours donnant les variations d’une fonction dérivable à partir du signe de sa dérivée, on en déduit le tableau de variations de
f suivant :
x
f '( x )
−∞
–
0
+∞
α
0
+
0
–
f (x )
0
c. La conjecture de la question est fausse puisque sur l’intervalle [0 ; α ]
(c’est-à-dire entre 0 et 0,21 environ) la fonction f est strictement croissante.
On peut expliquer ceci par le fait que l’intervalle [0 ; α ] est petit (moins de
0,21 de long) en regard de l’échelle du graphique de la question mais aussi
par le fait que f (α ) est petit (0, 002 environ). 112
© Cned - Académie en ligne
Corrigé Séquence 4 – MA01
C orrigé Séquence 5
Corrigé de l’activité du chapitre 2
Activité 1
Retour sur la fonction exponentielle
y
On sait que le tableau de variation de la
fonction exponentielle et sa courbe sont :
x
]1 ; +
exp ′( x )
f ′( x
0
+
1
exp(x )
1
O
Soit a un réel.
On s’intéresse au nombre de solutions de l’équation exp( x ) = a dans
à-dire au nombre d’antécédents de a par la fonction exp.
y
Ꮿexp
y = ex
1
x
c’est-
Deux cas de figure se présentent :
▶ Si a
≤ 0 alors :
la courbe représentant la fonction
exp et la droite d’équation y = a
n’ont pas de point d’intersection car
exp( x ) > 0 pour tout x de .
1
x
O
a
Dans ce cas, il n’y a aucun antécédent
de a donc aucune solution x à l’équation exp( x ) = a .
y=a
Corrigé Séquence 5 – MA01
113
© Cned - Académie en ligne
y
y = ex
▶ Si a
> 0 alors :
la courbe représentant la fonction exp
et la droite d’équation y = a ont un
unique point d’intersection.
Dans ce cas, l’équation exp( x ) = a
admet une et une seule solution α
dans .
y=a
a
1
O
114
© Cned - Académie en ligne
␣
Corrigé Séquence 5 – MA01
x
Corrigé des exercices
d’apprentissage du chapitre 2
Exercice 1
L’ingrédient essentiel à la résolution des calculs de cet exercice est la relation
fonctionnelle de la fonction ln :
ln(ab ) = lna + ln b , valable pour tous les réels a et b strictement positifs.
ln 4 = ln(2 × 2) = ln 2 + ln 2 = 2ln 2 (ici, a = b = 2).
ln 6 = ln(2 × 3) = ln 2 + ln 3 (ici, a = 2 et b = 3).
ln 24 = ln( 3 × 23 ) = ln 3 + ln(23 ) = ln 3 + 3ln 2.
( )
ln ( −4 )2 = ln( 42 ) = ln(24 ) = 4 ln 2.
ln 54 = ln(2 × 33 ) = ln 2 + ln( 33 ) = ln 2 + 3ln 3.
( )
ln 4 = ln 4 − ln 27 = ln(22 ) − ln( 33 ) = 2ln 2 − 3ln 3.
27
ln( 36 ) = ln(6 ) = ln(2 × 3) = ln 2 + ln 3.
ln( 9 ) = ln 9 − ln 8 = ln( 32 ) − ln(23 ) = 2ln 3 − 3ln 2.
8
Exercice 2
ln 63 − ln 7 = ln( 32 × 7) − ln 7 = ln( 32 ) + ln 7 − ln 7 = 2ln 3.
ln(27 3 ) = ln( 33 ) + ln( 3 ) = 3ln 3 + 1 ln 3 = 7 ln 3.
2
2
2
2ln 21− ln 49 = 2ln( 3 × 7) − ln( 7 ) = 2ln 3 + 2ln 7 − 2ln 7 = 2ln 3.
Exercice 3
Exercice 4
)
(
2
 2
A = ln( 11 − 10 ) + ln( 10 + 11) = ln ( 11 − 10 ) × ( 11 + 10 ) = ln  11 − 10  = ln(1) = 0.


 1
−3ln 5
B = ln   + ln 25 − 5ln 5 = − ln 5 + ln(52 ) − 5 × 1 ln 5 = − ln 5 + 2ln 5 − 5 ln 5 =
.
2
2
2
 5
 1
C = 5ln   − 4 ln 3 = 5 × ( − ln 3) − 4 × 1 ln 3 = −7ln 3.
2
 3
 4
 5
 3
D = ln   + ln   + ln   = ln 4 − ln 5 + ln 5 − ln 3 + ln 3 − ln 4 = 0.
 5
 3
 4
Exercice 5
ln 6 − ln 3
E =e
ln 6
=e
3
= eln 2 = 2.
Corrigé Séquence 5 – MA01
115
© Cned - Académie en ligne
F =e
G
− 1 ln 4
2
=e
− 1 ln( 22 )
2
=e
− 1 2ln( 2)
2
 28 
ln 
ln28−ln4
=e
= e  4  = eln7 = 7.
= e − ln( 2) =
1
ln( 2)
e
1
= .
2
2
3
2 3
H = e2ln 3+ 3 ln 2 = eln( 3 )+ ln( 2 ) = eln( 3 × 2 ) = 32 × 23 = 72.
 
I = ln  1  = − ln(e5 ) = −5.
 e5 
Exercice 6
Vrai / Faux
a. VRAI. On se souvient qu’une valeur approchée de e est 2,7. En tout cas, avec
certitude, on sait que 2 < e < 3.
Comme la fonction ln est strictement croissante sur  0 ; + ∞  , on en déduit
que ln 2 < ln e < ln 3, c’est-à-dire ln 2 < 1 < ln 3.
ln 2
, puisqu’en
ln 0, 5
divisant par ln 0, 5 qui est strictement négatif, l’inégalité change de sens.
b. FAUX. L’inéquation x × ln 0, 5 ≤ ln 2 est équivalente à x ≥
1
1
ln 2 2 ln 2 2 ln 2
1
Comme
=
=
= − , finalement l’inégalité est équivalente à
ln 0, 5 ln 1 − ln 2
2
2
1
x ≥− .
2
Conclusion
᏿ =  − 21 ; + ∞ 
c. FAUX. À l’aide de la relation fonctionnelle de la fonction ln, on calcule
x = ln 3 + ln 4 = ln( 3 × 4 ) = ln12, d’où ln x = ln(ln12). Reste à voir que ln(ln(12))
n’est pas égal à 12 ; si c’était le cas : ln(ln(12)) = 12 alors en appliquant la
fonction exp à chaque membre de cette égalité, on aurait eln(ln(12)) = e12 , soit
encore ln12 = e12 ce qui signifierait que les points A (12 ; ln12 et B (12 ; e12 )
seraient confondus. Ceci n’est pas vrai puisque le point A est sur la courbe de
la fonction ln, le point B est sur la courbe de la fonction exp et que ces courbes
n’ont pas de point commun (vérifiez-le sur votre calculatrice). Par conséquent,
ln(ln(12)) n’est pas égal à 12.
)
d. FAUX. D’après la relation fonctionnelle de la fonction exponentielle,
x = e5 × e7 = e5+ 7 = e12 , donc ln x = ln e12 = 12.
116
© Cned - Académie en ligne
Corrigé Séquence 5 – MA01
 5, 2 × 4 , 9 
 25, 48 
= ln 
.
e. VRAI. On calcule b − a = ln 5, 2 − ln11+ ln 4 , 9 = ln 

 11 
 111 
25, 48
Comme 25, 48 > 11,
> 1. Comme la fonction ln conserve l’ordre des
11
25, 48
inégalités (car elle est strictement croissante sur ]0 ; + ∞ [ ), ln
> ln1,
11
25, 48
autrement dit ln
> 0. On a donc montré que b − a > 0 soit a < b.
11
Conclusion
Si a = ln11− ln 4 , 9 et b = ln(5, 2) alors a < b.
Corrigé Séquence 5 – MA01
117
© Cned - Académie en ligne
Corrigé de l’activité du chapitre 3
Activité 2
La symétrie orthogonale par rapport à la première bissectrice
et Pour définir le point A de coordonnées (2 ; 3) dans le logiciel Geogebra, on
peut entrer dans le champ de saisie A = (2,3) ; autrement dit, on sépare
les coordonnées par une virgule (le séparateur décimal est le point).
B
5
4
A
3
C
D
2
E
1
0
–5
–4
–3
–2
–1
0
1
2
3
4
5
–1
F
–2
–3
G
:y=x
–4
–5
Pour construire le symétrique B ′ du point B , on trace la perpendiculaire à ∆
passant par B. Le point B ′ est sur cette perpendiculaire à la même distance que
B de la droite ∆ mais de l’autre côté de la droite ∆ (c’est-à-dire, dans l’autre
demi-plan bordé par ∆ ). On remarque que le point B ′, symétrique orthogonal
du point B par rapport à la 1re bissectrice, est en fait le point E. On construit de
même les autres symétriques :
118
© Cned - Académie en ligne
Corrigé Séquence 5 – MA01
B
5
E’
4
C’
3
A
D
2
D’
C
A’
E
1
B’
0
–5
–4
–3
–2
–1
F
1
0
2
3
4
5
–1
G’
–2
F’
G
–3
–4
:y=x
–5
En ajoutant au graphique précédent les points H’ et l’, on complète le tableau
suivant :
A
Point M
C
D
E
F
G
H
I
2
(2 ; 3) (1 ; 5) ( 4 ; 2) (2 ; 2) (5 ; 1) ( −3 ; − 1) ( −2 ; − 4 ) (3 ; –5) (π ; e )
Coordonnées de M
Point M‘ symétrique de M
Coordonnées de M‘
B
A′
B′
C′
D′
E′
F′
G′
H′
I′
2
(3 ; 2) (5 ; 1) (2 ; 4) (2 ; 2) (1 ; 5) (–1 ; –3) (–4 ; –2) ( −5 ; 3) (e ; π )
À l’aide du tableau précédent, compléter la conjecture suivante :
« Si un point M a pour coordonnées (a ; b ) alors le symétrique de M par rapport à la première bissectrice a pour coordonnées (b ; a )».
Corrigé Séquence 5 – MA01
119
© Cned - Académie en ligne
Corrigé des exercices
d’apprentissage du chapitre 3
Exercice 7
a. Le point A est sur la courbe de la fonction ln donc ses coordonnées vérifient
l’équation de cette courbe ; autrement dit, en notant ( x A ; y A ) les coordonnées de A, on doit avoir y A = ln x A .
Comme x A = e −2 , on calcule y A = ln(e −2 ) = −2ln e = −2.
(
)
La phrase complétée est donc A e −2 ; − 2 ∈Ꮿ ln .
b. Notons a le nombre par lequel il faut remplacer les pointillés dans
B (..... ; y ∈Ꮿ ln . Pour la même raison qu’à la question précédente, a doit
vérifier lna = y , c’est-à-dire, a = e y (y est une donnée de l’énoncé, même
si on ne sait pas combien vaut ce nombre). La phrase complétée est donc
B e y ; y ∈Ꮿ ln .
)
( )
c. L’opération consistant à échanger les coordonnées d’un point est la symétrie
par rapport à la première bissectrice du repère. Comme les points des courbes
des fonctions exp et ln s’échangent par cette symétrie, la phrase complétée
est C ( x ; y ) ∈Ꮿ exp ⇔ D ( y ; x ) ∈Ꮿ ln .
d. D’après une propriété du cours à savoir, ln x = 5 ⇔ x = e5 .
e. Remarquons d’abord que ln e x = x .
x
Puis e x × e − ln e = e x × e − x = e x − x = e0 = 1.
x
Il fallait donc écrire e x × e − ln e = 1.
Exercice 8
a. L’image de 0,5 par la fonction ln est ln 0, 5.
On a aussi ln 0, 5 = ln 1 = ln(2−1) = − ln 2.
2
b. Notons b l’antécédent de −0, 5 par la fonction ln.
On écrit que ln b = −0, 5 est équivalent à b = e −0,5 .
c. Si on note a le nombre ayant pour image 2,7 par la fonction exp alors
ea = 2, 7 c’est-à-dire a = ln 2, 7.
d. L’équation ln x = 0 est équivalente à x = e0 soit encore x = 1.
Donc l’ensemble des solutions est ᏿ = 1 .
{}
e. L’équation ln x = 1 est équivalente à x = e1 soit encore x = e.
Donc l’ensemble des solutions est ᏿ = e .
{}
120
© Cned - Académie en ligne
Corrigé Séquence 5 – MA01
Corrigé des activités du chapitre 4
Activité 3
Le sens de variation de la fonction réciproque
On considère les fonctions f et g définies pour tout réel positif x par
f ( x ) = x 2 et g ( x ) = x .
a) Pour montrer que les fonctions f et g sont réciproques nous allons utiliser
la définition suivante :
Définition
On dit que les fonctions f et g sont réciproques lorsque
partant d’un nombre a, en appliquant d’abord la fonction f puis la fonction g, on retrouve le nombre a.
Conclusion
Pour commencer, on se donne un nombre
réel positif a quelconque. En lui appliquant la
fonction f on obtient le nombre réel a 2.
En appliquant ensuite la fonction g au résultat précédent, on obtient a 2 .
Comme on a choisi a tel que a ≥ 0, on sait que a 2 = a. Par conséquent,
nous avons démontré que partant d’un nombre réel positif a et en lui
appliquant successivement f puis g on retrouve le nombre a de départ.
Les fonctions f et g
sont réciproques.
b) Appliquer la fonction f revient à appliquer la fonction « carré » aux réels positifs. Comme la fonction « carré » est strictement croissante sur [0 ; + ∞ [, la
fonction f est strictement croissante.
La fonction g (définie sur [0 ; + ∞ [) est la fonction « racine carrée », qu’on sait
strictement croissante.
Nous allons voir à la question suivante, que ce cas particulier se généralise :
une fonction et sa fonction réciproque ont la même monotonie (c’est-à-dire
que si l’une est strictement croissante, l’autre aussi ; si l’une est strictement
décroissante, l’autre aussi).
25
20
Cas général : On considère deux
fonctions u et v réciproques,
15
10
a. On dessine une courbe Ꮿ u d’une
fonction u (ici u est strictement croissante).
5
O
5
10
15
20
25
–5
Corrigé Séquence 5 – MA01
121
© Cned - Académie en ligne
25
puis la courbe Ꮿ v , symétrique de Ꮿ u
par rapport à la première bissectrice du
repère.
20
15
10
On constate que ces deux courbes
« montent ».
On conjecture donc que si u est strictement croissante, la fonction v, réciproque
de u, elle aussi, est strictement croissante.
5
O
5
10
15
20
25
–5
Pour le démontrer, on revient à la définition d’une fonction strictement croissante :
On dit que la fonction f est strictement croissante lorsqu’elle conserve l’ordre,
autrement dit, lorsque, pour deux nombres c et d quelconques, si c < d alors
f (c ) < f (d ).
On suppose donc que la fonction u est strictement croissante.
Ceci signifie que si c et d sont deux réels quelconques tels que c < d alors
u (c ) < u (d ).
On veut démontrer que la fonction v aussi est strictement croissante.
On va démontrer la contraposée, autrement dit :
Si a et b sont deux réels quelconques tels que v (a ) ≥ v (b ) alors a ≥ b.
On considère deux réels quelconques a et b tels que v (a ) ≥ v (b ).
On doit prouver que a ≥ b.
Pour cela, on a besoin de la définition de v : c’est la fonction réciproque de la
fonction u, autrement dit, pour tout réel x, lorsqu’on applique d’abord la fonction
u à x puis la fonction v au résultat, le résultat final estv le nombre x de départ.
On pose c = v (b ) (ce qu’on peut schématiser par b c ).
v
On pose aussi d = v (a ) (ce qu’on peut schématiser par a d ), de sorte que
c ≤ d.
u
u
On peut schématiser par c u (c ) et d u (d ).
v
u
Les deux schémas s’écrivent b c u (c ) . On applique donc successivement la
fonction v au nombre b puis la fonction u au nombre v (b)=c.
Comme v et u sont des fonctions réciproques, on retrouve le nombre b ; autrement dit, u (c ) = b.
v
u
De même, le schéma a d u (d ) signifie qu’on applique successivement deux
fonctions réciproques, donc u (d ) = a.
122
© Cned - Académie en ligne
Corrigé Séquence 5 – MA01
▶
Si c = d , alors u (c ) = u (d ).
Comme u (c ) = b et u (d ) = a , a = b. Donc, à coup sûr, a ≥ b (une égalité
est un cas (particulier) d’une inégalité large).
▶
Sinon, c < d .
Par hypothèse (u est strictement croissante), u (c ) < u (d ).
Comme u (c ) = b et u (d ) = a , l’inégalité s’écrit b < a. Là encore, à coup sûr,
a ≥ b (une inégalité stricte implique une inégalité large).
Dans tous les cas, on a démontré que a ≥ b.
Donc, si a et b sont deux réels quelconques tels que v (a ) ≥ v (b ) alors a ≥ b.
Conclusion : la fonction v est strictement croissante (dès que la fonction u
(réciproque de v) l’est).
b. On conjecture donc que si u est strictement décroissante, la fonction v, réciproque de u, aussi est strictement décroissante.
La démonstration de ce résultat est tout à fait analogue à celle faite dans la
question précédente en changeant le sens des inégalités et échangeant les
mots croissante et décroissante.
Activité 4
Quelle dérivée pour la fonction ln ?
Conformément au programme, on admet que la fonction ln est dérivable sur
 0 ; + ∞  .
On considère la fonction f définie sur  0 ; + ∞  par f ( x ) = eln( x ) .
Comme les fonctions exp et ln sont réciproques, pour tout réel x, on calcule
eln x = x , donc f ( x ) = x .
Comme pour tout réel x, f ( x ) = x , la dérivée de f vérifie pour tout réel x,
f ′( x ) = 1.
La dérivée de x eu ( x ) s’écrit, pour tout x réel, u ′( x )eu ( x ) . Par conséquent,
comme f ( x ) = eu ( x ) avec u ( x ) = ln x on en déduit f ( x ) = ln ( x )eln x soit
′
′
encore, f ′( x ) = x ln′( x ).
Aux questions et , nous avons vu que f ′( x ) = 1 et f ′( x ) = x ln′( x ).
1
Donc 1= ln′( x ) × x soit ln′( x ) = .
x
Corrigé Séquence 5 – MA01
123
© Cned - Académie en ligne
Corrigé des exercices
d’apprentissage du chapitre 4
Exercice 9
Exercice 10
Une équation de la tangente à la courbe de la fonction ln au point d’abscisse 1
est y − ln1 = ln′ 1( x − 1) soit y = 1( x − 1) ou encore y = x − 1.
1
En considérant que la courbe est proche de sa tangente en x = 1 lorsqu’on s’intéresse à des points de la courbe dont l’abscisse est proche de 1, on peut approcher l’ordonnée ln x d’un point de la courbe par l’ordonnée du point de même
abscisse (ici x) mais situé sur la tangente. On obtient ainsi que ln x est proche de
x − 1 lorsque x reste proche de 1.
Donc une valeur approchée de ln x au voisinage de 1 est 1,1− 1 = 0,1. En fait,
l’erreur faite en remplaçant ln1,1 par 0,1 est 0,1− ln1,1 soit, avec 15 décimales,
0, 004689820195675.
Résolution d’équations avec ln et exp
2
a. D’abord, il faut x > – . Des équivalences suivantes :
5
ln(5x + 2) = ln 3 ⇔ eln( 5x + 2) = eln( 3) ⇔ 5x + 2 = 3 ⇔ x = 1 ,
5
découle
᏿=
{ }.
1
5
1
b. D’abord, il faut x < . Des équivalences suivantes :
2
ln( −2x + 1) = 0 ⇔ eln( −2x +1) = e0 ⇔ − 2x + 1 = 1 ⇔ x = 0,
découle
᏿ = {0}.
1
c. D’abord, il faut x > – . Des équivalences suivantes :
2
ln( 3x + 1, 5) = ln 5 − ln 2 ⇔ ln( 3x + 1, 5) = ln 5
2
⇔ 3x + 1, 5 = 5
2
⇔ x = 1,
3
{}
découle ᏿ = 1 .
3
d. Il faut d’abord x > 5. Des équivalences suivantes :
)
ln( x − 5) + ln( x − 3) = ln 3 + ln 5 ⇔ ln(( x − 5)( x − 3) = ln15
ln(( x − 5)( x − 3)
)
= eln15
⇔ e
⇔ ( x − 5)( x − 3) = 15
124
© Cned - Académie en ligne
Corrigé Séquence 5 – MA01
puis ( x − 5)( x − 3) = 15 ⇔ x 2 − 8 x = 0 ⇔ x ( x − 8 ) = 0,
découle x = 0 ou x = 8. Comme x > 5,
᏿ = { 8}.
e. En appliquant la fonction ln à chacun des membres de l’égalité e x = 7, on
obtient ln e x = ln 7, soit x = ln 7.
Donc
᏿ = {ln 7}.
f. Comme la fonction exp ne prend que des valeurs strictement positives (donc
elle ne prend pas la valeur −2) , l’équation e x = −2 n’a pas de solution
réelle ; autrement dit, ᏿ = ∅.
Exercice 11
Résolution d’inéquations avec ln et exp
a. Il faut d’abord x > 0. La fonction ln étant (strictement) croissante, ln x > ln 3
équivaut à x > 3. Donc ᏿ =  3 ; + ∞  .
b. Il faut d’abord x > –1. La fonction ln étant strictement croissante,
ln( x + 1) ≥ ln 3 ⇔ x + 1 ≥ 3 ⇔ x ≥ 2. Donc ᏿ = 2 ; + ∞  .
c. Il faut d’abord x > 0. La fonction ln étant strictement croissante,
ln x ≤ 1 ⇔ ln x ≤ ln e ⇔ x ≤ e. Donc ᏿ =  0 ; + e  .
d. Comme
1
−x
= ex ,
e 2x
−x
< 6 ⇔ e2x + x < 6 ⇔ e 3x < 6 ; puis, comme la
e
e
fonction exp est (strictement) croissante
e 3x < 6 ⇔ 3x < ln 6 ⇔ x < ln 6 . Donc ᏿ =  −∞ ; ln6  .
3 

3
e. Il faut d’abord x ∈  −1 ; 1 . ln(1− x 2 ) ≥ 0 ⇔ ln(1− x 2 ) ≥ ln1 ⇔ 1− x 2 ≥ 1,
puisque la fonction ln est strictement croissante.
Enfin, 1− x 2 ≥ 1 ⇔ x 2 ≤ 0 ⇔ x = 0 (un carré est toujours positif ou nul).
Donc ᏿ = {0 }.
Exercice 12
Application de la fonction ln aux suites géométriques
n
a. Comme lim 2 = +∞ , il existe (au moins) un entier n tel que 2n ≥ 315.
n →+∞
n
Remarquons d’abord qu’on peut écrire 2n = eln 2 = en ln 2.
15
De même, 315 = eln 3 = e15 ln 3 .
Corrigé Séquence 5 – MA01
125
© Cned - Académie en ligne
Donc dire qu’un entier n vérifie 2n ≥ 315 est équivalent à
15ln 3
(car ln 2 > 0 ).
ln 2
15ln 3
Comme la calculatrice nous indique que 23 <
< 24 , le plus petit entier
ln 2
n
15
n pour lequel 2 ≥ 3 est n = 24.
en ln 2 ≥ e15 ln 3 ⇔ n ln 2 ≥ 15ln 3 ⇔ n ≥
b. Comme lim 0, 9n = 0, il existe (au moins) un entier n tel que 0, 9n ≤ 0, 001.
n →+∞
n
Remarquons d’abord qu’on peut écrire 0, 9n = eln 0,9 = en ln 0,9 .
−3
De même, 0, 001 = 10−3 = eln(10 ) = e −3 ln10 .
Donc dire qu’un entier n vérifie 0, 9n ≤ 0, 001 est équivalent à
−3ln10
en ln 0,9 ≤ e −3 ln10 ⇔ n ln 0, 9 ≤ −3ln10 ⇔ n ≥
.
ln 0, 9
(Attention au changement de sens de la dernière inégalité dû au fait qu’on a
divisé par ln 0, 9 < 0 ).
−3ln10
Comme la calculatrice nous indique que 65 <
< 66, le plus petit
ln 0, 9
n
entier n pour lequel 0,9 ≤ 0,001 est n = 66.
Exercice 13
a. Si x ∈] − 4 ; + ∞ [, alors x + 4 > 0. Donc la fonction f est bien définie et dérivable sur ] − 4 ; + ∞ [.
En posant u ( x ) = x + 4 on peut écrire f ( x ) = ln(u ( x )) dont la dérivée est
u ′( x )
1
f ′( x ) =
. Donc f ′( x ) =
.
u(x )
x +4
b. Si u ∈] − ∞ ; 0[, alors −3u > 0. Donc la fonction g est bien définie et dérivable
sur ] − ∞ ; 0[.
En posant h (u ) = −3u on peut écrire g (u ) = ln(h (u )) dont la dérivée est
h ′(u )
−3 1
g ′(u ) =
. Donc g ′(u ) =
= .
h (u )
−3u u
c. Si t ∈] − ∞ ; 0[, alors 7t 2 + 3 > 3 > 0. Donc la fonction h est bien définie et
dérivable sur ] − ∞ ; 0[.
En posant u (t ) = 7t 2 + 3 on peut écrire h (t ) = ln(u (t )) dont la dérivée est
u ′(t )
7 × 2t
14t
h ′(t ) =
. Donc h ′(t ) =
.
=
u (t )
7t 2 + 3 7t 2 + 3
Exercice 14
Soit f la fonction définie sur [ −4 ; 6] par f ( x ) = e x − 2x − 1.
a. La fonction f est dérivable sur [ −4 ; 6] et on calcule f ′( x ) = e x − 2.
126
© Cned - Académie en ligne
Corrigé Séquence 5 – MA01
On résout d’abord l’équation f ′( x ) = 0, équivalente à
e x − 2 = 0 ⇔ e x = 2 ⇔ x = ln 2.
La dérivée de f ne s’annule donc que pour une seule valeur réelle égale à ln 2.
On calcule aussi f (ln 2) = eln 2 − 2ln 2 − 1 = 2 − 2ln 2 − 1 = 1− 2ln 2.
Pour étudier le signe de f ′( x ) on résout ensuite l’inéquation f ′( x ) > 0.
f ′( x ) > 0 ⇔ e x − 2 > 0 ⇔ e x > 2 ⇔ x > ln 2, puisque la fonction exp est
strictement croissante sur [ −4 ; 6].
b. Des résultats précédents découlent le tableau de signes de f’ et le tableau de
variations de f :
x
–4
–
f '( x )
6
ln2
0
+
f (x )
1 – 2 ln2
Exercice 15
e
D’abord, pour qu’un réel x soit solution de l’équation ln x = il est nécessaire
x
d’avoir x > 0.
e
L’équation ln x = fait intervenir deux fois la variable x et on ne peut pas « l’isox
ler » à l’aide d’une factorisation. Ainsi, comme souvent dans une telle situation,
pour résoudre cette équation, on introduit une fonction auxiliaire ; ici la fonction
e
e
f définie par f ( x ) = ln x − , de sorte que, pour prouver que l’équation ln x =
x
x
admet une unique solution il faut et il suffit de prouver qu’il existe un unique réel
x, solution de l’équation f ( x ) = 0. On va donc étudier les variations de f et pour
ce faire, on calcule f ′( x ).
1  e  x +e
f ′( x ) = −  −  =
.
2
x  x2
x
Le signe de f ′( x ) est celui de x + e. Comme x > 0, f ′( x ) > 0.
Donc f est une fonction strictement croissante sur  0 ; + ∞  .
x
f '( x )
f ′( x
0
+
f (x )
Corrigé Séquence 5 – MA01
127
© Cned - Académie en ligne
Il nous faut montrer maintenant que la fonction f prend (au moins) une valeur
négative et (au moins) une valeur positive.
e
e
On calcule f (1) = ln1− = −e < 0 puis f ( 3) = ln 3 − .
1
3
Pour voir que f ( 3) > 0, il suffit d’observer que ln3 > 1 et e < 3.
On peut donc compléter le tableau de la manière suivante :
x
0
f '( x )
a
1
+
+
3
+
f ′( x
+
f (3)
0
f (x )
f (1)
Comme f est une fonction continue sur  0 ; + ∞  , strictement croissante sur
l’intervalle [1 ; 3], que f (1) < 0 et f ( 3) > 0, d’après la propriété des valeurs intere
médiaires, l’équation ln x = admet une unique solution. On la note a. Le tracé
x
sur la calculatrice (ou avec Geogebra) semble indiquer que a soit proche de 2,7.
On pense alors à e ≈ 2,7.
e
Le calcul de f (e) = ln(e) − = 1− 1 = 0 nous donne la réponse : étant donné que le
e
Conclusion
nombre a est l’unique réel x tel
que f ( x ) = 0 et que par ailleurs,
f (e ) = 0 ; nécessairement, a = e.
Exercice 16
L’ensemble des solutions de l’équation
e
ln x = est ᏿ = {e }.
x


La fonction f est définie sur [1; +∞[ par f ( x ) = ln  x − x 2 − 1.


2
Posons u ( x ) = x − x − 1, de sorte que f ( x ) = ln(u ( x )). (On admet que
u ( x ) > 0, dès que x > 1). La fonction f est dérivable sur ]1; +∞[ et sa dérivée
u '( x )
. Pour résoudre u '( x ) ≥ 0, on a besoin de calculer la dérivée
est f '( x ) =
u(x )
de u. Pour cela, on écrit : u ( x ) = x − x − 1 x + 1.
 1

1
D’où u '( x ) = 1− 
x + 1+
x − 1 (dérivée d’un produit)
 2 x −1

2 x +1
i.e. . u '( x ) = 1−
128
© Cned - Académie en ligne
Corrigé Séquence 5 – MA01
x + 1− ( x − 1) 2 x 2 − 1 − 2x
x 2 − 1− x
=
=
.
2 x 2 −1
2 x 2 −1
x 2 −1
2


u ′( x ) ≥ 0 ⇔ x 2 − 1 − x ≥ 0 ⇔ x 2 − 1 ≥ x ⇔  x 2 − 1 ≥ x 2


(car x ∈]1 ; + ∞ [ donc x > 0 ).
Donc u ′( x ) ≥ 0 ⇔ x 2 − 1 ≥ x 2 ⇔ − 1 ≥ 0. Comme cette dernière inégalité est toujours fausse, l’inégalité équivalente
u ′( x ) ≥ 0 aussi. Autrement dit, pour tout réel x de ]1 ; + ∞ [,
u ′( x ) < 0 ; par conséquent, f ′( x ) < 0, puisque u ( x ) > 0.
Conclusion
f est une fonction strictement
décroissante sur ]1 ; + ∞ [.
Corrigé des exercices de synthèse
de la séquence 5
Exercice I
On considère la fonction f définie sur l’intervalle [1 ; 3] par f ( x ) =
x ln x
−
.
2 x
Partie I
On considère la fonction u définie sur l’intervalle [1 ; 3] par u ( x ) = x 2 − 2 + 2ln x .
On calcule u ′( x ) = 2x +
 x 2 + 1
2
= 2
.
x
 x 
Tout réel x de l’intervalle [1 ; 3] vérifie x > 0 donc, u ′( x ) > 0 si x ∈[1 ; 3]. La
fonction u est donc strictement croissante sur [1 ; 3].
x
1
u ′( x )
3
+
7 + 2ln 3
u( x )
–1
Comme la fonction u est continue et strictement croissante sur l’intervalle
[1 ; 3] , que f (1) < 0 et f ( 3) > 0, la propriété des valeurs intermédiaires permet d’affirmer que l’équation u ( x ) = 0 admet une unique solution a dans
l’intervalle [1 ; 3]. Un tracé avec le logiciel Geogebra de la courbe de u (la
molette de la souris permet de zoomer) indique que 1, 2 < a < 1, 3. Par ailleurs,
à 10 −3 près, on calcule u (1,2) ≈ −0,195 u (1,3) ≈ 0,215 ce qui prouve que
a ∈[1, 2 ; 1, 3].
Comme
la fonction u est strictement croissante sur [1 ; a ],
que u (1) < 0 et u (a ) = 0, la fonction u reste strictement négative sur [1 ; a[.
De même, comme la fonction u est strictement croissante sur [a ; 3],
que u(a ) = 0 et u ( 3) > 0, la fonction u reste strictement positive sur ]a ; 3].
Corrigé Séquence 5 – MA01
129
© Cned - Académie en ligne
On résume ceci dans le tableau de signes suivant :
x
1
–
u( x )
3
a
0
+
Partie II
x ln x
−
. A l’aide des dérivées des fonc2 x
tions usuelles et de la formule donnant la dérivée d’un quotient on calcule :
 1 × x − ln x × 1
1
1 1 ln x x 2 − 2 + 2ln x
= − −
f ′( x ) = −  x
=
.
2
2
2
2 
 2
2
x
x
x


On remarque que le numérateur de l’expression précédente est égal à u ( x ),
u( x )
d’où : f ′( x ) =
.
2x 2
Comme 2x 2 est toujours strictement positif lorsque x ∈[1 ; 3], le signe de
f ′( x ) est le même que celui de u ( x ). Le tableau de signes de f ′( x ) est donc
le même que celui de la question I. .
a. On va calculer la dérivée de f ( x ) =
b. Le tableau de variations de f en découle :
x
–
f '( x )
f (x )
a
1
3
0
+
1
2
3 ln 3
−
2 3
f (a )
a lna
−
. Le nombre a est caractérisé (car c’est le seul)
2 a
lna 1 a
= − .
par u (a ) = 0, autrement dit, par a 2 − 2 + 2lna = 0 ; soit encore,
a a 2
lna
Ainsi, pour le calcul de f (a ), on peut remplacer l’expression
par l’expresa
1 a
a  1 a
1 a2 – 1
sion (qui lui est égale) − . On obtient f (a ) = −  −  = a − =
.
a 2
2  a 2
a
a
Numériquement, on trouve (en utilisant le zoom de Geogebra) a ≈ 1, 2478 et
c. Par définition, f (a ) =
f (a ) ≈ 0,44648.
( )
On note A le point de coordonnées 1 ; 1 .
2
1−0
1
2
a. Le coefficient directeur de la droite (OA ) est égal à
= . Le coefficient
directeur de la tangente ∆ à
130
© Cned - Académie en ligne
Corrigé Séquence 5 – MA01
Ꮿ
1− 0 2
au point d’abscisse e est égal à f ′(e ). On
calcule f ′(e ) =
2ln e − 2 + e2
2
=
2 − 2 + e2
2
1
= ; ce qui prouve qu’ils sont égaux
2
2e
2e
et donc que les deux droites sont parallèles.
b.
3
2,5
2
1,5
1
A
0,5
0
–1,5 –1 –0,5
0
–0,5
e
0,5
1
1,5
2
2,5
3
3,5
4
4,5
5
5,5
6
6,5
7
–1
–1,5
)
c. Sur le graphique suivant, on a placé le point B de coordonnées (a ; f (a ) et
tracé la tangente à la courbe Ꮿ au point B. Cette dernière est parallèle à l’axe
des abscisses puisque son coefficient directeur est égal à f ′(a ) = 0.
3
2,5
2
1,5
1
B
0,5
0
–1,5 –1 –0,5
0
–0,5
0,5
1 a 1,5
2
2,5
3
3,5
4
4,5
5
5,5
6
6,5
7
–1
–1,5
Corrigé Séquence 5 – MA01
131
© Cned - Académie en ligne
Exercice II
(d’après bac)
Dans l’intervalle ]0 ; + ∞ [ l’équation f ( x ) = e2 admet deux solutions puisque
e2 > 2ln 2 + 3.
La tangente à la courbe
Ꮿ au point d’abscisse ln(1,5) admet un coeffi-
cient directeur strictement négatif puisque ln(1, 5) appartient à l’intervalle
]0 ; ln 2[, sur lequel la fonction f est strictement décroissante et sa dérivée
strictement négative.
 1
f ( − ln( 2)) est égal à ln   ou encore à −2ln 2.
 4
Exercice III
On considère la suite (un ) définie par son 1er terme u 0 = 6 et par la relation de
récurrence :
1
Pour tout entier naturel n, un +1 = un + 3.
4
On pose v = u − 4.
n
n
a. On fixe arbitrairement un entier naturel n. On a un = v n + 4 et aussi,
un +1 = v n +1 + 4. En utilisant ces deux égalités, la relation de récurrence sur
1
1
la suite (un ) devient v n +1 + 4 = v n + 4 + 3, soit encore v n +1 = v n .
4
4
1
n étant arbitraire, ceci prouve que la suite (v n ) est géométrique de raison .
4
Son 1er terme vaut v 0 = u 0 − 4 = 2.
(
)
1
b. Le terme v n de la suite géométrique (v n ) de 1er terme v 0 = 2 et de raison
4
n
n
 1
 1
est donné par la relation v n = v 0   c’est-à-dire, v n = 2   .
 4
 4
n
 1
D’où un = 2   + 4.
 4
c. Comme (v n ) est une suite géométrique de raison positive et strictement inférieure à 1, elle tend vers zéro. La suite (un ) converge donc vers 4.
On pose a = lnv .
n
n
a. On fixe n dans
.
v
1
On calcule la différence an +1 − an = lnv n +1 − lnv n = ln n +1 = ln = −2ln 2,
vn
4
ce qui montre que (an ) est une suite arithmétique et de raison −2ln 2.
132
© Cned - Académie en ligne
Corrigé Séquence 5 – MA01
b. On calcule aussi le 1er terme
terme général est donné par :
a0 = ln 2. On sait que pour une telle suite le
an = ln 2 + ( −2ln 2) × n.
c. La valeur de n pour laquelle an est égal à
−13ln 2 = ( −2ln 2) × n + ln 2 c’est-à-dire, n = 7.
Exercice IV
−13ln 2
vérifie
(d’après bac)
Partie A
a. La recette pour 200 litres (c’està-dire, pour 2 centaines de
litres) de médicament vendus est
R (2) = 1, 5 × 2 = 3 milliers d’euros,
autrement dit, 3000 € .
Coût total
(en milliers d’euros)
y
9
b. Ci-contre, le graphique avec le
segment représentant la fonction
R.
8
Γ1
7
Lectures graphiques
6
a. Les bornes de la « plage de rentabilité » sont les abscisses des
points d’intersection de la courbe
et du segment. On lit sur le graphique : 0,5 et 4,5. Comme entre
0,5 et 4,5 les recettes sont supérieures aux coûts de production,
la plage de rentabilité est l’intervalle [0, 5 ; 4 , 5]; c’est-à-dire entre
50 litres et 450 litres produits.
5
R
4
3
2
1
A
쐌
O
1
2
␣ 3
b. On lit sur le graphique les ordonnées des points d’abscisse 2 :
R (2) = 3 et CT (2) = 1, 25. Par
conséquent, le bénéfice réalisé par
le laboratoire lorsque 200 litres de
médicament sont commercialisés
est approximativement égal à
B (2) = R (2) − CT (2) = 3 − 1, 25 = 1, 75
6 x
4
5
millier d’euros, soit 1750 euros.
Volume du médicament produit
(en centaines de litres)
Corrigé Séquence 5 – MA01
133
© Cned - Académie en ligne
c. Le bénéfice semble maximal pour 2,5 (ou un peu plus) centaines de litres de
médicament. Pour obtenir cette valeur graphiquement, on peut chercher à
« descendre le plus possible » le segment représentant R en le gardant parallèle au segment R initial et de sorte que ce segment coupe toujours la courbe
Γ 1 (ce qui revient aussi à tracer la tangente à la courbe Γ 1 qui est parallèle
au segment représentant R puis à lire l’abscisse du point de tangence) ; ainsi
on obtient un point de la courbe Γ 1 dont la distance (comptée verticalement)
au segment représentant R est la plus grande possible ; cette distance est
justement le bénéfice. Il vaut alors environ 4 , 25 − 2 = 2, 25 milliers d’euros,
soit 2250 € environ.
Partie B
Dans la suite de l’exercice, on admet que la fonction coût total CT est définie sur
2
l’intervalle [0, 25 ; 5] par CT ( x ) = x − 2x ln( x ).
Le bénéfice est la différence des recettes et des coûts de fabrication ; donc,
pour x centaines de litres commercialisés, il vaut B ( x ) = 1, 5x − x 2 + 2x ln( x )
en milliers d’euros.
On calcule B (2) = 4 ln 2 − 1 ≈ 1, 773. On retrouve (presque) la valeur 1,750
obtenue à la question b. de la partie A.


On calcule B ′( x ) = 1, 5 − 2x + 2 1× ln( x ) + x ×

1
= 3, 5 − 2x + 2ln( x ).
x 
On donne ci-dessous le tableau de variation de la fonction B ′ , dérivée de la
fonction B, sur l’intervalle [0, 25 ; 5].
x
0,25
1
5
1,5
B ’(x)
y1
y2
On précise les encadrements : 0, 22 < y 1 < 0, 23 et −3, 29 < y 2 < −3, 28.
a. Comme y 1 > 0 et que la fonction B ′ croît pour x ∈[0, 25 ; 1], l’équation
B ′( x ) = 0 n’admet pas de solution dans l’intervalle [0, 25 ; 1]. En revanche,
comme B ′(1) > 0, que B ′(5) < 0 et que la fonction est continue et strictement décroissante sur l’intervalle [1 ; 5], elle vérifie la propriété des valeurs
intermédiaires : il existe un réel α de [1 ; 5] tel que B ′(α ) = 0. Comme en
plus, la fonction B ′ est strictement décroissante sur l’intervalle [1 ; 5], ce réel
est unique.
134
© Cned - Académie en ligne
Corrigé Séquence 5 – MA01
Remarquons d’abord que B ’(2,76) = 0,010... et B ’(2,77) = – 0,002...
b.
x
α ≈ 2, 77
0,25
B ′( x )
–
0
5
+
2,13
B(x )
– 0,38
– 1,41
Dans ce tableau, les nombres sont des valeurs approchées.
a. Le bénéfice est maximal lorsque x = α (avec α ≈ 2, 77) c’est-à-dire pour une
production de 277 litres de médicament.
Le bénéfice maximal correspondant est alors environ de 2130 €.
b. A la question c. de la partie A, nous avions obtenu par lecture graphique
un bénéfice maximal de 2250 € pour un coût de production d’un peu plus de
250 litres de médicament. Corrigé Séquence 5 – MA01
135
© Cned - Académie en ligne
C orrigé Séquence 6
Corrigé des activités du chapitre 2
Activité 1
La distance parcourue est 3 × 10 m = 30 m.
Pour obtenir une valeur approchée de la distance parcourue dans ce deuxième
cas, on va considérer que la vitesse est constante entre deux valeurs. Par exemple
entre t = 0 s et t = 0, 5 s, on considère que la vitesse reste égale à 9 m.s –1,
une valeur approchée de la distance parcourue entre ces deux instants est alors
9 × 0, 5 m = 4 , 5 m. Ainsi la distance d parcourue pendant les 10 s peut être
estimée :
d ≈ (9 × 0,5 + 7,6 × 0,5 + 6,1× 0,5 + 4,6 × 0,5 + 3,7 × 0,5
+... + 0,4 × 1+ 0,2 × 1+ 0,2 × 1+ 0,1× 1) m
d ≈ 21,65 m.
왘
Le produit de la question peut être interprété comme le calcul de l’aire
d’un rectangle ayant un côté de mesure 10 (la durée) et un côté de mesure 3
(la vitesse). La distance parcourue est alors la mesure de l’aire du rectangle.
La mesure de l’aire est faite avec l’unité donnée par le repère (qui n’est pas
nécessairement orthonormé), cette unité est coloriée sur le graphique et elle
–1
représente 1 m de distance parcourue puisque 1m.s × 1s = 1m.
vitesse (en m.s–1)
1
0
0,5 1
2
3
4
unité d’aire = distance parcourue : 1m
9
10
temps
(en secondes)
La somme de la question peut être interprétée comme la somme d’aires de
rectangles dont les côtés parallèles à l’axe des abscisses sont les intervalles de
temps et dont les côtés parallèles à l’axe des ordonnées sont les vitesses.
왘
Corrigé Séquence 6 – MA01
137
© Cned - Académie en ligne
vitesse
(en m.s–1)
9
7,6
6,1
4,6
1
O
0,5 1
2
3
4
9
10
temps
(en secondes)
On évalue de façon analogue l’aire sous la courbe qui a été enregistrée.
v(t)
en m.s–1
1
O
1
5
t
10
en secondes
−1
L’aire d’un carreau correspond toujours au produit 1m.s × 1s = 1m.
La distance parcourue D peut être évaluée à au moins 32 m en comptant les
carreaux entiers qui sont coloriés. On peut estimer à environ 6 carreaux l’aire sous
la courbe qui n’a pas été prise en compte dans les carreaux entiers. Finalement, la
distance D peut être estimée à 38 m.
Activité 2
138
© Cned - Académie en ligne
n Les ordonnées des points O, B1, B2, B3 et B4 sont obtenues en utilisant la
fonction carré.
Corrigé Séquence 6 – MA01
1  2  1 2  2  2  3  2  4  2 
Ꮽ 1 =  0 +   +   +   +    u.a.
 5  5  5  5  
5
1 30
Ꮽ 1 = × u.a.
5 25
Ꮽ 1 = 0,24 u.a.
De même :

1   1 2  2  2  3  2  4  2
2
Ꮽ 2 =    +   +   +   + (1)  u.a.
5   5  5  5  5 

1 55
Ꮽ 2 = × u.a.
5 25
Ꮽ 2 = 0,44 u.a.
Les figures permettent de conjecturer un encadrement de Ꮽ ( Ᏹ) :
Ꮽ 1 ≤ Ꮽ ( Ᏹ) ≤ Ꮽ 2 , soit 0,24 u.a. ≤ Ꮽ ( Ᏹ) ≤ 0,44 u.a..
Remarque
On peut prendre la moyenne des termes extrêmes de l’encadrement pour
donner une valeur approchée : Ꮽ ( Ᏹ) ≈ 0,34 u.a.
Les figures ont été faites avec du papier millimétré et le carré OA 5B5C
qui donne l’unité d’aire vaut 25 cm2 donc Ꮽ ( Ᏹ) ≈ 0,34 × 25 cm2 , soit
Ꮽ ( Ᏹ) ≈ 8,5 cm2.
Dans cette séquence, étant données une fonction f à valeurs positives et sa
représentation graphique sur un intervalle a ; b  , on s’intéresse à « l’aire sous
b
la courbe ». La mesure de cette aire, en unités d’aire, est notée ∫ f ( x ) dx .
a
On admettra que pour la fonction carré,
b 2
∫a x
dx =
3
b −a
3
3
pour a ≤ b.
)
On obtient ainsi la valeur exacte de Ꮽ ( Ᏹ :
13 − 03
1
 1

Ꮽ ( Ᏹ =  ∫ x 2 d x  u.a. =
u.a. = u.a.
 0

3
3
)
Corrigé Séquence 6 – MA01
139
© Cned - Académie en ligne
Corrigé des exercices
d’apprentissage du chapitre 2
Exercice 1
12
13 03 1
t dt = − = . On admet que la courbe de la fonction racine est
0
3 3 3
∫
symétrique de la courbe de la fonction carré par rapport à la droite d’équation
y = x . Ainsi, dans le carré OIAJ, la mesure de l’aire au-dessus de la courbe de
1
la fonction racine est égale , l’intégrale précédente. L’aire du carré mesure 1,
3
on obtient la mesure de l’aire sous la courbe de la fonction racine par différence,
1
1 2
donc ∫ t dt = 1− = .
0
3 3
Dans le carré OIAJ, comme l’aire sous la courbe de la fonction carré est égale
à l’aire au–dessus de la courbe de la fonction racine, l’aire entre les deux courbes
est égale à l’aire du carré moins deux fois l’aire sous la courbe de la fonction
1
1 1
carré. Cette aire mesure donc : 1− 2∫ t 2 dt = 1− 2 × = .
0
3 3
y=x
Remarque
Les deux courbes partagent donc
le carré OIAJ en trois domaines
de même aire.
J
A
y= x
y = x2
O
Exercice 2
4 2
1
1  4 3 − ( −4 )3  64 16
µ1 =
t dt = 
= .
=
4 − ( −4 ) ∫−4
8
3
 12 3
µ2 =
Exercice 3
I
a 2
1
1  a 3 − ( −a )3  a 2
t
d
t
=

= .
a − ( −a ) ∫ − a
2a 
3
 3
n Pendant la première heure, l’accélération, c’est-à-dire la dérivée de la vitesse,
est constante, donc la vitesse est représentée par un segment de droite de
140
© Cned - Académie en ligne
Corrigé Séquence 6 – MA01
)
l’origine jusqu’au point de coordonnées (1; 80 . La vitesse est ensuite constante
pendant deux heures. Pendant la dernière demi-heure, l’accélération (négative)
est constante, la vitesse est représentée par un segment de droite jusqu’au point
)
de coordonnées ( 3, 5 ; 0 .
vitesse (en km.h-1)
80
20
O
1
temps (en heures)
Comme on l’a vu dans l’activité 1, la distance parcourue D peut être évaluée
en déterminant « l’aire sous la courbe ».
1
1
D = × 1× 80 + 2 × 80 + × 0, 5 × 80 = 220 km.
2
2
Le trajet a duré 3h 30min, on divise donc par 3,5 pour obtenir la vitesse moyenne :
220
vm =
≈ 62,86 km.h–1.
3,5
Exercice 4
D’après le théorème 1, F '( x ) =
1
et G '( x ) =
1
. On remarque que les
x +1
fonctions F et G ont la même fonction dérivée sur 2 ; 100  .
2
x +1
2
Or on sait que, pour tout x de 2 ; 100  , on a
x
1
2
1
x
1
∫1 t 2 + 1 dt = ∫1 t 2 + 1 dt + ∫2 t 2 + 1 dt
F (x ) = ∫
1
2
1 2
t +1
L’intégrale
2
d’après la relation de Chasles, et donc
dt + G ( x ).
1
∫1 t 2 + 1 dt
est une constante, donc en dérivant on retrouve bien
F '( x ) = G '( x ).
Corrigé Séquence 6 – MA01
141
© Cned - Académie en ligne
Exercice 5
Réponse c) : c’est la relation de Chasles.
Réponse b) : l’aire du rectangle construit
sur  −4 ; 0  et ayant pour hauteur la
valeur moyenne doit avoir une aire égale
à l’aire sous la courbe de la fonction f sur
 −4 ; 0  . L’aire hachurée qui dépasse du
1
rectangle doit être compensée par l’aire
-4
0
coloriée. Cela n’est pas possible pour 2 et
1
3,5 donc la bonne réponse est 3.
On peut estimer, à l’aide des carreaux, que l’intégrale I =
appartient à l’intervalle b) [ 9 ;11].
0
∫−3f ( x ) dx
La proposition « si deux fonctions f et g continues et positives sur a ; b  sont
b
b
f ( x ) d x = g ( x ) d x , alors f ( x ) = g ( x ) pour tout x de a ; b  »
telles que
a
a
∫
∫
est fausse. En effet on peut seulement conclure que l’aire sous la courbe de f
est égale à l’aire sous la courbe de g sur l’intervalle a ; b  . En particulier, on
obtient une telle égalité si on prend pour fonction g la fonction constante égale
à la valeur moyenne de f (voir la figure de la définition 2).
142
© Cned - Académie en ligne
Corrigé Séquence 6 – MA01
Corrigés des activités
du chapitre 3
Activité 3
Pour tout réel x, on obtient F '( x ) = G '( x ) = H '( x ) = 3x 2. Les fonctions dérivées
sont égales, mais les fonctions ne sont pas égales puisqu’elles sont chacune la somme
de la fonction cube et d’une constante, les trois constantes utilisées étant différentes.
2
Pour tout x de ]1; + ∞[ , on a F '( x ) = G '( x ) = H '( x ) =
.
( x − 1)2
Là encore les trois fonctions F, G et H ont les mêmes fonctions dérivées. Si on
x −3
. Les fonctions F et G sont
transforme G ( x ) on obtient G ( x ) = F ( x ) =
x −1
égales, mais la fonction H est différente (par exemple H (0 ) = 5 et G (0 ) = 3).
Activité 4
On considère les deux fonctions f et F définies sur ]0 ; + ∞[ par f ( x ) = ln x et
F ( x ) = x ln x − x .
En appliquant la règle de dérivation d’un produit, on obtient, pour tout x de
]0 ; + ∞[ ,

1
F '( x ) =  1× ln x + x ×  − 1 = ln x + 1− 1 = f ( x ).
x

Pour trouver une fonction G différente de F, telle que G ' = F ', il
suffit d’ajouter une constante à l’expression F ( x ) : posons par exemple
G ( x ) = F ( x ) + 2 = x ln x − x + 2. Et, pour trouver une fonction H différente
de F et de G, il suffit d’ajouter une constante différente, posons par exemple
H ( x ) = F ( x ) − 3,01 = x ln x − x − 3,01.
On peut ainsi créer une infinité de fonctions ayant pour dérivée la fonction f, il
suffit d’ajouter une constante à la fonction F. Ici on a une condition supplémentaire :
on cherche donc une constante C telle que K ( x ) = F ( x ) + C = x ln x − x + C avec
K (1) = 1× ln1− 1+ C = 0. On en déduit que C = 1 et donc la fonction K est définie
par K ( x ) = x ln x − x + 1.
Activité 5
On reconnaît la somme des dérivées de x x 6 , x x 2 et x x . Une
6
2
possibilité est donc F ( x ) = x + x + x .
Remarque : on peut aussi ajouter en plus une constante.
Ici c’est un peu moins simple car on ne reconnaît pas les dérivées de fonctions
puissances, un facteur multiplicatif est nécessaire.
Corrigé Séquence 6 – MA01
143
© Cned - Académie en ligne
1 6
3
Ainsi x x 5 est la fonction dérivée de x x ; x x est la fonction
6
1 4
dérivée de x x ; x −3 est la fonction dérivée de x −3x .
4
fonction qui convient est la fonction F définie par
1
1
F ( x ) = x 6 + x 4 − 3x + 1000 (on peut choisir d’ajouter n’importe quelle
6
4
Une
constante).
−1
1
]
[
Pour f ( x ) =
+ sur 0 ; + ∞ , on reconnaît la somme des dérivées de
x2 x
la fonction inverse et de la fonction logarithme, on peut proposer par exemple
1
F ( x ) = + ln x + 52,3.
x
L’expression de la dérivée de la fonction F définie sur
par F ( x ) = (ax + b )e x
est F '( x ) = ae x + (ax + b )e x = (ax + (a + b ))e x (on a dérivé un produit). Pour
obtenir l’expression de f ( x ), c’est-à-dire ( 3x + 1)e x , il suffit que les nombres a
a = 3
a = 3
, soit 
. Donc la fonction F définie sur
et b vérifient le système 
a + b = 1
b = −2
par F ( x ) = ( 3x − 2)e x a pour dérivée f.
144
© Cned - Académie en ligne
Corrigé Séquence 6 – MA01
Corrigé des exercices
d’apprentissage du chapitre 3
Exercice 6
F (x ) = x 5 −
F (x ) =
Exercice 7
1
3 4
x − x 3 + 2x 2 + 2x + 12, 3
4
F (x ) = −
x2
1
+4 x
x
F (x ) = −
F (x ) =
5
+ 333
x
1 2
x + 2x + 3ln x .
2
Dans chaque cas, sur l’intervalle I, on détermine la primitive F de la fonction f
telle que F ( x 0 ) = y 0 .
F ( x ) = x 2 + 3x + k et F ( 2) = 22 + 3 × 2 + k = 0, d’où k = −10 et
F ( x ) = x 2 + 3x − 10.
F ( x ) = e x + k et F ( 0 ) = e0 + k = −4 , d’où k = −5 et F ( x ) = e x − 5.
F ( x ) = ln x + k et F (1) = ln1+ k = 2, d’où k = 2 et F ( x ) = ln x + 2.
Exercice 8
u
Dans cet exercice, il faut savoir que les primitives de la fonction f = u 'e sont
toutes les fonctions F = eu + k , où k est une constante.
F ( x ) = e x + e 2x − e − x + k
x
F ( x ) = −2e 2 + k
−
(
F (x ) =
4 3x
e +k
3
(x 2 )
F (x ) = e
+k
1
2
F ( x ) = e2x − 3e x + 4 x + k
)
Comme f ( x ) = e2x + 5e x − 1 e − x = e x + 5 − e − x , on obtient
F ( x ) = e x + 5x + e − x + k .
Exercice 9
On raisonne comme à la question de l’activité 5.
L’expression de la dérivée de la fonction F définie sur par F ( x ) = (ax + b )e x
est F '( x ) = ae x + (ax + b )e x = (ax + (a + b ))e x . Pour obtenir l’expression de
Corrigé Séquence 6 – MA01
145
© Cned - Académie en ligne
f ( x ), c’est-à-dire (2x + 3)e x , il suffit que les nombres a et b vérifient le système
a = 2
a = 2
, soit 
. Donc la fonction F définie sur par F ( x ) = (2x + 1)e x

3
1
a
+
b
=
b
=


est une primitive de f.
Exercice 10
On sait que : x = ( x − 1) + 1.
2001
D’où f ( x ) = [( x − 1) + 1] ( x − 1)
; f ( x ) = ( x − 1)2002 + ( x − 1)2001.
n
On connaît une primitive de x ( x − 1) qui est x Ainsi F ( x ) =
Exercice 11
1
( x − 1)n +1.
n +1
1
1
( x − 1)2003 +
( x − 1)2002.
2003
2002
fonction f1 est à valeurs strictement positives, donc F1 est strictement
croissante sur . Seule la courbe d peut convenir.
▶ La
fonction f2 est à valeurs négatives, puis positives, puis négatives, donc
F2 est décroissante, puis croissante, puis décroissante. Seule la courbe c peut
▶ La
convenir.
▶ Les fonctions
f3 et f4 sont à valeurs négatives sur ]−∞ ; 0[ et à valeurs positives
sur [ 0 ; + ∞[ . Donc F3 et F4 sont décroissantes sur ]−∞ ; 0[ et croissantes sur
[0 ; + ∞[ . Les courbes a et b correspondent à ces variations.
Quand on observe les tangentes à la courbe b, on peut voir que les coefficients
directeurs diminuent, puis augmentent, puis diminuent. Comme les coefficients
directeurs sont donnés par les nombres dérivés, la fonction dérivée de la fonction
représentée par la courbe b est décroissante, puis croissante, puis décroissante,
ce qui correspond à la fonction f3 .
Donc F3 est représentée par la courbe b et F4 est représentée par la courbe a.
146
© Cned - Académie en ligne
Corrigé Séquence 6 – MA01
Corrigé de l’activité du chapitre 4
Activité 6
On rappelle la propriété 10 : soit f une fonction continue et positive sur a ; b 
b
et F une de ses primitives, on a alors : ∫ f (t ) dt = F (b ) − F (a ).
a
1
Une primitive sur  2 ; 5  de la fonction « carré » est la fonction t t 3
3
5
1
1
117
donc I = ∫ t 2 dt = 53 − 23 =
. Une primitive de la fonction inverse sur
2
3
3
3
51
dt = ln5 − ln2.
2 ; 5  est la fonction logarithme népérien ln, donc J = ∫
2t
1
Une primitive sur  2 ; 5  de la fonction t t 2 + est la fonction
t
1 3
t t + lnt .
3
5
1
 1
 117
1
+ ln 5 − ln 2.
Donc K = ∫  t 2 +  dt =  53 + ln 5 −  23 + ln 2 =
2
3
t

3
 3
On observe immédiatement que K = I + J .
Une primitive sur  2 ; 5  de la fonction t 6t 2 est la fonction t 2t 3 .
5
Donc L = ∫ 6t 2 dt = 2 × 53 − 2 × 23 = 234. On obtient 6I = 6 ×
2
117
= 234 = L.
3
On généralise ici ce qui a été observé dans la question .
On a
b
b
b
∫a f (t ) dt = F (b ) − F (a ) et ∫a g (t ) dt = G (b ) − G (a ), donc
b
∫a f (t ) dt + ∫a g (t ) dt = F (b ) − F (a ) + G (b ) − G (a ).
Comme F est une primitive de f sur a ; b  et G une primitive de g, on en déduit,
d’après les règles de la dérivation, que (F + G )' = F '+ G ' = f + g . Ainsi la fonction
F + G est une primitive de f + g et on a
∫a (f + g )(t ) dt = (F (b ) + G (b )) − (F (a ) + G (a )) = F (b ) − F (a ) + G (b ) − G (a ).
b
)
On a donc bien : ∫ (f + g (t ) dt = ∫ f (t ) dt + ∫ g (t ) dt .
b
a
b
b
a
a
On généralise ici ce qui a été observé dans la question .
D’après les règles de la dérivation (αF )' = αF ' = αf , donc la fonction αF est
une primitive de la fonction αf et on a :
b
b
∫a (αf ) (t ) dt = (αF )(b ) − (αF )(a ) = α(F (b ) − F (a )) = α ∫a f (t ) dt .
Corrigé Séquence 6 – MA01
147
© Cned - Académie en ligne
Corrigés des exercices
d’apprentissage du chapitre 4
Exercice 12
2
2 
24 − ( −3)4 −65
A = ∫ 2x d x =  x 4  =
=
.
−3
2
2
 4  −3
2
B=
3
2
2 q
e dq = eq  = e2 − e0 = e2 − 1.
0
 0
∫
2
2 1
 1  1  1
1
1
C =
dt =  −  =  −  −  −  = − + 1 = .
1 2
2
2
 t 1  2  1
t
∫
D=
=
E=
e
2
 1
1

1 2
 e
=
+
=
+
x
+
d
x
x
ln
x
lne

 −  + ln1


∫1  x 
 2

2

1  2
e
e2 − 1
e2 + 1
+ 1− 0 =
.
2
2
∫
1
 e −4   e 4  e 4 − e −4
1 −4 x

 1
e
dx =  − e −4 x  =  −
.
 −−  =
−1
4
 −1  4   4 
 4
On utilise ici le fait que u 'u a pour primitive
1 2
u .
2
e
e lnt
e1
1
1
1
1
2
F=
dt =
× lnt dt =  (lnt )  = (ln e )2 − (ln1)2 = .
1 t
1 t
2
2
1 2
2
∫
∫
Remarque
Dans ces calculs d’intégrales, il est prudent de faire
des contrôles (calculer des valeurs approchées avec la
calculatrice, vérifier que l’intégrale sur a ; b  d’une
fonction positive sur a ; b  est positive…).
148
© Cned - Académie en ligne
Corrigé Séquence 6 – MA01
3
Exercice 13
1 3 −2x
1  1 −2x 
1 −2× 3  1 −2× 0  1− e −6
=
−
−− e
e
d
x
e
e
=
−

 = 6 .
3 − 0 ∫0
3  2
6
 6
0
Exercice 14
Sur 0 ; + ∞ F '( x ) = ln x + x ×
]
1
+ 3x 2 − 1 = ln x + 3x 2 = f ( x ), donc F est une
x
[
primitive de f sur ]0 ; + ∞[ .
)
(
e
)(
(
)
e
A = ∫ ln x + 3x 2 d x =  x ln x + x 3 − x  = e ln e + e 3 − e − 1ln1+ 13 − 1 = e 3 .
1
1

Exercice 15
Montrons que, pour tout réel x, les points de Ꮿ d’abscisses x et − x sont
)
symétriques par rapport à l’axe des ordonnées. Ces deux points, M( x ; f ( x ) et
)
M'( − x ; f ( − x ) , ont la même ordonnée car x
n’apparaît qu’au carré dans l’expression de
M'
f(-x)=f(x)
M
f ( x ). Les deux points M et M’ sont donc bien
symétriques par rapport à l’axe des ordonnées
et la courbe Ꮿ est donc symétrique par rapport
J
à l’axe des ordonnées.
Dans cet exercice, la symétrie permettra
d’utiliser l’égalité :
-x
I
O
x
b
0
∫−b f ( x ) dx = ∫0 f ( x ) dx .
0
a
Et on rappelle que ∫ f ( x ) d x = − ∫ f ( x ) d x .
0
a
Calculs approchés d’intégrales
Pour A
0
1
−1
0
La symétrie donne A = ∫ f ( x ) d x = ∫ f ( x ) d x = I(1).
D’où A ≈ 0,34134.
Pour B
2
2
−2
0
B = ∫ f ( x ) d x = 2∫ f ( x ) d x .
B = 2 × l(2)
B ≈ 0,954 5.
Corrigé Séquence 6 – MA01
149
© Cned - Académie en ligne
Pour C
C= ∫
3,5
1,5
3,5
=∫
0
0
3,5
1,5
1,5
0
f ( x ) dx = ∫ f ( x ) dx + ∫
f ( x ) dx
f ( x ) dx − ∫ f ( x ) dx
0
= l(3,5) – l(1,5)
C ≈ 0,499 77 – 0,433 20
C ≈ 0,066 57.
Pour D
1
0
1
−3
3
−3
1
0
0
0
D = ∫ f ( x ) dx = ∫ f ( x ) dx + ∫ f ( x ) dx
D = ∫ f ( x ) d x + ∫ f ( x ) d x car f est paire.
D = l(3)+l(1)
D ≈ 0,839 99.
Pour E
−1
4
0
4
–4
4
1
1
1
0
0
0
E = ∫ f ( x ) dx = ∫ f ( x ) dx = ∫ f ( x ) dx + ∫ f ( x ) dx
E = ∫ f ( x ) dx − ∫ f ( x ) dx
E = I(4) − I(1)
E ≈ 0,158 63.
Ainsi
A = 0,341 34
B = 0,954 5
C = 0,066 57
D = 0,839 99
E = 0,158 63
Pour G, on obtient
4
G = 2∫ f ( x ) d x = 2 × I(4).
0
G ≈ 0,99994.
On remarque que G est vraiment très proche de 1 et que G semble aussi très
proche de l’aire totale sous la courbe considérée entièrement, x variant dans tout entier.
C’est pour obtenir un tel résultat que le coefficient
1
est introduit dans la
2π
fonction f (ce coefficient peut paraître un peu surprenant mais sa justification
n’est pas accessible en terminale).
La fonction f et les intégrales analogues à celles que vous venez de calculer
sont extrêmement importantes en probabilité et en statistiques car elles nous
serviront à calculer des probabilités où l’univers est infini.
150
© Cned - Académie en ligne
Corrigé Séquence 6 – MA01
Exercice 16
La fonction dérivée de f est définie sur
par :
f '( x ) = e − x − ( x + 3)e − x = ( − x − 2) e − x .
Comme une exponentielle est toujours positive, f '( x ) est du signe de − x − 2,
donc la fonction f est croissante sur ]−∞ ; − 2] et décroissante sur [−2 ; +∞[ .
La fonction F est une primitive de f sur si, pour tout réel x, on a :
F '( x ) = f ( x ), soit ae − x − (ax + b )e − x = ( x + 3)e − x ,
c’est-à-dire −axe − x + (a − b )e − x = ( x + 3)e − x . Il suffit donc que les réels a et b
 −a = 1
, ainsi les réels a = −1 et b = −4 conviennent.
vérifient le système 
a − b = 3
Pour tout x supérieur à –3, f ( x ) est positif car x + 3 et e − x sont positifs. Donc
la mesure, en unités d’aire, de l’aire A1 du domaine du plan limité par la courbe
Ꮿ, l’axe des abscisses et les droites d’équation x = −3 et x = 4, est donnée par
4
∫−3
f ( x ) d x = F ( x )
4
= ( − x − 4 )e − x
−3 
)(
(
4
)
 = −8e −4 − −1× e 3 = e 3 − 8e −4 .
 −3
Donc A1 = e 3 − 8e−4 u.a. ≈ 19,94 u.a.
B
C
e2
On
trouve
f ( −2) = e2.
Les
variations de la fonction f montrent
que f admet un maximum en −2,
donc f ( x ) ≤ e2 pour tout réel x.
L’aire A 2 est donc égale à l’aire
du rectangle ABCD moins l’aire
précédente.
D’où
J
A
Exercice 17
(
)
A 2 = 7e2 − A1 = 7e2 − e 3 − 8e −4 u.a.
≈ 31,78 u.a.
I
O
]
D
[
Sur 0 ; + ∞ , on a f '( x ) = 1+ 2 qui est positif, donc la fonction f est crois-
sante sur ]0 ; + ∞[ .
x2
Corrigé Séquence 6 – MA01
151
© Cned - Académie en ligne
2
, donc cette différence est négative sur ]0 ; + ∞[ et la
x
courbe Ꮿ est située sous la droite Ᏸ.
On a f ( x ) − ( x + 1) = −
y = f(x)
y = x+1
J
O
I
e
En unités d’aire, l’aire A(Ᏹ) du domaine Ᏹ est mesurée par l’intégrale
e
e 2 
∫1 ((x +1) – f (x )) dx = ∫1  x  dx = [2ln x ]1e = 2lne − 2ln1= 2.
Comme l’unité de longueur dans le repère est égale à 2 cm, on a :
A(Ᏹ) = 2u.a. = 2 × 4cm2 = 8cm2.
Exercice 18
Le graphique sur papier millimétré nous permet d’évaluer approximativement
l’aire du domaine compris entre le segment [OB] et la courbe de Lorenz.
On peut estimer cette aire à 19cm2 environ.
2
Comme l’aire du triangle OAB est égale à 50cm , on obtient :
19
IG ≈ , c’est-à-dire IG ≈ 0, 38.
50
(
)
La droite (OB) a pour équation y = x car on a B 100 ; 100 .
La courbe Ꮿ a pour équation y = 0, 009 x 2 + 0, 033x .
152
© Cned - Académie en ligne
Corrigé Séquence 6 – MA01
En unités d’aire, l’aire de concentration est mesurée par l’intégrale K, avec
K=∫
100
0
100
K=∫
0
( x − (0,009x
2
+ 0,033x
)) d x
(−0,009x 2 + 0,967x ) dx
100
0,967 2 

K =  −0,003x 3 +
x 
2

0
K = 1835.
En unités d’aire, le triangle OAB mesure
L’indice de Gini est alors égal à IG =
100 × 100
= 5000.
2
1835
≈ 0,37, ce qui est une valeur proche de
5000
la valeur trouvée graphiquement.
Dans ce pays, l’indice de Gini vaut donc environ 0,37, la répartition des revenus
des ménages est assez concentrée.
D’après le tableau, on peut dire que les 20 % des ménages les moins
favorisés détiennent seulement 6,01 % du revenu national. En calculant
100 − 56,15 = 43, 85, on voit par contre que les 20 % des ménages les plus riches
détiennent 43,85 % du revenu national.
De même les 10 % des ménages les plus riches détiennent 27,72 % du revenu
national, soit un peu plus du quart.
Exercice 19
Le nombre moyen m de personnes malades est égal à la valeur moyenne de la
fonction f sur l’intervalle 0 ; 10  .
m=
1 10
1  3 1 4 10
(
)d
10t − t 
f
t
t
=
10 ∫0
10 
4 0
m=
1
10000 
 10000 −
 = 750.
10
4 
Il y a donc en moyenne 750 personnes malades par jour durant les dix premiers
jours.
Sur l’intervalle  0 ; 30  , on a :
f '(t ) = 60t − 3t 2 = 3t (20 − t ).
Corrigé Séquence 6 – MA01
153
© Cned - Académie en ligne
t
+
f '(t )
30
20
0
0
−
4000
f (t )
0
0
Donc t 0 = 20 et le nombre maximum de malades est f (20 ) = 4000.
On calcule le nombre moyen M de malades sur l’intervalle 16 ; 24  :
24
1
1  3 1 4 24 1
M=
f (t )dt = 10t − t  = (55296 − 24576) = 3840.
24 − 16 ∫16
8
4 16 8
154
© Cned - Académie en ligne
Corrigé Séquence 6 – MA01
Corrigés des exercices
de synthèse de la séquence 6
Exercice I
(Exercice proposé au baccalauréat) Nouvelle Calédonie mars 2007.
Sur  0 ; 1 la fonction f est dérivable et f '( x ) = 8( − x + 1) qui prend des valeurs
positives donc f est croissante sur 0 ; 1 .
1
1− x
qui prend des
Sur 1; 5  la fonction f est dérivable et f '( x ) = − 1 =
x
x
valeurs négatives donc f est décroissante sur 1; 5  .
a) La fonction F définie sur  0 ; 1 par F ( x ) =
−4 3
x + 4 x 2 est une primitive
3
de f sur 0 ; 1 .
1
−4
8
1  −4 3
2
+4= .
b) On a : f ( x ) d x = F ( x ) =  x + 4 x  =
0
0
3
0 3
 3
∫
1
Donc l’aire du domaine plan limité par la courbe ( Ꮿ), l’axe des abscisses, et la
droite d’équation x = 1, est égale à
8
u.a.
3
1
− 1 = ln x = g ( x ). Donc la fonction
x
G est une primitive de la fonction g sur 1; 5  .
a) Sur 1; 5  , on a G '( x ) = 1× ln x + x ×
déduit que la fonction F définie sur 1; 5  par
x2
x2
F ( x ) = G ( x ) − + 5x = x ln x − + 4 x est une primitive de f sur 1; 5  . On
2
2
remarque que, comme la fonction f, la fonction F a des expressions différentes
b)
On
en
sur les intervalles 0 ; 1 et 1; 5  .
On a :

5
5
x2
5
∫1 f ( x ) dx = [F ( x )]1 = x ln x − 2 + 4 x  = (5ln5 + 7,5) − (3,5) = 5ln5 + 4.

1
Donc l’aire du domaine plan limité par la courbe ( Ꮿ), l’axe des abscisses et les
droites d’équation x = 1 et x = 5, est égale à (5 ln5 + 4) u.a.
L’énoncé précise que les appels arrivent de façon continue. On constate que
la fonction f est continue sur [0 ; 5] puisque la valeur des deux expressions
Corrigé Séquence 6 – MA01
155
© Cned - Académie en ligne
étant la même pour x = 1, on peut tracer la courbe de f sans lever le crayon. Par
5
conséquent, on peut considérer l’intégrale ∫ f ( x )dx .
0
D’après la relation de Chasles, on a :
1
5
8
f
(
x
)
d
x
=
f
(
x
)
d
x
+
∫0
∫0
∫1 f ( x ) dx = 3 + 5ln5 + 4 ≈ 14,714.
Le nombre total d’appels reçus pendant 5 minutes est donné, en milliers, par
8
+ 5ln5 + 4, ce nombre est donc à peu près égal à 14700.
3
5
f ( 0 ) = 3 (c’est l’ordonnée du point A) et f '( 0 ) = 2 (c’est le coefficient directeur
Exercice II
de la droite (AB)).
La tangente T a donc pour
T
B
équation y = 2x + 3.
L’aire Ꮽ, qui est coloriée, est
comprise entre 3 et 4 carreaux, soit
3u.a. ≤ Ꮽ ≤ 4 u.a.
A
ex
on a
x=1
J
O
ax + b
a) Comme f ( x ) = 1+
Cf
f '( x ) = 0 +
f '( x ) =
I
b) Ainsi f (0 ) = 1+ b et f '(0 ) =
,
a × e x − (ax + b ) × e x
(ex )
2
−ax + a − b
ex
.
a−b
= a − b. Il suffit donc de trouver deux
1
1+ b = 3
, les nombres a = 4 et b = 2 conviennent.
nombres a et b tels que 
a − b = 2
On a donc bien f ( x ) = 1+
4x + 2
pour tout réel x.
ex
−4 x − 6
On admet que la fonction définie sur par F ( x ) = x +
est une
x
e
primitive de f sur .
La mesure, en unités d’aire, de la partie du plan limitée par la courbe Ꮿf , l’axe
156
© Cned - Académie en ligne
Corrigé Séquence 6 – MA01
des abscisses, l’axe des ordonnées et la droite d’équation x = 1, est égale à :
1
10
−4 x − 6   −10   −6 
1 
=  1+
−   = 7 − ≈ 3, 32.
f ( x ) d x = F ( x ) =  x +


0
0
e
e   1
e x 0 

∫
1
 10 
Donc Ꮽ =  7 −  u.a. ≈ 3,32u.a ce qui est cohérent avec le résultat de la

e
question : 3u.a. ≤ Ꮽ ≤ 4 u.a.
Exercice III
D’après le graphique le coût unitaire est minimal lorsque l’artisan fabrique 36
bijoux par mois. Cela correspond à l’abscisse du point le plus bas de la parabole P.
On sait que f ( x ) = x 2 + bx + c .
Comme f (0 ) = 1 400 alors c = 1 400.
On peut aussi écrire : f '( 36 ) = 0.
Or f '( x ) = 2x + b d’où 2( 36 ) + b = 0.
Ainsi b = −72 et f ( x ) = x 2 − 72x + 1 400.
On pouvait aussi utiliser d’autres renseignements :
f ( 36 ) = 104 et f (80 ) = 2 040.
Pour calculer le coût moyen unitaire on va calculer des valeurs moyennes
d’intégrales.
40
1 40
1 1 3

2
Calculons
(
)
36
1
400
f
x
dx
=
x
−
x
+
x

40 ∫0
40  3
0
1  59 200 


40  3 
1 480
.
=
3
=
80
1 80
1 1 3

2
(
)
36
1
400
f
x
dx
=
x
−
x
+
x

80  3
80 ∫0
0
1  156 800 


80 
3 
1 960
.
=
3
=
Corrigé Séquence 6 – MA01
157
© Cned - Académie en ligne
La calculatrice donne
1 480
1 960
= 493, 33... et
= 653, 33...
3
3
Pour 40 bijoux par mois le coût unitaire moyen est environ 493 €
Pour 80 bijoux par mois le coût unitaire moyen est environ 653 €
Exercice IV
CT (q ) = CF + CV (q )
Coût total
Coût fixe Coût variable
Comme C ( x ) est la dérivée de C ( x ) alors
ma
T
q
q
A = C ma ( x ) dx = CT ( x ) = CT (q ) − CT (0 ).
0
0
∫
On donne CV (0 ) = 0 et CF = 5 000.
D’où CT (0 ) = CF + CV (0 ) = 5 000 = CF
A = CT (q ) − CF = CT (q ) − 5 000.
On donne C (q ) = 3q 2 − 120q + 1 000.
ma
On peut écrire CT (q ) = CF + A d’après la question q
CT (q ) = CF + ∫ C ma ( x ) dx
0
q
soit CT (q ) = C F + ∫ (3x 2 − 120 x + 1 000)dx
0
q
CT (q ) = CF +  x 3 − 60 x 2 + 1 000 x 
0

CT (q ) = CF + q 3 − 60q 2 + 1 000q
Soit finalement CT (q ) = q 3 − 60q 2 + 1 000q + 5 000.
Exercice V
Partie A
Pour tout réel x de  0, 5 ; 25  , on a f '( x ) =
qui est l’expression proposée.
158
© Cned - Académie en ligne
Corrigé Séquence 6 – MA01
18
18 − 2x 2 + 16 x
− 2x + 16 =
ce
x
x
Comme x est positif sur l’intervalle  0, 5 ; 25  , le signe de f '( x ) est le même
que celui du numérateur ou, en divisant par 2, que celui de − x 2 + 8 x + 9. Pour
ce dernier trinôme ∆ = 82 − 4 × ( −1) × 9 = 100 donc il y a deux racines qui sont
x1 =
−8 − 10
−8 + 10
= 9 et x 2 =
= −1. Le trinôme, donc f '( x ), est du signe de
2 × ( −1)
2 × ( −1)
a (ici a = −1) sauf à l’intérieur des racines. Sur l’intervalle 0, 5 ; 25  on obtient
donc :
x
25
9
0,5
+
Signe de f '( x )
−
0
La fonction f est donc croissante sur 0, 5 ; 9  et décroissante sur 9 ; 25  .
a) f (1) = 18 × ln1− 1+ 16 − 15 = 0.
b) f (18 ) ≈ 1, 03 et f (19 ) ≈ −19, donc le produit f (18 ) × f (19 ) est négatif. Or la
fonction f est continue (car dérivable) et strictement décroissante sur 18 ; 19 
donc l’équation f ( x ) = 0 admet une solution unique α sur 18 ; 19  . Comme
f (18, 05) ≈ 0, 07 et f (18, 06 ) ≈ −0,117, on trouve α ≈ 18, 05 à 10−2 par défaut.
c) Les résultats précédents et les variations de f permettent d’en déduire le signe
de f ( x ) suivant les valeurs de x.
x
1
0,5
+
Signe de f '( x )
α
9
+
0
–
25
–
f (9)
Variation
de f
0
0
f (0,5)
Signe de f ( x )
f (25)
–
0
+
0
–
Donc f ( x ) est positif sur 1; α  et négatif sur 0, 5 ; 1 et sur α ; 25  .
Corrigé Séquence 6 – MA01
159
© Cned - Académie en ligne
Comme f ( x ) indique le bénéfice mensuel de l’entreprise, l’entreprise est
bénéficiaire lorsque f ( x ) > 0, c’est-à-dire lorsque 1< x < α. Le réel x désigne
le nombre de centaines de panneaux solaires fabriqués et vendus, donc, pour
être bénéficiaire l’entreprise doit fabriquer au moins 101 panneaux solaires et
au plus 1805.
Le bénéfice maximum est égal à f ( 9 ) en milliers d’euros. Or f ( 9 ) ≈ 87, 55,
donc le bénéfice maximum est égal à 87550 €, l’entreprise ne peut donc pas
réaliser un bénéfice mensuel de 100000 €.
Partie B
0, 5 ; 25  , la
fonction f a pour primitive, sur cet intervalle, la fonction F définie par
x3
x3
F ( x ) = 18G ( x ) −
+ 8 x 2 − 15x , soit F ( x ) = 18 x ln x −
+ 8 x 2 − 33x .
3
3
Comme G est une primitive de la fonction ln sur
On calcule la valeur moyenne de la fonction f sur 1; 18  :
m=
1 18
1
18
f ( x ) d x = F ( x ) ≈ 59, 753.
∫
1
1
18 − 1
17
La valeur moyenne du bénéfice mensuel de l’entreprise est donc environ 59800 €.
Exercice VI
a) La droite (OA ) a pour équation y = x . Sur  0 ; 1 le signe de la différence
)
(
d ( x ) = x − f ( x ) = x 1− e x −1 est le même que le signe de 1− e x −1.
Comme 0 ≤ x ≤ 1, on a −1 ≤ x − 1 ≤ 0 ce qui donne e −1 ≤ e x −1 ≤ e0 puisque la
fonction exponentielle est croissante sur
. On en déduit que e x −1 ≤ 1 donc
1− e x −1 ≥ 0. La différence d ( x ) est positive sur 0 ; 1 donc la courbe Ꮿ f est
située sous le segment [OA], la courbe Ꮿ f est une courbe de Lorenz.
0 ; 1 par F ( x ) = (ax + b )e x −1
est une primitive de la fonction f si F '( x ) = f ( x ) sur 0 ; 1 . Or
b)
La
fonction
F
définie
sur
F '( x ) = ae x −1 + (ax + b )e x −1 = (ax + (a + b ))e x −1, il suffit donc que les réels
a = 1
. Les nombres a = 1 et b = −1 conviennent, donc
a et b vérifient 
a + b = 0
la fonction F définie sur 0 ; 1 par F ( x ) = ( x − 1)e x −1 est une primitive de la
fonction f sur 0 ; 1 .
160
© Cned - Académie en ligne
Corrigé Séquence 6 – MA01
c)
La
situé
mesure,
entre
le
en
unités
segment
[OA]
1
d’aire,
et
la
de
courbe
1
l’aire
du
Ꮿf
est
domaine
égale
à
)
x2

x2

1
1 1
x −1 



= − −( −1)e −1 = − .
x
−
f
(
x
)
d
x
=
−
F
(
x
)
=
−
(
x
−
)
e
1
(
∫0
2
2 e
 2

 2

0
0 
1
L’aire du triangle OAI a pour mesure . Donc, si on suppose que la courbe Ꮿ f
2
)
1
(
représente la répartition de la masse salariale des ouvriers d’une entreprise,
1 1
−
l’indice de Gini correspondant est égal au quotient 2 e = 1− 2 ≈ 0,264.
1
e
2
On sait que la courbe de la fonction carré est située sous la droite (OA).
x3
La fonction G définie sur 0 ; 1 par G ( x ) =
est une primitive de la fonction
3
carré sur 0 ; 1 .
Comme
1
x3 
1 2
1
1 1 1
x d x =   = , l’aire de concentration vaut − =
et
0
3
2 3 6
 3 
0
∫
l’indice de Gini correspondant à une masse salariale représentée par la courbe de
1 1
−
1
la fonction carré est : 2 3 = ≈ 0,333.
1
3
2
J
A
Les deux coefficients de Gini montrent que
ces répartitions sont plutôt égalitaires, la
première plus que la seconde.
Les courbes montrent que les deux répartitions
sont différentes, surtout pour les salaires les
plus bas, alors que pour les salaires les plus
importants les deux répartitions sont assez
semblables puisque les courbes sont presque
confondues.
Cf
Cg
O
I
Corrigé Séquence 6 – MA01
161
© Cned - Académie en ligne
Exercice VII
Partie A
Résolutions graphiques
f ( x ) = 8 pour x = 3.
f ( x ) = 4 pour x = 8.
g ( x ) = 11 pour x = 12.
f ( x ) = g ( x ) pour x = 6.
f ( x ) ≥ g ( x ) pour 0 ≤ x ≤ 6.
Lecture graphique
f (0) = 20 ; f (14) = 2,5 ; g (0) = 3 ; g (3) = 3,5.
La fonction f est décroissante sur [ 0 ; +∞[ et la fonction g est croissante sur
[0 ; +∞[ .
Ainsi f ' ≤ 0 et g ' ≥ 0 sur [ 0 ; +∞[ .
Partie B
Interprétation graphique
a) ▶ Le prix est de 80 euros ; les prix sont en ordonnée et exprimés en dizaines
d’euros. On cherche donc x tel que f ( x ) = 8. On trouve x = 3.
Les quantités sont en centaines d’unités.
Pour un prix de 80 euros la quantité achetée est égale
à 300 unités.
Conclusion
▶ De
Prix en euros
Quantité achetée
80
300
40
800
même si le prix est de 40 euros, on a f (8 ) = 4.
Pour un prix de 40 euros la quantité achetée est égale
à 800 unités.
b) Le prix est de 110 euros. Pour trouver la quantité
vendue on cherche x tel que g ( x ) = 11. On trouve
x = 12.
Si le prix est 110 euros la quantité vendue est égale à 1 200 unités.
c) Pour la courbe Ꮿ g l’ordonnée minimale est égale à 3. Cette ordonnée
correspond à un prix de 30 euros.
Le prix « plancher » de la centaine d’unités est de 30 euros.
162
© Cned - Académie en ligne
Corrigé Séquence 6 – MA01
Équilibre du marché
Les deux courbes Ꮿf et Ꮿ g semblent se couper au point K ( 6 ; 5 ) .
Vérifions-le par le calcul.
40
= 5. Ainsi K ∈Ꮿf .
On a f (6 ) =
8
1
On a g (6 ) = × 36 + 3 = 5. Ainsi K ∈Ꮿ g .
18
Les coordonnées du point K sont donc K (6 ; 5).
Le prix d’équilibre p0 , exprimé en dizaines d’euros, est égal à 5.
Le prix d’équilibre est de 50 euros.
La situation est favorable au consommateur
Remarque
lorsque g ( x ) > f ( x ) et défavorable aux
Pour x = 6 la situation est équilibrée (ni
favorable, ni défavorable).
consommateur lorsque g ( x ) < f ( x ).
Pour 0 ≤ x < 6 la situation est défavorable
aux consommateurs.
Pour 6 < x la situation est favorable aux
consommateurs.
Rente des producteurs
x
On donne PP = x 0 p0 − ∫ 0 g ( x ) dx en milliers d’euros.
0
1
Comme g ( x ) = x 2 + 3, une primitive G de g est définie pour x ≥ 0 par
18
1 3
G ( x ) = x + 3x .
54
6
 1 3

P
=
30
−
x
+
3
x
D’où p

 54
0
 216

Pp = 30 − 
+ 18 = 30 − 22.
 54

Pp = 8 en milliers d’euros.
Le profit des producteurs est égal à 8 000 euros.
L’intégrale
6
∫0 g ( x ) dx
est égale à l’aire, exprimée en u.a., du domaine plan limité
par la courbe Ꮿ g , l’axe des abscisses et les droites d’équation x = 0 et x = 6.
Le produit x0p0 est la mesure de l’aire du rectangle limité par l’axe des abscisses,
la droite d’équation p = p0, l’axe des ordonnées et la droite d’équation x = x0.
Ainsi Pp est la mesure de l’aire du domaine limité par la courbe Ꮿ g , l’axe des
ordonnées et la droite parallèle à l’axe des abscisses d’équation p = 5.
Corrigé Séquence 6 – MA01
163
© Cned - Académie en ligne
Cette aire est coloriée sur la figure.
f : fonction de demande
40
f (x ) =
x +2
PRIX
20 (en dizaines d'euros)
g : fonction d’offre
g(x ) =
1 2
x +3
18
Ꮿg
15
10
K
5
4
Ꮿf
3
2
1
x
O
1
2
3
4
5
6
10
QUANTITÉ (en centaines)15
164
© Cned - Académie en ligne
Corrigé Séquence 6 – MA01
C orrigé Séquence 7
Corrigé des activités du chapitre 2
Activité 1
La probabilité d’obtenir 3 en lançant un dé cubique non truqué est égale
1
≈ 0,16667 car on utilise la loi équirépartie sur Ω = 1, 2, 3, 4 , 5, 6 . De
6
même, en lançant un dé dodécaédrique non truqué cette probabilité est égale
1
à
≈ 0, 08333.
12
{
à
}
Pour calculer la probabilité d’obtenir le nombre d en choisissant au hasard
dans [0 ; 1[ un nombre décimal s’écrivant avec au plus 9 chiffres après la virgule,
−9
on utilise la loi équirépartie. On trouve donc 10 .
par f ( x ) = x 3 + x − 1. La fonction f est
continue, dérivable sur , strictement croissante sur (car f '( x ) = 3x 2 + 1 > 0
a) Soit f la fonction définie sur
sur ). On a f (0 ) = −1 et f (1) = 1, on obtient donc le tableau de variation
suivant :
x
−∞
0
α
1
+∞
1
f (x )
0
−1
b) On choisit au hasard un nombre X de l’intervalle 0 ; 1 . Dans les questions
précédentes, les probabilités obtenues avec des lois équiréparties sont de plus en
plus proches de 0 car les univers contiennnent de plus en plus d’éléments. Il y a
maintenant un nombre infini de possibilités. La situation est nouvelle. On propose
P ( X = α ) = 0.
Corrigé Séquence 7 – MA01
165
© Cned - Académie en ligne
Commentaire
Dans le cas où l’univers est fini, la probabilité d’un événement A est
P (A) =
∑
ai ∈ A
P (ai ), où A est formé par la réunion des ai . De façon analogue,
dans le cas fini, pour une variable aléatoire on a P ( X ∈ E ) =
∑
P ( X = x i ).
x i ∈E
On ne peut pas généraliser ceci puisque, ici, d’une part, E peut avoir une infinité
d’éléments et d’autre part toutes les probabilités P ( X = x i ) sont nulles ce qui
donnerait une somme nulle, ce qui, bien sûr, ne convient pas.
On est donc amené à définir différemment la probabilité d’un événement comme
on le verra dans le cours.
Activité 2
[0 ; 20[
Montant des achats (en €)
[20 ; 40[ [40 ; 60[ [60 ; 100[ [100 ; 140[ [140 ; 200]
Amplitude de la classe =
20
largeur du rectangle
Aire du rectangle
0,09
Hauteur du rectangle
0,0045
20
20
40
40
60
0,20
0,01
0,22
0,011
0,24
0,006
0,16
0,004
0,09
0,0015
La graduation maximale indiquée sur l’axe des ordonnées est 0,011, c’est la
hauteur du troisième rectangle.
Ce ne sont pas les fréquences qui sont indiquées sur l’axe des ordonnées car les
fréquences sont les mesures des aires des rectangles.
Les hauteurs des rectangles sont obtenues en divisant les aires par les amplitudes,
c’est-à-dire en divisant les fréquences par les amplitudes. On dit que les résultats
obtenus sont des densités de fréquence. Ce sont les densités de fréquence qui
sont indiquées sur l’axe des ordonnées.
densité
de fréquence
2,5 %
0,01
montant des
achats en €
0
166
© Cned - Académie en ligne
20
40
Corrigé Séquence 7 – MA01
60
100
140
200
Remarque
▶ La
classe modale est la classe qui a la plus grande densité (la troisième
pour cet exemple), elle ne coïncide pas toujours avec celle qui a la plus
grande fréquence (la quatrième ici).
▶ Quand les classes ont toutes la même amplitude, les hauteurs des rectangles
sont proportionnelles aux aires, les densités sont proportionnelles aux
fréquences, il peut alors exister une confusion sur la nature des graduations
de l’axe des ordonnées (confusion volontaire ou involontaire).
▶ Quand
l’axe des ordonnées est gradué et nommé, on peut enlever le
rectangle qui donne l’unité pour les aires.
▶ La
densité de fréquence peut être supérieure à 1 (ce n’est pas une
fréquence !).
Corrigés des exercices
d’apprentissage du chapitre 2
Exercice 1
Il ne s’agit pas d’une densité de probabilité car l’aire sous la courbe ne mesure
1
pas 1 mais 2 × .
3
La fonction représentée est constante
Remarque
sur I = 0 ; 0, 5  , donc continue. La
constante vaut 2 qui est positif. L’aire
sous la courbe est celle d’un rectangle
1
qui mesure 2 × = 1. Il s’agit bien d’une
2
densité de probabilité.
Remarque
Cette densité est supérieure à 1,
ce qui est possible car il s’agit
d’une densité et non pas d’une
probabilité.
La fonction n’est pas continue en 0,5 et en 1,5, il ne s’agit
donc pas d’une densité de probabilité.
L’intervalle I contient des nombres négatifs ce qui est tout à fait
possible !
La fonction est affine par morceaux sur I =  −1; 1 , elle
est donc continue. Elle est à valeurs positives et l’aire sous
la courbe mesure
2×1
= 1. Il s’agit donc d’une densité de
2
probabilité.
Corrigé Séquence 7 – MA01
167
© Cned - Académie en ligne
Exercice 2
La fonction f est une fonction polynôme, elle est donc continue sur  0 ; 1 ;
elle est à valeurs positives.
1
1
3
4
∫0 4 x dx =  x 0 = 1 donc
la fonction f est bien une densité de
On
a
4
probabilité.
)
(
a) La probabilité P 0, 5 ≤ X ≤ 0, 75 est
la mesure de l’aire colorée sur le graphique.
1
On a
O
P ( 0,5 ≤ X ≤ 0,75) = ∫
O,5 O,75 m
1
0,75
0,75
4 x 3 dx =  x 4 
 0,5
0,5
= 0,754 − 0,54 ≈ 0,254.
b) On a
P ( X ≤ m ) = 0, 5 ⇔ ∫ 4 x 3 d x = 0, 5
m
0
4
⇔ m = 0, 5
Remarque
2
⇔ m = 0, 5
⇔m =
D’où m ≈ 0, 84.
0, 5 .
L’aire sous la courbe est partagée en
deux parties d’aire 0,5 par le segment
formé par les points d’abscisse m (en
pointillés sur le graphique). On dit
parfois que m est la médiane.
Corrigés des exercices
d’apprentissage du chapitre 3
Exercice 3
Comme le facteur peut arriver à tout instant avec les mêmes chances, on
modélise son heure d’arrivée par la variable aléatoire F qui suit la loi uniforme
sur 0 ; 30  .
1
1
Sa densité est donc la fonction constante sur 0 ; 30  qui vaut
= .
30 − 0 30
168
© Cned - Académie en ligne
Corrigé Séquence 7 – MA01
a) On a P (F = 25) = 0 : c’est le cas pour toutes les probabilités de la forme
P ( X = α ) pour toute valeur α prise par une variable aléatoire à densité. Par
conséquent, dans ce qui suit, on peut écrire des inégalités larges ou strictes, des
intervalles ouverts ou fermés sans que cela change les résultats.
20 − 15 1
= .
b) On a P (15 < F < 20 ) = P X ∈15 ; 20  =
30 − 0 6
)
(
20 − 0 2
= .
c) On a P (F < 20 ) = P F ∈0 ; 20  =
30 − 0 3
)
(
30 − 15 15 1
=
= .
d) On a P (F > 15) = P F ∈15 ; 30  =
30 − 0 30 2
)
(
L’heure moyenne de son passage est donnée par l’espérance de la variable
aléatoire F.
Comme E(F ) =
10 h 15 min.
Exercice 4
0 + 30
= 15 on peut dire que le facteur passe en moyenne à
2
Le tram passe à 7 h, 7 h 10 et 7 h 20. L’événement E « Ayana attend le tram
{ {}
}
moins de deux minutes » est l’événement E = X ∈ 7 ∪ 8 ; 10  ∪ 18 ; 20  .
On a donc :
P (E ) = P ( X = 7) + P ( 8 ≤ X ≤ 10 ) + P (18 ≤ X ≤ 20 )
= 0+
10 − 8 20 − 18 4
+
= = 0, 2.
200 − 0 20 − 0 20
L’événement F « Ayana attend le tram plus de cinq minutes » est l’événement
{
}
F = X ∈0 ; 5  ∪ 10 ; 15  .
On a donc :
P (F ) = P ( 0 < X ≤ 5) + P (10 < X ≤ 15)
=
Exercice 5
5 − 0 15 − 10 10
+
= = 0, 5.
20 − 0 20 − 0 20
Les valeurs prises par la variable aléatoire T correspondent aux dix chiffres
possibles : 0, 1, 2,… , 9.
Il s’agit donc d’une variable aléatoire discrète, prenant un nombre fini de valeurs.
Corrigé Séquence 7 – MA01
169
© Cned - Académie en ligne
La
variable
aléatoire
T
est
la
première
décimale
de
X
donc
P (T = 0 ) = P ( 0 ≤ X < 0,1) = 0,1− 0 = 0,1 puisque X suit la loi uniforme sur
0 ; 1 .
)
On a aussi P (T = 1) = P ( 0,1 ≤ X < 0, 2 = 0, 2 − 0,1 = 0,1.
On obtient ainsi P (T = k ) = 0,1 pour tout entier k tel que 0 ≤ k < 10.
{
}
La loi de T est donc la loi équirépartie sur l’ensemble fini 0, 1, 2,... , 8, 9 .
Corrigé de l’activité 5 du chapitre 4
Activité 5
Étude de la fonction définie sur
par f ( x ) =
1
2π
x2
e 2.
−
Pour tout réel x, on a f ( − x ) = f ( x ) puisque x n’apparaît que par x 2. Deux points
de la courbe représentative de f d’abscisses opposées ont donc la même ordonnée,
on en déduit que la courbe est symétrique par rapport à l’axe des ordonnées.
La fonction f est de la forme
1
2π
eu où u est la fonction définie sur
x2
. La fonction f est donc dérivable sur et, pour tout réel x
2
x2
−
−x
on a f '( x ) =
e 2 qui est du signe contraire de x. La fonction f est donc
2π
par u ( x ) = −
croissante sur ]−∞ ; 0 ] et décroissante sur [ 0 ; + ∞[ .
La calculatrice donne
10
∫−10f (t ) dt ≈ 1.
Cette valeur et l’allure de la courbe permettent de conjecturer que l’aire tout
entière mesure une unité d’aire.
1 / 2/ 0,4
C’est pour obtenir ce résultat que le
1
coefficient
est introduit dans la
2π
fonction f (ce coefficient peut paraître
un peu surprenant mais sa justification
O
170
© Cned - Académie en ligne
Corrigé Séquence 7 – MA01
1
n’est pas accessible en terminale).
La fonction f est continue sur
puisqu’elle est dérivable, elle est à valeurs
positives comme l’exponentielle, l’aire sous la courbe de f semble égale à 1 u.a.,
la fonction semble donc être une densité de probabilité.
Corrigé des exercices d’apprentissage
du chapitre 4
Exercice 6
X suit la loi ᏺ(0 ; 1).
a) P ( −0, 5 ≤ X ≤ 1, 3) ≈ 0, 5946
b) P ( X ≤ −1) ≈ 0,1586
c) P ( X ≥ 1, 8 ) ≈ 0, 0359.
Le réel x tel que P ( X ≤ x ) = 0, 8 est tel que x ≈ 0, 8416.
Exercice 7
X suit la loi ᏺ(12 ; 9 ).
a) P (10 ≤ X ≤ 14) ≈ 0,4950 (attention σ = 3) b) P ( X ≤ 13) ≈ 0, 6305
c) P ( X ≥ 7) ≈ 0, 9522.
Le réel x tel que P ( X ≤ x ) = 0,9 est tel que x ≈ 15, 8446.
Exercice 8
Chaque courbe est symétrique par rapport à la droite d’équation x = µ et,
l’écart-type σ mesurant la dispersion, plus il est important, plus la courbe est
« étalée ». On en déduit donc que
▶ la
courbe 1 représente la fonction de densité de la loi ᏺ(5 ; 0, 52 ),
▶ la
courbe 2 représente la fonction de densité de la loi ᏺ(7 ; 1),
▶ la
courbe 3 représente la fonction de densité de la loi ᏺ(5 ;1),
▶ la
courbe 4 représente la fonction de densité de la loi ᏺ(7 ; 22 ).
La courbe C est symétrique par rapport à la droite d‘équation x = 9 et elle est
« étalée » comme la courbe 1, elle représente donc la fonction de densité de la
loi normale ᏺ(9 ; 0, 52 ).
Exercice 9
On utilise la variable aléatoire Z =
ᏺ(0 ; 1).
X − µ X − 15
=
, Z suit la loi normale
σ
σ
Corrigé Séquence 7 – MA01
171
© Cned - Académie en ligne
Comme X < 12 ⇔ σZ + 15 < 12 ⇔ Z <
calculatrice donne

−3
−3 
, on obtient P  Z <  = 0, 4. La
σ
σ

−3
≈ −0,2533 on en déduit σ ≈ 11, 84.
σ
On utilise la variable aléatoire Z =
ᏺ(0 ; 1).
Comme X < 7 ⇔ 0, 5Z + µ < 7 ⇔ Z <
La calculatrice donne
Exercice 10
X −µ X −µ
=
, Z suit la loi normale
σ
0, 5

7− µ
7− µ
, on obtient P  Z <
= 0, 3.
0, 5
0, 5 

7− µ
≈ −0, 5244 , on en déduit µ ≈ 7, 2622.
0, 5
X suit la loi normale ᏺ(61, 5 ; 0, 42 ).
La probabilité qu’une pièce tirée au hasard, ait un diamètre compris entre
60,7 mm et 62,3 mm vaut P (60,7 ≤ X ≤ 62,3) ≈ 0,9545.
Il s’agit de la probabilité de l’événement contraire, soit environ 0, 0455.
Exercice 11
P (( X ≤ 61) ∩ (60, 7 ≤ X ≤ 62, 3)) P (60, 7 ≤ X ≤ 61)
≈ 0, 0868.
=
P (60, 7 ≤ X ≤ 62, 3)
P (60, 7 ≤ X ≤ 62, 3)
2
X suit la loi normale ᏺ(6000 ; 400 ).
(
)
P 5800 ≤ X ≤ 6200 ≈ 0, 3829.
P ( X ≤ 5700 ) ≈ 0, 2266.
P ( X ≥ 6400 ) ≈ 0,1586.
Les vaches qui produisent moins de x litres de lait par an forment 30% du
troupeau. Le réel x tel que P ( X ≤ x ) = 0, 30 correspond donc à la production
maximale des 30 % des vaches les moins productives du troupeau. On trouve
x ≈ 5790 litres.
La production minimale prévisible des 20% des vaches les plus productives du
troupeau est le nombre x tel que P ( X ≥ x ) = 0, 20 c’est-à-dire P ( X ≤ x ) = 0, 80
et la calculatrice donne x ≈ 6336 litres.
172
© Cned - Académie en ligne
Corrigé Séquence 7 – MA01
Corrigés des exercices
de synthèse de la séquence 7
Exercice I
La corde AM a une longueur supérieure à
3 lorsque le point M appartient
(qui est en trait épais). En appliquant la loi uniforme aux
au « petit » arc BC
longueurs des arcs, la probabilité demandée est égale au quotient des longueurs
1
de l’arc et du cercle, on obtient .
3
A
M
3
3
I
O
I
3
B
C
I
O
3
E
D
M
On obtient la corde de milieu I en traçant la perpendiculaire à la droite (OI). La
corde de milieu I a une longueur supérieure à
3 lorsque le point I appartient
au segment [OE], le point E étant le milieu de [OD]. En appliquant la loi uniforme
aux longueurs des segments du segment OD  , la probabilité demandée est
1
égale au quotient des longueurs des segments [OE] et OD  , on trouve .
2
La corde de milieu I a une longueur supérieure à
inférieure à
3 lorsque la distance OI est
1
, c’est-à-dire lorsque le point I appartient au disque de centre O et
2
1
. En appliquant la loi uniforme aux aires incluses dans le disque de
2
2
 1
π 
 2
1
= .
centre O et de rayon 1, on trouve la probabilité
2
4
π ×1
de rayon
Corrigé Séquence 7 – MA01
173
© Cned - Académie en ligne
On observe que les probabilités obtenues sont différentes. L’expression
« choisie au hasard » n’est donc pas assez précise dans cet exemple puisque le
résultat dépend de la façon de choisir la corde. Ce phénomène est connu sous le
nom de « paradoxe de Bertrand » du nom de Joseph Bertrand qui a mis cela en
évidence en 1888.
Exercice II
a) On souhaite que P ( X > 83) ≤ 0, 01. On pose Z =
X −µ X −µ
=
, d’où
σ
2

83 − µ 
X = 2Z + µ ce qui s’écrit aussi P (2Z + µ > 83) ≤ 0, 01, soit P  Z >
≤ 0, 01
2 


83 − µ 
ou encore P  Z ≤
≥ 0, 99.
2 

La variable aléatoire Z suit la loi normale ᏺ(0 ; 1), pour P (Z ≤ z ) = 0, 99 la
calculatrice donne z 1 ≈ 2,327. La fonction z P (Z ≤ z ) est croissante (l’aire
sous la courbe augmente quand z augmente), on obtient donc 83 −µ ≥ 2,327
2
d’où µ ≤ 78, 35.
b) Si X suit la loi ᏺ (78,35 ; 4), on trouve P ( X ≤ 75) ≈ 0, 047.
La législation n’est pas respectée car environ 4 % des bouteilles contiennent
moins de 75 cl.
a) Pour respecter la législation on cherche µ tel que P ( X ≤ 75) ≤ 0, 001
soit
P (2Z + µ ≤ 75) ≤ 0, 001
ou
encore

75 − µ 
P Z ≤
≤ 0, 001.
2 

P (Z ≤ z ) = 0, 001, la calculatrice donne z 2 ≈ −3, 09. D’où
Pour
75 − µ
≤ −3, 09 soit
2
µ ≥ 81,18.
b) On choisit µ = 81,18 et la probabilité qu’une bouteille déborde au
remplissage est alors P ( X ≥ 83) ≈ 0,181 soit environ 18 %.
On cherche maintenant µ et σ tels que P ( X ≤ 75) ≤ 0, 001 et P ( X ≥ 83) ≤ 0, 01,


75 − µ 
83 − µ 
c’est-à-dire tels que P  Z ≤
≤ 0, 001 et P  Z ≥
≤ 0, 01.

σ 
σ 


174
© Cned - Académie en ligne
Corrigé Séquence 7 – MA01
75 − µ
83 − µ
≤ z 2 et
≥ z1, c’est-àσ
σ
dire 75 − σz 2 ≤ µ ≤ 83 − σz1 soit à peu près 75 + 3, 09σ ≤ µ ≤ 83 − 2, 326σ.
Les nombres µ et σ doivent donc vérifier
La condition sur les valeurs extrêmes est 75 + 3, 09σ ≤ 83 − 2, 326σ. Il suffit
donc de choisir σ ≤ 1, 47 puis une valeur de µ comprise entre les deux valeurs
extrêmes.
Exercice III
La
probabilité
qu’une
plaque
soit
conforme
est
égale
à
P ( µ − 3σ ≤ X ≤ µ + 3σ ) ≈ 0, 997 donc la probabilité qu’une plaquette ne soit
pas conforme vaut environ 0,003.
Première méthode
On utilise la variable aléatoire Z =
ᏺ(0 ;1).
Comme
X − µ X − 125
=
, Z suit la loi normale
σ
0, 5
µ − h ≤ X ≤ µ + h ⇔ 125 − h ≤ 0, 5Z + 125 ≤ 125 + h ⇔ −2h ≤ Z ≤ 2h ,
on cherche h tel que P ( −2h ≤ Z ≤ 2h ) = 0, 99.
On sait que la courbe de la loi centrée réduite ᏺ(0 ; 1) est symétrique par rapport
à l’axe des ordonnées donc P ( −2h ≤ Z ≤ 0 ) = P (0 ≤ Z ≤ 2h ) =
On sait que P (Z ≤ 0 ) =
0, 99
= 0, 495.
2
1
et, comme
2
P (Z ≤ 2h ) = P (Z ≤ 0 ) + P (0 ≤ Z ≤ 2h ) = 0, 5 + 0, 495 = 0, 995,
il suffit de chercher h tel que P (Z ≤ 2h ) = 0, 995.
La calculatrice donne alors 2h ≈ 2, 576, d’où h ≈ 1, 29 et les poids d’alerte sont
µ − h ≈ 123, 71 et µ + h ≈ 126, 29.
Grâce à des échantillons prélevés en fin de chaîne, ces poids d’alerte permettent
de déceler l’existence d’anomalies de fonctionnement avant le dépassement des
normes µ − 3σ et µ + 3σ.
Deuxième méthode
En utilisant la symétrie de la courbe de la fonction de densité de la
loi ᏺ( µ ; σ 2 ) par rapport à la droite d’équation x = µ, on obtient
Corrigé Séquence 7 – MA01
175
© Cned - Académie en ligne
1
P ( µ ≤ X ≤ µ + h ) = P ( µ − h ≤ X ≤ µ + h ) = 0, 495. La même symétrie prouve
2
1
que P ( X ≤ µ ) = , on cherche donc h tel que P ( X ≤ µ + h ) = 0, 995 car
2
1
P ( X ≤ µ + h ) = P ( X ≤ µ ) + P (µ ≤ X ≤ µ + h ) = + P (µ ≤ X ≤ µ + h )
2
= 0,5 + 0,495 = 0,995.
La calculatrice donne µ + h ≈ 126, 29, d’où h ≈ 1, 29 et µ − h ≈ 123,71.
176
© Cned - Académie en ligne
Corrigé Séquence 7 – MA01
C orrigé Séquence 8
Corrigé des activités du chapitre 2
Activité 1
Tangentes et parabole
Partie A
Soit Ꮿ la parabole d’équation y = c ( x ) = x 2 et T la tangente à Ꮿ au point
K
K (1 ; 1).
La tangente TK a pour équation y = 2x − 1
Pour situer Ꮿ par rapport à TK
on étudie le signe de la différence
d ( x ) = x 2 − (2x − 1).
Pour tout x réel on a d ( x ) = x 2 − 2x + 1 = ( x − 1)2 , d’où d ( x ) ≥ 0.
▶ La
courbe Ꮿ est entièrement située au-dessus de la tangente en K.
Soit A(a ; a 2 ) un point quelconque de la courbe Ꮿ et T la tangente à Ꮿ
A
au point A(a ; a 2 ).
La tangente TA a pour équation y = 2a ( x − a ) + a 2 , soit y = 2ax − a 2 .
Pour situer Ꮿ par rapport à TA on étudie le signe de la différence
d ( x ) = x 2 − (2ax − a 2 ) = x 2 − 2ax + a 2 = ( x − a )2.
Pour tout x réel on a d ( x ) ≥ 0.
▶ La
courbe Ꮿ est entièrement située au-dessus de la tangente en A.
Conclusion
La parabole Ꮿ d’équation y = x2 est située au-dessus de toutes ses
tangentes.
Les tangentes à Ꮿ aux points d’abscisses a = −2 , a = −1 , a = 0 , a = 1 , a = 2
sont tracées sur la figure 1.
La courbe Ꮿ d’équation y = f ( x ) = − x 2 est la courbe symétrique de la
f
courbe Ꮿ par rapport à l’axe des abscisses.
Corrigé Séquence 8 – MA01
177
© Cned - Académie en ligne
Les tangentes à la courbe Ꮿf sont les droites symétriques des tangentes à la
courbe Ꮿ par rapport à l’axe des abscisses.
On en déduit que la courbe Ꮿf est entièrement située en dessous de toutes ses
tangentes.
Conclusion
La parabole Ꮿf d’équation y = − x 2 est située en dessous de toutes ses
tangentes.
Partie B
Soit ᏾ la courbe d’équation y = r ( x ) =
K (1 ; 1).
x et TK la tangente à ᏾ au point
La tangente TK a pour équation y = 0, 5x + 0, 5 soit y = 0, 5( x + 1).
Pour situer ᏾ par rapport à TK on étudie le signe de la différence
d ( x ) = x − (0, 5x + 0, 5) = −0, 5( x − 2 x + 1).
Pour tout x ≥ 0 on a d ( x ) = −0, 5( x − 1)2 , d’où d ( x ) ≤ 0.
▶ La
courbe ᏾ est entièrement située en dessous de la tangente en K.
Soit A(a ; a ) un point quelconque de la courbe ᏾ et T la tangente à ᏾
A
au point A(a ; a ), avec a > 0.
1
1
(x − a ) + a =
( x − a + 2a ), soit
La tangente TA a pour équation y =
2 a
2 a
1
y=
( x + a ).
2 a
Pour situer ᏾ par rapport à TA on étudie le signe de la différence
1
1
d (x ) = x −
(x + a ) = −
x −2 a x +a .
2 a
2 a
1
( x − a )2 , d’où d ( x ) ≤ 0.
Pour tout x ≥ 0 on a d ( x ) = −
2 a
▶ La courbe ᏾ est entièrement située en dessous de la tangente en A.
(
)
Conclusion
La courbe ᏾ d’équation y = x est située en dessous de toutes ses
tangentes.
178
© Cned - Académie en ligne
Corrigé Séquence 8 – MA01
Les tangentes à ᏾ aux points d’abscisses a = 1 , a = 2 , a = 4 sont tracées
sur la figure 2.
La courbe Ꮿ d’équation y = g ( x ) = − x est la courbe symétrique de la
g
courbe ᏾ par rapport à l’axe des abscisses.
Les tangentes à la courbe Ꮿ g sont les droites symétriques des tangentes à la
courbe ᏾ par rapport à l’axe des abscisses.
On en déduit que la courbe Ꮿ g est entièrement située au-dessus de toutes ses
tangentes.
Conclusion
La courbe Ꮿ g d’équation y = − x est située au-dessus de toutes ses
tangentes.
y
y
4
2
3
2
K
1
-2
-1
Ꮿ y = x2
1
0
-1
2
x
᏾ y = 兹x
1
K
-2
-3
-4
0
1
2
Figure 1
Activité 2
3
4
5 x
Figure 2
Tangentes et hyperbole
Soit ᐄ l’hyperbole d’équation y = i ( x ) =
1
, définie pour x ≠ 0.
x
On se place sur l’intervalle ]0 ; + ∞[
La tangente TK , tangente à la courbe ᐄ au point K (1 ; 1), a pour équation
y = −x + 2 .
Pour situer ᐄ par rapport à TK
d (x ) =
on étudie le signe de la différence
1
1 x 2 − 2x + 1
− ( − x + 2) = x − 2 + =
.
x
x
x
Corrigé Séquence 8 – MA01
179
© Cned - Académie en ligne
Pour tout x > 0 on a d ( x ) =
▶ Sur
x 2 − 2x + 1 ( x − 1)2
=
, d’où d ( x ) ≥ 0.
x
x
l’intervalle ]0 ; + ∞[ la courbe ᐄ est entièrement située au-dessus de la
tangente en K.
On se place sur l’intervalle ] − ∞ ; 0[.
La tangente TH , tangente à la courbe ᐄ au point H (– 1 ; – 1), a pour équation
y = − x − 2.
Pour situer ᐄ par rapport à TH on étudie le signe de la différence
1
1 x 2 + 2x + 1
d ( x ) = − ( − x − 2) = x + 2 + =
.
x
x
x
Pour tout x < 0 on a d ( x ) =
x 2 + 2x + 1 ( x + 1)2
=
, d’où d ( x ) ≤ 0.
x
x
l’intervalle ] − ∞ ; 0[ la courbe ᐄ est entièrement située en dessous de la
tangente en H.
▶ Sur

1
La tangente T , tangente à la courbe ᐄ au point A  a ;  , a pour équation
A
 a
y =−
1
1
1
( x − a ) + = − ( x − a − a ).
a
a2
a2
La tangente TA a pour équation y = −
1
( x − 2a ).
a2
Pour situer ᐄ par rapport à TA on étudie le signe de la différence
d (x ) =
1 1
1 x 2 − 2ax + a 2
+ ( x − 2a ) = ×
.
x a2
x
a2
se place sur l’intervalle ]0 ; + ∞[
1 ( x − a )2
, d’où d ( x ) ≥ 0.
Pour tout x > 0 on a d ( x ) = 2 ×
x
a
▶ On
Conclusion
Sur l’intervalle ]0 ; + ∞[, l’hyperbole ᐄ d’équation y =
dessus de toutes ses tangentes.
180
© Cned - Académie en ligne
Corrigé Séquence 8 – MA01
1
est située aux
▶ On
se place sur l’intervalle ] − ∞ ; 0[
Pour tout x < 0 on a d ( x ) =
1
a2
×
( x − a )2
, d’où d ( x ) ≤ 0.
x
Conclusion
1
Sur l’intervalle ] − ∞ ; 0[, l’hyperbole ᐄ d’équation y =
est située en
x
dessous de toutes ses tangentes.
Les tangentes à ᐄ aux points d’abscisses a = – 2, a = – 1, a = −
1
1
, a= ,
2
2
a = 1 , a = 2 sont tracées sur la figure 3.
y
3
1
Ᏼ y =—
x
2
K
1
–3
–2
–1
0
1
2
3
4
x
–1
H
–2
Figure 3
Corrigé Séquence 8 – MA01
181
© Cned - Académie en ligne
Tangentes aux courbes des fonctions " exp " et " ln "
Activité 3
Soit C
exp
la courbe d’équation Soit
C exp
la
courbe
d’équation
y = exp( x ) = e x et TK la tangente à
y = exp( x ) = e x et TA la tangente à C exp
C exp au point K (0 ; 1).
au point A(a ; ea ).
La tangente TK a pour équation
La
tangente
TA
a
pour
équation
y = ea ( x − a ) + ea , soit
y=x+1 .
Pour situer C exp par rapport à TK
on étudie le signe de la différence
y = ea x + ea (1− a ).
Pour situer C exp par rapport à TA on étudie
d ( x ) = e x − ( x + 1).
le signe de d a ( x ) = e x − ea x − ea (1− a ).
Étudions la fonction d et déterminons Étudions la fonction d et déterminons son
a
son signe.
signe.
On a d '( x ) = e x − 1.
On a d a '( x ) = e x − ea .
D’où ▶ d '( x ) = 0 pour x = 0 ;
▶ d '( x ) > 0
pour x > 0 ;
▶ d '( x ) < 0
pour x < 0.
x
Ꮿ
0
TA
d '( x )
D’où ▶ d a '( x ) = 0 pour x = a ;
–
d (x )
0
+
0
Pour tout x réel on a d ( x ) ≥ 0.
Conclusion
La courbe C exp est entièrement
située au-dessus de la tangente
en K (0 ; 1).
182
© Cned - Académie en ligne
Corrigé Séquence 8 – MA01
▶ d
a '( x ) > 0 pour x > a ;
▶ d
a '( x ) < 0 pour x < a.
x
d a '( x )
Ꮿ
a
TA
–
da ( x )
0
+
0
Pour tout x réel on a d a ( x ) ≥ 0.
Conclusion
La courbe C exp est entièrement
située au-dessus de la tangente
en A(a ; ea ).
Les tangentes à C
exp aux points d’abscisses a = – 1, a = 0, a = 1 sont tracées
sur la figure 4.
y
3
e
2
y = exp(x)
–2
K
1
–1
0
1H
2
e 3
x
–1
–2
y = In(x)
Figure 4
y = x –1
2
y = In(x)
1
H
–1
0
1
2
3
4
–1
y = —x+1
Figure 5
y = —In(x)
–2
Corrigé Séquence 8 – MA01
183
© Cned - Académie en ligne
Soit C la courbe d’équation y = ln x et T Soit C la courbe d’équation y = ln x et T
ln
H
ln
A
la tangente à C ln au point H(1 ; 0).
la tangente à C ln au point A(a ; lna ).
La tangente TH a pour équation y = x − 1.
Pour situer C ln par rapport à TH on étudie le signe
de la différence d ( x ) = lnx − ( x − 1).
La tangente TA a pour équation
1
y = ( x − a ) + lna.
a
Pour situer C ln par rapport à TA on étudie le
1
signe de d a ( x ) = lnx − ( x − a ) − lna.
a
Étudions la fonction d et déterminons son signe.
Étudions la fonction d a et déterminons son signe.
1
1− x
.
On a d '( x ) = − 1 =
1 1 a−x
x
x
.
On a d a '( x ) = − =
x a
x
D’où ▶ d '( x ) = 0 pour x = 1
D’où ▶ d a '( x ) = 0 pour x = a.
▶ d '( x ) > 0 pour 0 < x < 1
▶ d '( x ) > 0 pour 0 < x < a
a
▶ d '( x ) < 0
x
pour 1 < x.
0
d '( x )
▶ d
Ꮿ
1
+
0
–
0
d (x )
a '( x ) < 0 pour a < x.
x
Ꮿ
a
0
+
d a '( x )
0
–
0
da ( x )
Pour tout x > 0 on a d ( x ) ≤ 0.
Conclusion
Pour tout x > 0 on a d a ( x ) ≤ 0.
Conclusion
La courbe C ln est entièrement
située en dessous de la tangente
La courbe C ln est entièrement
située en dessous de la tangente
en H (1 ; 0).
en A(a ; lna ).
Les tangentes à C
sur la figure 4.
ln aux points d’abscisses a = e
−1
, a = 1, a = e sont tracées
Soit h la fonction définie sur ]0 ; + ∞ [ par h ( x ) = − ln x et (C) sa courbe
représentative.
Les fonctions h et ln sont opposées : les courbes C ln et (C) sont symétriques par
rapport à l’axe des abscisses : voir figure 5.
D’après la symétrie axiale on peut dire que la courbe (C) est située, sur ]0 ; + ∞ [,
au-dessus de ses tangentes.
184
© Cned - Académie en ligne
Corrigé Séquence 8 – MA01
Corrigé des exercices
d’apprentissage du chapitre 2
Exercice 1
Soit f la fonction définie sur par f ( x ) = x 3 + 6 x 2 + 9 x et C sa courbe
représentative dans un repère orthogonal du plan.
La dérivée de f est définie, pour tout x réel, par
f '( x ) = 3x 2 + 12x + 9 = 3( x 2 + 4 x + 3), soit f '( x ) = 3( x + 1)( x + 3).
On a ▶ f '( x ) = 0 pour x = – 3 et pour x = – 1
▶ f
'( x ) > 0 si x < −3 ou x > −1
▶ f
'( x ) < 0 si − 3 < x < −1.
La fonction f est
▶ croissante
sur ] − ∞ ; − 3] et sur [ − 1; + ∞ [
▶ décroissante
x
–3
A
f '( x )
sur [– 3 ; – 1].
+
0
–1
–
0
H
+
0
f (x )
–4
Le tracé de la courbe C est sur la figure 6.
a) La dérivée seconde de f est définie, pour tout x réel, par f "( x ) = 3( 2x + 4 ),
soit f "( x ) = 6( x + 2).
La dérivée seconde s’annule et change de signe pour x = – 2.
Le point K (–2 ; – 2) est un point d’inflexion de la courbe C.
Calculons f ’(–2) = –3. Une équation de TK est y = −3( x + 2) − 2, soit y = –3x − 8.
b) Développons ( x + 2)3 = ( x + 2)( x + 2)2 = ( x + 2)( x 2 + 4 x + 4 ), soit
( x + 2)3 = x 3 + 6 x 2 + 12x + 8.
Pour montrer que la tangente TK ne recoupe pas la courbe C on va montrer que
l’équation f ( x ) = −3x − 8 possède une solution unique.
Résolvons l’équation x 3 + 6 x 2 + 9 x = −3x − 8, c’est-à-dire
x 3 + 6 x 2 + 12x + 8 = 0.
Corrigé Séquence 8 – MA01
185
© Cned - Académie en ligne
D’après ce qui précède on obtient ( x + 2)3 = 0, d’où x = – 2.
L’équation f ( x ) = −3x − 8 admet une solution unique ; la tangente TK ne recoupe
pas la courbe C.
La courbe C et la tangente TK ont un seul point commun, le point d’inflexion
K (– 2 ; – 2).
Autre méthode
C y = f(x) = x3 + 6x2 + 9x
TK y = –3x – 8
4
On a f "( x ) = 6 x + 12 = 6( x + 2).
f "( x ) = 0 pour x = – 2 ;
2
TK
–4
–3
f "( x ) > 0 pour x > – 2 ;
–2
–1
C
0
–2
K
f "( x ) < 0 pour x < – 2.
Ainsi :
C est au-dessus de la tangente TK sur
]− 2 ; + ∞[ ;
–4
C est en dessous de la tangente TK sur
] − ∞ ; − 2[.
–6
La tangente TK et la courbe C ont donc un
seul point commun, le point d’inflexion
K (– 2 ; – 2).
Figure 6
Exercice 2
1
1
3
Soit f la fonction définie sur [0 ; 4] par f ( x ) = − x 3 + x 2 et C sa courbe
32
16
représentative.
La dérivée de f est définie sur [0 ; 4] par f '( x ) = −

3 2 3
3  1
x + x = ×  − x 2 + x .
32
8
8  4

1
Soit P la parabole d’équation y = − x 2 + x . Cette parabole a pour sommet le
4
1
point S (2 ; 1) et ses branches sont orientées vers le bas car − < 0.
4
La fonction dérivée est donc croissante sur [0 ; 2] et décroissante sur [2 ; 4].
La fonction f est convexe sur [0 ; 2] et concave sur [2 ; 4].
Rassemblons ces résultats dans un tableau.
186
© Cned - Académie en ligne
Intervalle
Pente de la tangente = f ‘
f
[0 ; 2]
Croissante
Convexe
[2 ; 4]
Décroissante
Concave
Position de C
C au-dessus de TK
C en dessous de TK
Corrigé Séquence 8 – MA01
La dérivée seconde de la fonction f est définie sur [0 ; 4] par
f "( x ) = −
▶
3
3 3
x + = ( − x + 2) .
16
8 16
f "( x ) = 0 pour x = 2 ▶ f "( x ) > 0 pour 0 ≤ x < 2
▶
f "( x ) < 0 pour 2 < x ≤ 4.
La dérivée seconde s’annule, en changeant de signe, pour x = 2.
Le point K(2 ; 0,5) est un point d’inflexion de la courbe C.
Soit TK la tangente à la courbe C au point K.
La courbe C est au-dessus de TK sur [0 ; 2] et en dessous de TK sur [2 ; 4] (voir
tableau).
La pente de la tangente T en M ( x ; f ( x )) est définie
M
Remarque
par p ( x ) = f '( x ).
Ce résultat démontre la conjecture
faite dans l’exercice II de synthèse
de la Séquence 2.
▶
La
dérivée
de
p '( x ) = f "( x ) = −
la
fonction
p
est
telle
que
3
3 3
x + = (2 − x ).
16
8 16
p’(x ) = 0 pour x = 2 ▶ p '( x ) > 0 pour x ∈[0 ; 2[
▶
p '( x ) < 0 pour x ∈]2 ; 4 ].
La pente est croissante sur [0 ; 2] et décroissante sur [2 ; 4]. Elle est donc
maximale pour x = 2.
La pente est maximale au point d’inflexion K d’abscisse x = 2.
Calculons pmax = p (2) = f '( 2) =
3
= 0, 375.
8
En
pourcentage
on
obtient
pmax = 37, 50 %.
Exercice 3
Soit f la fonction définie sur par f ( x ) = x 4 − 6 x 2 + 8 x − 3 et C sa courbe
représentative dans un repère orthogonal du plan.
Développons ( x + 2)( x − 1)2 = ( x + 2)( x 2 − 2x + 1),
d’où ( x + 2)( x − 1)2 = x 3 − 3x + 2.
La dérivée de f est définie sur
par f '( x ) = 4 x 3 − 12x + 8 = 4( x 3 − 3x + 2).
On en déduit f '( x ) = 4( x + 2)( x − 1)2.
▶
f '( x ) = 0 pour x = – 2 et pour x = 1 ▶ f '( x ) < 0 pour x < – 2 ▶ f '( x ) > 0 pour x > – 2 et x ≠ 1.
La fonction f est décroissante sur ] − ∞ ; − 2] et croissante sur [ − 2 ; + ∞ [.
Corrigé Séquence 8 – MA01
187
© Cned - Académie en ligne
On peut maintenant dresser le tableau de variation de la fonction f.
x
–2
A
–
f '( x )
–1
0
+
0
H
+
0
f (x )
–27
La fonction dérivée seconde de f est définie pour tout x réel par
f "( x ) = 4( 3x 2 − 3) = 12( x 2 − 1), d’où f "( x ) = 12( x − 1)( x + 1).
▶ f "( x ) = 0 pour x = – 1 et pour x = 1▶ f ''( x ) < 0 pour – 1 < x < 1 ▶ f ‘‘( x ) > 0
pour x < – 1 ou x > 1.
x
–1
A
f "( x )
+
f
Convexe
1
0
–
0
Concave
H
+
Convexe
La dérivée seconde s’annule deux fois en changeant de signe. La courbe C admet
donc deux points d’inflexion.
Calculons f ( −1) = −16 et f (1) = 0.
Les points K ( −1 ; − 16) et A(1 ; 0) sont les deux points d’inflexion de la courbe
C. On a f '( −1) = 16 et f '(1) = 0.
L’équation de la tangente en K est y = 16( x + 1) − 16, soit y = 16 x et celle de la
tangente en A est y = 0.
Le signe de la dérivée seconde nous permet de dire quand f est convexe et quand
elle est concave (voir tableau).
Les tracés demandés sont sur les figures 7 a et 7 b.
a) D’après le graphique on conjecture que la tangente T , qui est l’axe des
A
abscisses, coupe la courbe C en B (– 3 ; 0).
Remarque
Calculons f ( −3) = 81− 54 − 24 − 3 = 0.
On peut écrire f ( x ) = ( x + 3)( x − 1)3 .
La tangente TA recoupe C en B (– 3 ; 0).
188
© Cned - Académie en ligne
Corrigé Séquence 8 – MA01
b) L’écran de la calculatrice nous montre que la tangente TK coupe la courbe C
en un point E.
On conjecture xE = 3. Calculons f ( 3) = 81− 54 + 24 − 3 = 48 et 16 × 3 = 48. Le
point E se trouve donc sur C et sur TK .
La tangente TK recoupe C en E (3 ; 48).
On peut, à l’aide de la calculatrice, donner les positions relatives de la courbe C
et de la tangente TK .
x
−∞
–1
3
C au-dessus de TK
Positions
+∞
C au-dessus de TK
C traverse TK
C coupe TK
relatives
C en dessous de TK
E n’est pas
un point
d’inflexion
K est un point
d’inflexion
Inflexion
48
E
40
TK
C y = x4 – 6x2 + 8x – 3
TK y = 16x
20
C
B
–3
A
0
–2
–1
K
0
–20
1
2
3
–16
–27
Convexe
–40
Concave
Convexe
Figure 7 a
Figure 7 b
Corrigé Séquence 8 – MA01
189
© Cned - Académie en ligne
Exercice 4
Pour tout x réel on pose f ( x ) =
4
et g ( x ) = ln(f ( x )).
x +3
La fonction f est définie et strictement positive sur . La fonction g est donc
bien définie sur
2
.
f '( x )
. Comme f ( x ) > 0, g '( x ) a le même signe que f '( x ).
f (x )
Les fonctions f et g ont le même sens de variation sur .
On a g '( x ) =
On peut écrire g "( x ) =
f "( x ) × f ( x ) − f '( x )
f ( x )
On a
f "( x ) × f ( x ) − [f '( x )] =
2
=
D’où g "( x ) =
2
24( x 2 + 3)( x 2 − 1)
( x 2 + 3)4
96( x 2 − 1) − 64 x 2
( x 2 + 3)4
2
.
64 x 2
4
× 2
−
x + 3 ( x 2 + 3)4
=
32( x 2 − 3)
( x 2 + 3)4
.
32( x 2 − 3) ( x 2 + 3)2
2( x 2 − 3)
×
,
g
"(
x
)
=
.
soit
16
( x 2 + 3)4
( x 2 + 3)2
a) La dérivée g " a le même signe que x 2 − 3 = ( x − 3 )( x + 3 ).
▶
g "( x ) = 0 pour x = − 3 et pour x = 3
x> 3
▶
▶ g "( x ) > 0 pour x < − 3 ou
g "( x ) < 0 pour − 3 < x < 3.
La fonction g est convexe sur  − ∞ ; − 3  et sur  3 ; + ∞  ; la fonction g
est concave sur  − 3 ; 3  .


b) Les points A et B, d’abscisses respectives x A = − 3 et x B = 3, sont deux
points d’inflexion de la courbe C g .
8 3
 2
f '( − 3 )
3
= 36 =
.
Calculons g ( − 3 ) = ln   et g '( − 3 ) =
2
3
 3
f (− 3 )
3
L’équation
y=
190
© Cned - Académie en ligne
de
la
 2
3
x + 1+ ln   .
3
 3
Corrigé Séquence 8 – MA01
tangente
en
A
est y =
 2
3
( x + 3 ) + ln   ,
3
 3
soit
8 3
−
 2
f '( 3 )
3
Calculons g ( 3 ) = ln   et g '( 3 ) =
= 36 = −
.
2
3
 3
f ( 3)
3
 2
3
( x − 3 ) + ln   , soit
L’équation de la tangente en B est y = −
3
 3
 2
3
y =−
x + 1+ ln   .
3
 3
Les courbes C et C sont tracées sur la figure 8.
f
g
Cf y = f ( x ) =
1,5
4
x2 + 3
1
C g y = g ( x ) = ln(f ( x ))
Cf
0,5
–兹3
–2,5
–2
–1,5
A
兹3
0
–1
–0,5
0
0,5
1
1,5
2
B
–0,5
2,5
Cg
–1
Figure 8
Exercice 5
Soit f, g et h les trois fonctions définies sur
par f ( x ) = ( x 2 + 2x + 2) e x ,
g ( x ) = ( x 2 + 2x + 3) e x et h ( x ) = ( x 2 + 2x + 4 ) e x .
Fonction
f
g
h
Fonction dérivée
f '( x ) = ( x 2 + 4 x + 4 ) e x
g '( x ) = ( x 2 + 4 x + 5) e x
h '( x ) = ( x 2 + 4 x + 6 ) e x
f "( x ) = ( x 2 + 6 x + 8 ) e x
g "( x ) = ( x 2 + 6 x + 9 ) e x
h "( x ) = ( x 2 + 6 x + 10 ) e x
f "( x ) = ( x + 2)( x + 4 ) e x
g "( x ) = ( x + 3)2 e x
On a x 2 + 6 x + 10 > 0.
Fonction dérivée
seconde
f "( x ) = 0 pour x = – 2 et g "( x ) = 0 pour x = – 3.
Signe de la
pour x = – 4.
Pour tout x réel, h "( x ) > 0.
dérivée seconde f " s’annule en – 2 et en – 4, g " s’annule en – 3, sans
h " ne s’annule pas.
changer de signe.
en changeant de signe.
Point(s)
d’inflexion
Cf admet 2 points
d’inflexion.
Pas de point d’inflexion pour C g .
A( −4 ; 10e −4 ) et B ( −2 ; 2e −2 )
Pas de point d’inflexion pour C h .
Corrigé Séquence 8 – MA01
191
© Cned - Académie en ligne
Exercice 6
La courbe C représentant la fonction " ln" sur l’intervalle [1; + ∞ [ est sur la
figure 9 a. Soit k un réel tel que k ≥ 1.
Considérons les points A(k ; 0 ), B (k + 1 ; 0 ), M (k ; lnk ), N (k + 1 ; ln(k + 1)).
Appelons (D ) le domaine limité par l’axe des abscisses, la courbe C et les droites
d’équations x = k et x = k + 1.
Comme la fonction" ln" est concave sur l’intervalle [1; + ∞ [ la corde [MN ] est
située sous la courbe C sur l’intervalle [k ; k + 1]. La corde [MN ] et la courbe de
la fonction ln sont presque confondues. Le trapèze A B N M est sur la figure 9 b.
On a : aire( AB N M ) ≤ aire(D ) (les aires sont exprimées en u.a).
Le quadrilatère A B N M est un trapèze rectangle et son aire est :
aire( ABNM ) =
BN + AM
ln(k + 1) + lnk ln k (k + 1)
× AB =
=
.
2
2
2
L’aire du domaine (D ) est : aire(D ) = ∫
L’inégalité se traduit par
ln k (k + 1)
2
k +1
lnt dt .
k
≤∫
k +1
lnt dt .
k
Remarque
Pour k = 2,5 la calculatrice nous donne : aire (A B N M) = 1,084 526… et
aire(D ) = ∫
3 ,5
2,5
lnt dt = 1, 093 943...
N
N
In(k+1)
1
In(k)
M
M
Trapèze
(ABNM)
(D)
A
0
k
1
Figure 9 a
192
© Cned - Académie en ligne
B
Corrigé Séquence 8 – MA01
k+1
A
B
k
k+1
Figure 9 b
Exercice 7
Cas particulier
Au cours du premier trimestre, Yann a obtenu comme notes 9 et 16 en
mathématiques. Voici les résultats des 4 moyennes possibles :
Moyenne
arithmétique
x +y
2
G= xy
9 + 16
= 12, 5
2
G = 9 × 16 = 12
A=
A=
Moyenne
géométrique
Moyenne
harmonique
H=
H=
Moyenne
quadratique
2x y
x +y
2 × 9 × 16
= 11, 52
9 + 16
Q=
Q=
x2 + y2
2
92 + 162
= 12, 98...
2
Pour cet exemple on a H < G < A < Q . Yann aura une moyenne égale à 13 .
Nous allons voir si, dans le cas général, ce classement reste le même.
a) Soit f la fonction définie, pour x ≥ 0, par f ( x ) = x 2 et soit a et b deux réels
strictement positifs.
La fonction f est convexe sur l’intervalle [0 ; + ∞[, donc aussi sur [a ; b].
2
 a + b  f (a ) + f ( b )
a +b
a2 + b2
.
L’inégalité des milieux f 
≤
nous
donne
≤
 2 
2
2
 2 
2
a +b
a2 + b2
,
La fonction x x est croissante sur [0 ; + ∞[ d’où 
≤
2
 2 
a +b
a2 + b2
soit
.
≤
2
2
On a donc A ≤ Q .
b) Soit f la fonction définie, pour x > 0, par f ( x ) = ln x et soit a et b deux réels
strictement positifs.
La fonction f est concave sur l’intervalle [0 ; + ∞[, donc aussi sur [a ; b].
 a + b  f (a ) + f ( b )
 a + b  lna + ln b
,
L’inégalité des milieux f 
≥
nous donne ln 
≥

2
2
 2 
 2 
a +b 1
soit ln 
≥ ln(ab ).
 2  2
a +b
1
≥ ln ab , d’où
Pour x > 0 on a ln x = ln x . On obtient ainsi ln 
2
 2 
a +b
≥ ab car la fonction " ln" est croissante.
2
On a donc G ≤ A .
Corrigé Séquence 8 – MA01
193
© Cned - Académie en ligne
c) Soit f la fonction définie, pour x > 0, par f ( x ) = ln x et soit a et b deux réels
strictement positifs.
La fonction f est concave sur l’intervalle ]0 ; + ∞[,.
Appliquons l’inégalité des milieux aux deux réels
1 1
et . Sachant que
a b
 1
ln   = − ln x on peut successivement écrire :
x

 1 1  ln  1 + ln  1 

 a 
 b 
 2  − lna − ln b
a+b
ln 
; − ln 
;
≥
≥
1
1
2
2
2
 + 


a b




 2  1
 2ab 
ln 
 ≤ ln(ab ) ; ln 
 ≤ ln ab .
a
+
b
2
+
a
b



 ab 
Comme la fonction" ln" est croissante on obtient
2ab
≤ ab . D’où H ≤ G.
a +b
d) En récapitulant tous les résultats on obtient H ≤ G ≤ A ≤ Q .
La moyenne la plus favorable est donc toujours la moyenne quadratique.
Remarque
▶ Si
a = b les 4 moyennes donnent le même résultat.
▶ Ces
formules peuvent être démontrées sans utiliser la convexité de
fonctions.
Ainsi, en développant
(
x− y
( x − y )2 on montre que A ≤ Q .
Complément
)
2
on montre que G ≤ A et en développant
On peut définir des moyennes analogues pour 3 notes (ou plus).
Les trois réels x, y et z étant strictement positifs on définit quatre moyennes.
Moyenne
Moyenne
Moyenne
Moyenne
arithmétique
géométrique
harmonique
quadratique
A=
x +y +z
3
G = 3xyz
3x y z
G est la racine H =
xy + yz + zx
cubique de x y z
On a G 3 = xyz 


Q=
x2 + y2 + z2
3
On pourrait montrer que H ≤ G ≤ A ≤ Q. Le vérifier pour x = 9, y = 12 et z = 16.
194
© Cned - Académie en ligne
Corrigé Séquence 8 – MA01
Exercice 8
Soit f et g les deux fonctions définies sur
par f ( x ) = e
−
x2
2
et g ( x ) =
2e x
.
ex + 1
On désigne par Cf et C g leurs courbes respectives dans un repère du plan.
Étude de f
−
La dérivée de f est définie par f '( x ) = − x e
dérivée a le même signe que – x.
x2
2
, d’où f '( x ) = − x f ( x ). Cette
La fonction f est croissante sur ] − ∞ ; 0] et décroissante sur [0 ; + ∞[.
Étude de g
La dérivée de g est définie par g '( x ) = 2 ×
d’où g '( x ) =
2e x
(e x + 1)2
e x (e x + 1) − e x × e x
(e x + 1)2
,
.
Pour tout x réel on a g '( x ) > 0. La fonction g est strictement croissante sur
.
Déterminons les fonctions dérivées secondes de f et de g.
▶ On
a f "( x ) =  − x f ( x ) '= −f ( x ) − xf '( x ) = −f ( x ) + x 2f ( x ),
soit f "( x ) = ( x 2 − 1)f ( x ).
f "( x ) a le même signe que ( x − 1)( x + 1).
▶ f "( x ) = 0 pour x = – 1 et pour x = 1 ▶ f "( x ) < 0 pour – 1 < x < 1 ▶ f "( x ) > 0
pour x < – 1 ou x > 1.
La fonction f est convexe sur ] − ∞ ; − 1] et sur [1; + ∞[ ; la fonction f est
concave sur [– 1 ; 1].
▶ On a
g "( x ) = 2 ×
e x (e x + 1)2 − e x × 2(e x + 1)e x
x
(e x + 1)4
x
= 2e (e + 1) ×
1− e x
(e x + 1)4
= 2e x (e x + 1) ×
e x + 1− 2e x
(e x + 1)4
.
g "( x ) a le même signe que 1− e x .
▶ g "( x ) = 0 pour x = 0 ▶ g "( x ) < 0 pour x > 0 ▶ g "( x ) > 0 pour x < 0.
La fonction g est convexe sur ] − ∞ ; 0] et concave sur [0 ; + ∞[.
▶ La courbe C possède deux points d’inflexion, l’un A d’abscisse – 1 et l’autre
f
B d’abscisse1.
Corrigé Séquence 8 – MA01
195
© Cned - Académie en ligne
)
)
Calculons f ( −1 = e −0,5 et f (1 = e −0,5 .
(
) (
Les points A −1 ; e −0,5 et B 1 ; e −0,5
courbe Cf .
) sont les deux points d’inflexion de la
Le tracé de la courbe Cf est sur la figure 10.
La courbe C g possède un point d’inflexion K d’abscisse 0. Calculons
2e0
g (0) =
= 1.
e0 + 1
▶ Le point K (0 ; 1) est le point d’inflexion de la courbe C g . Le tracé de la courbe
C g est sur la figure 11.
Remarque
La courbe Cf est une courbe " en cloche".
Elle est symétrique par rapport à l’axe des
ordonnées.
On pourrait montrer que le point K est centre
de symétrie pour la courbe C g .
1,2
TA
TB
1
y = exp (–0,5x2)
0,8
A
Cf
B
0,6
0,4
0,2
0
–2,5
–2
–1,5
–1
–0,5
0
0,5
1
1,5
2
2,5
–0,2
Convexe
Concave
Convexe
Figure 10
2
TK
Cg
1,5
1
Convexe
Concave
K
0,5
–2
–1
O
Figure 11
196
© Cned - Académie en ligne
Corrigé Séquence 8 – MA01
1
2
Exercice 9
Dans tout l’exercice les aires seront exprimées en unités d’aire ; l’unité d’aire est
l’aire d’un carré de côté 1.
On a aire(O A B A ) = OA × OA , d’où aire(O A1B A2 ) = 168.
1 2
1
2
D’après l’énoncé on a (figure 16 du cours) aire (O A B ) = 84 et (figure 17
1
du cours) aire (O A2 B ) = 85.
On
déduirait : aire(O A1B A2 ) = aire(O A1B ) + aire(OB A2 ) = 84 + 85
en
soit
aire(O A1B A2 ) = 169.
Dans la question on a trouvé aire(O A1B A2 ) = 168.
Les calculs précédents nous donneraient 168 = 169 ce qui évidemment est faux.
Il nous reste à trouver l’erreur du raisonnement précédent.
a) Calculons les coefficients directeurs des droites (OB ), (OH ) et (OK ). On
peut les appeler respectivement mB , mH et mK .
8
B = 21 = 0, 380...
▶ m
3
H = 8 = 0, 375
▶ m
5
K = 13 = 0, 384...
▶ m
On a donc mH < mB < mK . Les points H et K ne sont pas sur la droite (OB ).
Le point H est situé en dessous de la droite (OB ) alors que le point K est situé
au-dessus de la droite (OB ).
Les points O, H, B, K forment donc un quadrilatère non aplati (en réalité un
parallélogramme).
b) Vu la position des points O, H, B, K (voir les figures 16 et 17) on peut écrire :
aire(OH B K ) = aire(O A2 B K ) − aire(O A1B H ) = aire("carré blanc").
D’où aire (O H B K) = 1.
c) Dans la question on a raisonné comme si les points O, H et B de la figure 16
du cours et les points O, K et B de la figure 17 du cours étaient alignés.
A2
B
Si on place sur une même figure les 4
points O, H, B, K ceux-ci apparaissent
" presque" alignés (voir figure 12).
Seul le calcul permet d’affirmer que les
4 points O, H, B, K ne sont pas alignés.
K
H
A1
O
Figure 12
Corrigé Séquence 8 – MA01
197
© Cned - Académie en ligne
a) Soit (P ) la parabole d’équation y = f ( x ) = ax 2 + bx passant par les
H
points O, H et B.
On peut écrire : f (8 ) = 3 et f (21) = 8.
64 a + 8 b = 3
D’où 
441a + 21b = 8
−21
8
441
−64
En utilisant les coefficients multiplicateurs indiqués on obtient
2 184 a = 1 et 2 184 b = 811.
D’où a =
y=
1
811
et b =
. La parabole (PH ) a pour équation
2 184
2 184
1
( x 2 + 811x ).
2 184
Le coefficient de x 2 étant positif la parabole a ses branches orientées vers le
haut :
la fonction f est convexe.
Résolvons l’équation f ( x ) = 0. On a x 2 + 811x = x ( x + 811) = 0, d’où x = 0 ou x = – 811.
La parabole (PH ) recoupe l’axe des abscisses au point
E (– 811 ; 0)
(voir le tracé sur la figure 13).
b) Soit (PK ) la parabole d’équation y = g ( x ) = cx 2 + dx passant par les points
O, K et B.
On peut écrire : g (13) = 5 et g (21) = 8.
169c + 13d = 5
D’où 
441c + 21d = 8
−21
13
441
−169
En utilisant les coefficients multiplicateurs indiqués on obtient
2 184 c = −1et 2 184 d = 853
D’où c = −
y=
1
853
et d =
.
2 184
2 184
La
parabole (PK )
a
pour
équation
1
( − x 2 + 853x ).
2 184
Le coefficient de x 2 étant négatif la parabole a ses branches orientées vers le
bas : la fonction g est concave.
Résolvons l’équation g ( x ) = 0. On a − x 2 + 853x = x ( − x + 853) = 0, d’où x = 0
ou x = 853.
198
© Cned - Académie en ligne
Corrigé Séquence 8 – MA01
La parabole (PK ) recoupe l’axe des abscisses au point F (853 ; 0) (voir le tracé
sur la figure 13).
(PK)
50
g est concave
E –811
–800
853
0
–600
–400
–200
f est convexe
0
–50
(PH)
200
400
600
800 F 1000
Les deux courbes se coupent
aux points de coordonnées
(0 ; 0) et (21 ; 8)
Figure 13
Remarque
Les nombres 8, 13 et 21n’ont pas été choisis au hasard. Ce sont 3 termes
consécutifs de la suite de Fibonacci (vers 1180 – 1250) ainsi définie :
u0 = 1, u1 = 1 et, pour tout entier n ≥ 0, un + 2 = un +1 + un .
On obtient alors la suite 1, 1, 2, 3, 5, 8, 13, 21, 34, 55, 89, 144,…
2
On aurait pu choisir un rectangle 3 × 8 pour trouver "3 × 8 = 5 ", ou un
rectangle 13 × 34 pour trouver " 13 × 34 = 212" ; etc.
Exercice 10
Soit f la fonction définie sur
par f ( x ) = e− x ln(1+ e x )
La fonction dérivée est définie sur par f '( x ) = − e − x ln(1+ e x ) + e − x ×
d’où f '( x ) =
1
1+ e
x
− e − x ln(1+ e x ).
ex
1+ e x
,
Les coordonnées du point A sont (0 ; f (0 )) ce qui donne A(0 ; ln 2).
1

1
Calculons f '(0 ) = − ln 2. Une équation de la tangente TA est y =  − ln 2 x + ln 2.
2

2
D’après le graphique on peut dire que la tangente TA recoupe la courbe C en un
point B dont les coordonnées sont environ (2,5 ; 0,2).
D’après le graphique on conjecture
i la dérivée semble négative sur ;
i la dérivée semble décroissante sur un intervalle ] − ∞ ; α ] ,
puis croissante sur [α ; + ∞ [.
Corrigé Séquence 8 – MA01
199
© Cned - Académie en ligne
Le graphique nous montre que le sens de variation
de la dérivée change pour une valeur x = α.
La courbe C change donc de convexité pour x = α.
Le graphique nous montre que 0 ≤ α ≤ 1.
D’après le tableur on obtient les résultats suivants :
a) f '(0, 8 ) < f '(0, 7) et f '(0, 8 ) < f '(0, 9 ).
D’où 0,7 < α < 0,9.
b) f '(0,77) < f '(0,76) et f '(0,77) < f '(0,78)
D’où 0,76 < α < 0,78.
c) f '(0, 771) < f '(0, 770 ) et f '(0, 771) < f '(0, 772).
D’où
0,770 < α < 0,772.
L’intervalle [0,770 ; 0,772] a bien pour amplitude 0,002.
On choisit
α = 0,771.
La fonction f semble
200
© Cned - Académie en ligne
Corrigé Séquence 8 – MA01
i concave sur ] − ∞ ; α ] ;
i convexe sur [α ; + ∞[.
Corrigé des exercices de synthèse
de la séquence 8
Exercice I
Partie A – Préliminaire
Comme GA + GB + GC = 0,
on a x A − xG + x B − xG + xC − xG = 0 et y A − y G + y B − y G + y C − y G = 0.
 x + x + x = 3xG
,
On obtient le système  A B C
 y A + y B + y C = 3y G
x +x +x
y +y +y
d’où xG = A B C et y G = A B C .
3
3
Partie B
Le point G, centre de gravité du triangle ABC, est toujours situé à l’intérieur
du triangle. Comme la fonction f est concave, les trois cordes [AB], [BC] et [CA]
sont situées en dessous de la courbe Ꮿ . Le point G est donc situé en dessous de
la courbe Ꮿ et, en particulier, en dessous du point K.
On a xG = x K et y G ≤ y K .
y +y +y
f (a ) + f (b ) + f (c )
.
L’ordonnée de G est y G = A B C =
3
3
 a + b +c 
.
L’ordonnée de K est y K = f ( x K ) = f ( xG ) = f 
3 

 a + b + c  f (a ) + f (b ) + f (c )
≥
.
Comme y K ≥ y G , on obtient f 
3 
3

Soit a, b et c trois réels tels que a > 0, b > 0 et c > 0. On sait que la fonction
ln est concave sur ]0 ; + ∞[.
 a + b + c  ln(a ) + ln(b ) + ln(c )
.
L’inégalité précédente s’écrit alors ln 
≥
3 
3

La relation fonctionnelle des logarithmes népériens nous permet d’écrire :
ln(a) + ln(b) + ln(c) = ln(ab) + ln(c) = ln((ab)c) = ln(abc) .
 a + b + c  ln(abc )
.
≥
D’où ln 
3 
3

Corrigé Séquence 8 – MA01
201
© Cned - Académie en ligne
Exercice II
Soit f la fonction définie sur l’intervalle ]1; + ∞[ par f ( x ) = ln(ln x ).
Déterminons la dérivée seconde de f.
1
1
ln x + x ×
1
x , d’où f "( x ) = − 1+ ln x .
On a f ' x ) = x =
et f "( x ) = −
2
ln x x ln x
( x ln x )
( x ln x )2
Comme x > 1, on a 1+ ln x > 0 et f "( x ) < 0.
La fonction f est concave sur l’intervalle ]1; + ∞ [.
Soit x et y deux réels tels que x > 1 et y > 1.
La fonction f étant concave sur l’intervalle [x ; y] on peut écrire, d’après l’inégalité
 x + y  f (x ) + f (y )
des milieux, f 
≥
.
2
 2 
  x + y   ln(ln x ) + ln(ln y )
ce qui s’écrit encore
On a donc ln ln 
≥
2
  2 
  x + y   ln (ln x )(ln y )
ln ln 
.
≥
2
  2 
  x +y
1
Sachant que, pour x > 0, ln x = ln x on a ln ln 
  ≥ ln (ln x )(ln y ).
2
  2 
La fonction"ln" étant croissante sur ]1; + ∞[ on en déduit
x +y
ln 
≥
 2 
Exercice III
(ln x )(ln y ).
On sait que f ( x ) = ax 2 et f ( 4 ) = −2. On a donc 16a = −2,
1
d’où a = − = −0,125.
8
La fonction f définie sur [0 ; 4] par f ( x ) = −0,125x 2 est une fonction trinôme
représentée par un arc de parabole orienté vers les ordonnées négatives.
La fonction f est concave sur [0 ; 4].
La dérivée de f est définie sur [0 ; 4] par f '( x ) = −0, 25x ce qui donne
f '( 4 ) = −1.
La tangente TA a pour équation y = −( x − 4 ) − 2, soit y = – x + 2.
La tangente TA coupe l’axe des ordonnées au point F (0 ; 2).
202
© Cned - Académie en ligne
Corrigé Séquence 8 – MA01
On a AK 2 = ( x − x )2 + ( y − y )2 , soit AK 2 = ( 8 − 4 )2 + ( 2 + 2)2 = 32.
K
A
K
A
D’où AK = 32 = 4 2.
De même AF 2 = (0 − 4 )2 + (2 + 2)2 = 32. D’où AF = 4 2.
On a aussi FK = 8.
Calculons AF 2 + AK 2 = 32 + 32 = 64. Ainsi AF 2 + AK 2 = FK 2 , ce qui prouve que
le triangle AFK est rectangle en A (réciproque du théorème de Pythagore).
On a donc ( AK ) ⊥ ( AF ). Comme la droite (FA) est perpendiculaire à la droite
(AK), la droite (FA) est tangente à l’arc de cercle AB en A.
La courbe C traverse sa tangente au point A : le point A est un point d’inflexion
pour la courbe C.
a) Calculons OA 2 = 42 + 22 = 20, d’où OA = 2 5, soit OA = 4,47… On
élimine la valeur 4,47.
La longueur de l’arc de parabole OA est intuitivement inférieure à 5. On élimine
les valeurs 4,47 et 5,12.
La longueur de l’arc de parabole OA est donc égale à 4,59 (en unités de longueur).
Remarque
Soit J le milieu du segment [A E]. Un calcul nous donne
OJ + JA = 17 + 1 = 5,123…
On a OA < longueur (arc OA) < OJ + JA,
soit 4,472… < longueur (arc OA) < 5,123…
= 45° . L’angle
b) Le triangle AFK est un triangle isocèle rectangle en A, d’où AKF
est un angle droit ce qui nous donne AKB
= 45° . L’arc de cercle AB est
BKF
donc un huitième de cercle.
La longueur de l’arc de cercle AB est
1
1
× 2π r = × 2π × 4 2 = 4,442...
8
8
La longueur L de la courbe C est L = 4,59 + 4,44 = 9,03.
L’unité de longueur est égale à 100 m ; la longueur de la courbe C est égale à 903 m.
Corrigé Séquence 8 – MA01
203
© Cned - Académie en ligne
Remarque
De O à A la bretelle tourne vers la droite alors que de A à B elle tourne vers
la gauche. Au point A il y a un changement de convexité : la bretelle change
de courbure.
Soit g la fonction, définie sur [4 ; 8], dont la courbe représentative est l’arc
de cercle AB.
a) D’après le graphique, la fonction g est convexe sur [4 ; 8] (la route tourne vers
la gauche).
b) Soit D1 le domaine délimité par l’axe des abscisses, l’arc de parabole OA et la
droite d’équation x = 4.
Soit D2 le domaine délimité par l’axe des abscisses, l’arc de cercle AB et les
droites d’équations x = 4 et x = 8.
On a aire(D) = aire (D1) + aire (D2 ) en u.a.
▶ Sur [0 ; 4] la fonction f est concave et négative ; son opposée −f sera positive
(et convexe) sur [0 ; 4].
4
4
0
0
On a aire (D1) = ∫ −f ( x ) dx = ∫
4
1 2
8
 1 3
x dx = 
x  = (en u.a).
8
 8 × 3 0 3
du domaine D2 est en grisé sur la figure 14. L’aire du triangle AEG est
égale à celle du triangle GHK.
▶ L’aire
L’aire du domaine D2 est égale à celle du domaine délimité par les deux segments
[KA] et [KB] et l’arc de cercle AB.
Ce domaine représente un huitième du disque de centre K et de rayon KA.
Aire (D2 ) =
1 2 1
8
π r = π × 32 = 4π (en u.a). D’où aire (D) = + 4π u.a.
8
8
3
F
K
1
TA
E
Autoroute
0
1
D1
4
C
–2
G
H
8
D2
D’après l’énoncé 1 u.a =
10 000 m2 = 1 ha.
La calculatrice donne
8
+ 4π = 15, 233 037...
3
d’où
aire (D) = 15,23 ha
A
(arrondi à 0,01 près).
B
Figure 14
204
© Cned - Académie en ligne
Corrigé Séquence 8 – MA01
Exercice IV
Soit f la fonction définie sur
représentative.
par f ( x ) = x 4 − 6x 2 + 5 et C sa courbe
Résolvons l’équation X 2 − 6 X + 5 = 0.
Cette équation a pour discriminant ∆ = 16 et pour racines X 1 = 1 et X 2 = 5.
Résolvons l’équation x 4 − 6 x 2 + 5 = 0. En posant X = x 2 , avec X ≥ 0, cette
équation s’écrit X 2 − 6 X + 5 = 0.
D’après ce qui précède on obtient X = 1 ou X = 5.
▶ si x
2
= 1 alors x = – 1 ou x = 1
Soit ᏿
▶ si x
2
= 5 alors x = − 5 ou x = 5.
l’ensemble des solutions de l’équation f ( x ) = 0. On a alors
{
}
᏿ = − 5 ; − 1; 1; 5 .
a) La dérivée de la fonction f est définie sur
par
f '( x ) = 4 x 3 − 12x = 4 x ( x 2 − 3), soit f '( x ) = 4 x ( x − 3 )( x + 3 ).
Donnons le signe de f '( x ) dans un tableau.
x
−∞
0
– 3
+∞
3
x
–
–
x− 3
–
–
–
x+ 3
–
0
+
+
f '( x )
–
0
+
0
+
0
+
0
+
+
–
0
+
On peut maintenant dresser le tableau de variation de la fonction f.
x
f '( x )
−∞
0
– 3
–
0
+
0
+∞
3
–
0
+
5
f (x )
–4
–4
Corrigé Séquence 8 – MA01
205
© Cned - Académie en ligne
b) Le tracé de la courbe C est sur la figure 15.
8
C y = f(x) = x4 – 6x2 + 5
6
4
2
–兹5
(D1)
A
0
–2
0
B
(D2)
2 兹5
–2
–4
Convexe
Concave
Convexe
Figure 15
a) La dérivée seconde de la fonction f est définie sur
par
f "( x ) = 12x 2 − 12 = 12( x 2 − 1), soit f "( x ) = 12( x − 1)( x + 1).
x
−∞
–1
x–1
x+1
–
–
f "( x )
f
+
+∞
1
0
0
–
+
+
+
0
–
0
+
Convexe
Concave
Convexe
La dérivée seconde s’annule deux fois, en changeant de signe : la courbe C admet
deux points d’inflexion.
Les points A( −1 ; 0 ) et B (1 ; 0 ) sont les deux points d’inflexion de la courbe C.
b) La fonction f est convexe sur les intervalles ] − ∞ ; − 1] et [1; + ∞[, concave sur
l’intervalle [– 1 ; 1] (voir tableau).
c) Calculons f '( −1) = 8 et f '(1) = −8.
206
© Cned - Académie en ligne
Corrigé Séquence 8 – MA01
La tangente en A (– 1 ; 0) a pour équation y = 8( x + 1). La tangente en B (1 ; 0)
a pour équation y = −8( x − 1).
Sur l’intervalle [0 ; 1] la fonction f est positive. Le domaine (D ) délimité par
1
l’axe des abscisses, la courbe C et les droites d’équations respectives x = 0 et x = 1
est situé au-dessus de l’axe des abscisses.
1
D’où aire(D1) = ∫ f ( x ) dx u. a.
0
Une primitive F de f est définie sur
1
5
par F ( x ) = x 5 − 2x 3 + 5x .
1

1
1
aire(D1) = F ( x ) =  x 5 − 2x 3 + 5x  = − 2 + 5.
0
0 5
5
1
D’où aire(D1) = 3,2 (en u.a.)
Le domaine (D2 ) limité par l’axe des abscisses, la courbe C et les droites
d’équations respectives x = 1 et x = 5 est situé en dessous de l’axe des
abscisses. Sur l’intervalle [1 ; 5 ] la fonction f est négative, d’où la fonction – f
est positive.
On en déduit : aire(D2 ) = ∫
Ainsi aire(D2 ) = F ( x )
5
1
− f ( x ) dx = ∫
1
f ( x ) dx u.a.
5
1
1
5

1
=  x 5 − 2x 3 + 5x  = F (1)
 5
5
car F ( 5 ) = 0.
D’où
aire(D2 ) = 3, 2 (en u.a.)
Comme 1 u.a = 2 cm2 on obtient aire(D1) = aire(D2 ) = 6, 4 cm2.
Exercice V
Partie A
a) Soit f la fonction définie sur
2
3
par f ( x ) = − x 2 + 2.
La fonction f est une fonction trinôme définie sur et le coefficient de x 2 est
négatif.
La fonction f est concave sur .
1
b) Soit g la fonction définie sur par g ( x ) = x 2 − 2x + 3.
3
La fonction g est une fonction trinôme définie sur
positif.
La fonction g est convexe sur
2
3
et le coefficient de x 2 est
.
a) Calculons f (1) = − + 2 =
4
1
4
et g (1) = − 2 + 3 = . Les courbes Cf et C g
3
3
3
 4
passent par le point E  1 ;  .
 3
Corrigé Séquence 8 – MA01
207
© Cned - Académie en ligne
4
4
La dérivée de f est définie par f '( x ) = − x , d'où f '(1) = − .
3
3
2
2
4
La dérivée de g est définie par g '( x ) = x − 2, d'où g '(1) = − 2 = − .
3
3
3
On en déduit f '(1) = g '(1).
Les tangentes aux deux courbes au point E ont même coefficient directeur, elles
sont donc confondues.
 4
Les courbes Cf et C g passent par le point E  1 ;  et ont la même tangente T
 3
en ce point.
4
4
4
Une équation de la tangente T est y = − ( x − 1) + soit y = − ( x − 2).
3
3
3
4
La pente p de la tangente T est égale au nombre dérivé, d’où p = − .
3
Exprimée en pourcentage la pente est égale à – 133,33 %.
b) Les tracés demandés sont sur la figure 16.
c) La fonction f est concave, elle est donc située en dessous de la tangente T.
La fonction g est convexe, elle est donc située au-dessus de la tangente T.
2
La courbe Cf a pour équation y = f ( x ) = − x 2 + 2.
3
2,5
2
A
1,5
4/3
E
K
1
La courbe C g a pour équation
1
y = g ( x ) = x 2 − 2x + 3.
3
Ꮿ
Ch
Pour 0 ≤ x ≤ 1 on a Ꮿ = Cf .
0,5
B
0
–0,5
0,5
1
1,5
2
T
Pour 1 ≤ x ≤ 3 on a Ꮿ = C g .
2,5 TK 3
Figure 16
La
courbe C h
y = h( x ) =
a
pour
équation
4 3 2 2
x − x + 2.
27
3
La tangente à C h au point K a pour équation
5
y = −x + .
2
4
La droite T, d’équation y = − ( x − 2), est tangente à
3
Ꮿ en E.
208
© Cned - Académie en ligne
Corrigé Séquence 8 – MA01
Partie B
Soit h la fonction définie sur
par h( x ) =
4 3 2 2
x − x + 2.
27
3
On a h (0 ) = 2 et h ( 3) = 4 − 6 + 2 = 0. La condition (c1) est vérifiée.
4
4
La dérivée de h est définie par h '( x ) = x 2 − x , d’où h '( 3) = 0 et h '(0 ) = 0. La
9
3
condition (c 2 ) est vérifiée.
8
4 8
3
a) La dérivée seconde de h est définie par h "( x ) = x − =
x − .

9
3 9
2
3
Pour x = la dérivée seconde s’annule en changeant de signe.
2
3 
 3
Calculons h   = 1. Le point K  ; 1 est un point d’inflexion de la courbe C h .
2 
 2
3
3
b) On a x − > 0 sur l’intervalle ]1,5 ; 3] et x − < 0 sur l’intervalle [0 ; 1,5[.
2
2
La fonction h est concave sur [0 ; 1,5[ et convexe sur ]1,5 ; 3].
 3
a) Calculons h '   = 1− 2 = −1. Une équation de la tangente T
K
 2
est

3
5
y = −  x −  + 1, soit y = − x + .
2
2

La pente de la tangente TK est p = h '(1,5) = −1. En pourcentage on obtient une
pente de −100 %.
b) La courbe C h et la tangente TK sont tracées sur la figure 16.
c) Au point K la courbe C h traverse sa tangente. D’après la convexité de h on
peut dire que :
Exercice VI
▶
sur [0 ; 1,5[ la courbe C h est située en dessous de la tangente TK ;
▶
sur ]1,5 ; 3] la courbe C h est située au-dessus de la tangente TK .
La fonction f est définie sur
par f ( x ) =
x 3 + x 2 − 5x + 3
.
x2 + 3
On a ( x + 3)( x − 1)2 = ( x + 3)( x 2 − 2x + 1) = x 3 − 2x 2 + x + 3x 2 − 6 x + 3, soit
( x + 3)( x − 1)2 = x 3 + x 2 − 5x + 3.
Les abscisses des points où la courbe C coupe l’axe des abscisses sont, si elles
existent, les solutions de l’équation f ( x ) = 0.
Les solutions de l’équation f ( x ) = 0
sont les solutions de l’équation
x 3 + x 2 − 5x + 3 = 0.
Corrigé Séquence 8 – MA01
209
© Cned - Académie en ligne
On résout ( x + 3)( x − 1)2 = 0 dont les solutions sont x = −3 et x = 1.
La courbe C coupe l’axe des abscisses aux points A (– 3 ; 0) et B (1 ; 0).
Comme x 2 + 3 > 0, f (x ) a le même signe que ( x + 3)( x − 1)2.
Le signe de f ( x ) est donné dans le tableau suivant :
−∞
x
–3
x+3
–
( x − 1)2
+
f (x )
–
a) Résolvons dans
0
0
+∞
1
+
+
+
0
+
+
0
+
» l’équation X 2 + 14 X − 15 = 0.
On a ∆ = 256 ; les solutions de l’équation sont X = – 15 et X = 1.
b) La fonction dérivée f ' est définie sur
f '( x ) =
» par
( 3x 2 + 2x − 5)( x 2 + 3) − 2x ( x 3 + x 2 − 5x + 3)
f '( x ) =
f '( x ) =
( x 2 + 3)2
3x 4 + 2x 3 + 4 x 2 + 6 x − 15 − 2x 4 − 2x 3 + 10 x 2 − 6 x
( x 2 + 3)4
x 4 + 14 x 2 − 15
( x 2 + 3)2
.
2
En posant X = x 2 le numérateur de la dérivée s’écrit X + 14 X − 15. Ce trinôme
admet comme solutions X = – 15 et X = 1.
▶ x
2
= −15 n’a pas de solution.
▶ x
2
= 1 pour x = – 1 et pour x = 1.
La dérivée s’écrit f '( x ) =
( x 2 − 1)( x 2 + 15)
( x 2 + 3)2
trinôme x 2 − 1 = ( x − 1)( x + 1).
▶
f '( x ) = 0 pour x = – 1 et pour x = 1
▶ f
. Cette dérivée a le même signe que le
▶ f '( x ) > 0 pour x < – 1 ou x > 1
'( x ) < 0 pour – 1 < x < 1.
c) Dressons le tableau de variation de la fonction f.
210
© Cned - Académie en ligne
Corrigé Séquence 8 – MA01
−∞
x
–1
+
f '( x )
+∞
1
0
–
0
+
2
f (x )
0
a) La fonction dérivée seconde est définie sur
f "( x ) =
par
( 4 x 3 + 28 x )( x 2 + 3)2 − ( x 4 + 14 x 2 − 15) × 2 × 2x ( x 2 + 3)
(xx 2 + 3)4
.
4 x ( x 2 + 3) ( x 2 + 7)( x 2 + 3) − x 4 − 14 x 2 + 15 
,

D’où f "( x ) =
4
2
( x + 3)
soit f "( x ) =
4 x ( x 2 + 3)( 36 − 4 x 2 )
( x 2 + 3)4
On obtient f "( x ) =
.
16 x ( x 2 + 3)(9 − x 2 )
( x 2 + 3)4
, soit f "( x ) =
16 x ( x 2 + 3)( 3 − x )( 3 + x )
( x 2 + 3)4
.
La dérivée seconde a le même signe que le produit x ( 3 − x )( 3 + x ).
x
−∞
–3
0
x
–
9− x2
–
0
+
f "( x )
f
+
0
–
Convexe
–
+∞
3
0
+
0
Concave
+
+
0
–
+
0
–
Convexe
Concave
b) ▶ La fonction f est convexe sur ] − ∞ ; − 3] et sur [0 ; 3] ;
▶ La
fonction f est concave sur [– 3 ; 0] et sur [3 ; + ∞[.
Les coordonnées des points d’inflexion A, K et H sont données dans le tableau.
Point d’inflexion
Abscisse x
A
–3
K
0
H
3
Ordonnée f ( x )
0
1
2
f '( x )
4
3
Équation de la
tangente
4
y = x +4
3
5
3
4
3
5
y = − x +1
3
4
y = x −2
3
−
Corrigé Séquence 8 – MA01
211
© Cned - Académie en ligne
1
L’équation réduite de la droite (AH) est y = x + 1. Pour x = 0 on obtient y = 1,
3
ce qui prouve que le point K (0 ; 1) est sur la droite ( AH ).
Les trois points d’inflexion A, K et H sont alignés.
Remarque
On pouvait aussi montrer que K est le milieu de [ AH ].
x +x
y +y
On a A H = 0 = x K et A H = 1 = y K .
2
2
c) Les équations des tangentes à la courbe (C ) aux points d’inflexion sont
données dans le tableau.
Les tracés de la courbe et des trois tangentes sont sur la figure 17.
(C) y = f(x) =
3
x3 + x2 – 5x + 3
x2 + 3
(C)
(D)
A
–5
–4
H
2
1
K
B
0
–3
–1
–2
0
1
2
3
4
–1
Convexe
Concave
Convexe
–2
Concave
Figure 17
a) On peut écrire x + 1−
d’où f ( x ) = x + 1−
8x
2
x +3
8x
x2 + 3
=
( x + 1)( x 2 + 3) − 8 x
x2 + 3
=
x 3 + x 2 − 5x + 3
.
Posons u ( x ) = x 2 + 3, d’où u '( x ) = 2x . On a alors f ( x ) = x + 1−
Une primitive F de f est définie sur
212
© Cned - Académie en ligne
Corrigé Séquence 8 – MA01
x2 + 3
1
2
4 u '( x )
.
u( x )
par F ( x ) = x 2 + x − 4 ln( x 2 + 3).
,
b) Sur l’intervalle [– 3 ; 3] la fonction f est positive.
3
L’aire du domaine (D ) est donnée par : aire (D ) = ∫ f ( x ) dx en u.a.
−3
3
Ainsi aire (D ) = F ( x ) = F ( 3) − F ( −3).
−3
9
 9

aire (D ) =  + 3 − 4 ln12 −  − 3 − 4 ln12 , d’où aire (D ) = 6 u.a.
2
 2

Ce résultat est cohérent avec le graphique de la figure 17.
Exercice VII
Partie A
Comme K est le milieu du segment [MR ], son
x
x + xR
e +e
ordonnée est y = M
, d’où y =
2
2
On pose y = f ( x ) d’où f ( x ) =
5
M
−x
4
.
e x + e− x
.
2
3
K
(voir figure 18)
2
Les deux fonctions x convexes sur
.
ex
e− x
et x 2
2
sont
1
R
0
Comme la somme de deux fonctions convexes sur
–2
–1
est convexe sur on peut dire que
la fonction f est convexe sur
0
1
x
2
Figure 18
.
La dérivée de f est définie sur
▶ Résolvons
par f '( x ) =
e x − e− x
.
2
l’équation f '( x ) = 0. On obtient e x − e − x = 0 d’où e x = e − x .
On en déduit x = – x ce qui donne x = 0.
▶ Résolvons
l’inéquation f '( x ) > 0. On obtient e x − e − x > 0 d’où e x > e − x .
On en déduit x > – x ce qui donne x > 0.
La fonction f est décroissante sur ] − ∞ ; 0 ] et croissante sur [0 ; + ∞[.
La courbe (C ) représentant la fonction f sur l’intervalle I = [ −2 ; 2] est tracée
sur la figure 19.
Corrigé Séquence 8 – MA01
213
© Cned - Académie en ligne
4
4
3
3
Cv
ᏼ
2
F
1
(C)
–1
0
1
2
–2
E
1
A
0
–2
(D)
2
B
0
–1
Figure 19
0
1
2
Figure 20
Partie B
a) La longueur LC de la chaînette (C ) est donnée par L =
C
 e x − e− x
On a 1+ [f '( x )] = 1+ 
2

2
2
∫−2
1+ f '( x ) 2 dx .
2

4 + e2x + e −2x − 2  e x + e − x
=
 =
4
2


2

 .

2
e x + e− x
= f ( x ). Comme LC = ∫
1+ f '( x ) 2 dx on en
D’où 1+ f '( x ) =
−
2
2
2
2
déduit que LC = ∫ f ( x )dx .
−2
2
La longueur LC de la chaînette (C ) est donnée par LC = ∫ f ( x )dx .
−2
1
b) Une primitive F de f est définie sur [– 2 ; 2] par F ( x ) = (e x − e − x ).
2
2
2
2
1 x
1
LC = ∫ f ( x )dx = F ( x ) = e − e − x  = e2 − e −2  − e −2 − e2 
−2
 −2 2 
 

2
−2
(
d’où LC = e2 − e −2.
)
Ce résultat est exprimé en unités de longueur. Ici l’unité de longueur est égale à
2 cm. D’où LC = 2(e2 − e −2 ) cm.
La calculatrice donne LC = 14,507…
Une valeur arrondie à 0,01 près est LC = 14,51 cm.
Calculons f ( 2) =
e2 + e −2
= 3, 762... et appelons α l’arrondi à 0,01 près de
2
cette valeur. D’où α = 3,76.
214
© Cned - Académie en ligne
Corrigé Séquence 8 – MA01
Soit g une fonction définie sur l’intervalle I = [–2 ; 2] par g ( x ) = ax 2 + 1 et telle
que g (2) = α .
On a g (2) = 4a + 1 = 3, 76 d’où a = 0,69.
La fonction g est définie sur I = [–2 ; 2] par g ( x ) = 0, 69 x 2 + 1.
a) À l’aide d’un logiciel on obtient les courbes (C ) et ᏼ représentant
respectivement les fonctions f et g sur I = [ −2 ; 2].
Les deux courbes sont sur la figure 19.
On observe que sur l’intervalle I = [ −2 ; 2] les deux courbes sont très proches
l’une de l’autre. La courbe (C ) semble se situer légèrement en dessous de la
courbe ᏼ.
b) La longueur de l’arc de parabole représentant la fonction g est égale, arrondie
à 0,01 près, à 14,26 cm.
On a trouvé LC = 14,51 cm. L’écart entre les deux longueurs, noté e, est donc e = 0,25 cm.
Partie C
a) Soit v la fonction définie sur l’intervalle I = [ −2 ; 2] par v ( x ) = 1+ f ( 2) − f ( x ).
La fonction f étant convexe sur I , la fonction –f est concave sur I . Le fait
d’ajouter une constante, ici 1+ f (2), ne change pas la concavité de –f.
La fonction v est concave sur I .
b) Le tracé de la courbe Cv est sur la figure 20.
Sur l’intervalle I = [ −2 ; 2] la courbe C est au-dessus de la droite d’équation
v
y = 1.
L’aire du domaine (D ) limité par la courbe Cv , la droite d’équation
y = 1 et les droites d’équations x = – 2 et x = 2 est donnée, en unités d’aire, par
aire(D ) = ∫
2
−2
v ( x ) − 1 dx = ∫
aire(D ) = [f (2) × x ]
2
−2
2
2
−2
f (2) − f ( x )  dx .
− ∫ f ( x ) dx = 4 f (2) − (e2 − e −2 ) = 2(e2 + e −2 ) − (e2 − e −2 )
aire(D ) = e2 +3e −2 u.a.
−2
Ici 1 u.a = 4 cm2 d’où aire(D ) = 4(e2 + 3 e −2 ) cm2.
D’après la calculatrice, 4(e2 + 3 e −2 ) = 31,180...
Une valeur arrondie de cette aire, au mm2 près, est aire(D ) = 31,18 cm2.
Ce résultat est cohérent avec ce que l’on observe sur le graphique de la figure 20.
Corrigé Séquence 8 – MA01
215
© Cned - Académie en ligne
Exercice VIII
Partie A – Étude du coût marginal
Le coût marginal C
8
ma est défini sur l’intervalle [0 ; 5] par C ma ( x ) = x + 2x + 1.
La fonction dérivée est définie sur [0 ; 5] par
C 'ma ( x ) = 1−
16
(2x + 1)2
D’où C 'ma ( x ) =
=
(2x + 1)2 − 16
(2x + 1)2
(2x − 3)(2x + 5)
(2x + 1)2
=
(2x + 1− 4 )(2x + 1+ 4 )
(2x + 1)2
.
. Cette dérivée est du signe de 2x – 3.
ma ( x ) = 0 pour x = 1,5 ▶ C 'ma ( x ) > 0 pour 1, 5 < x ≤ 5
▶ C '
ma ( x ) < 0 pour 0 ≤ x < 1, 5.
▶ C '
a) La fonction coût marginal est
i décroissante sur [0 ; 1, 5] ;
i croissante sur [1, 5 ; 5].
Le coût marginal est minimal pour x = 1,5. Calculons C ma (1, 5) = 3, 5.
Le coût marginal est minimal pour x = 1,5 et est égal à 3,5 (x est exprimé en
centaines et C ma ( x ) en milliers d’euros).
b) Les rendements marginaux sont
i croissants sur [0 ; 1, 5] ;
i décroissants sur [1, 5 ; 5].
Partie B – Étude du coût total
a) On définit la fonction coût total, notée C , comme la primitive de la fonction
T
coût marginal sur l’intervalle [0 ; 5] qui est égale à 7,5 – 8 ln2 pour x = 0.
Soit F une primitive quelconque de la fonction coût marginal sur [0 ; 5].
2
u '( x )
On peut écrire C ma ( x ) = x + 4 ×
= x +4×
2x + 1
u( x )
en posant u ( x ) = 2x + 1, avec u ( x ) > 0.
1
Toutes les primitives F sont définies par F ( x ) = x 2 + 4 ln(2x + 1) + k où k est
2
une constante réelle.
b) Déterminons k sachant que F (0 ) = 7, 5 − 8 ln 2.
Comme F (0 ) = k on obtient k = 7, 5 − 8 ln 2.
La fonction coût total est définie sur [0 ; 5] par
1
CT ( x ) = x 2 + 4 ln(2x + 1) + 7, 5 − 8 ln 2.
2
216
© Cned - Académie en ligne
Corrigé Séquence 8 – MA01
La dérivée de la fonction coût total est la fonction coût marginal qui est
strictement positive sur [0 ; 5].
La fonction CT est strictement croissante sur [0 ; 5].
a) Déterminons la dérivée seconde de la fonction C , c’est-à-dire la dérivée
T
première de la fonction C ma .
CT "( x ) = C 'ma ( x ) d’où CT "( x ) =
(2x − 3)(2x + 5)
(2x +1)2
.
CT "( x ) a le même signe que 2x − 3. Ainsi CT "( x ) s’annule en changeant de
signe pour x = 1,5.
9
15
69
69
Calculons CT (1, 5) = + 4 ln 4 + − 8 ln 2 = + 8 ln 2 − 8 ln 2 = .
8
2
8
8
 3 69 
Le point K  ;  est un point d’inflexion pour la courbe Ꮿ.
2 8 
8 7
b) Le coefficient directeur de la tangente en K est égal à C ma (1, 5) = 1, 5 + = .
4 2
7
27
7
3  69
d’où y = x + .
La tangente en K a pour équation y =  x −  +
2
8
2
2 8
Partie C
Le coût moyen est défini sur l’intervalle ]0 ; 5] par
C (x ) x
ln(2x + 1) 7, 5 − 8 ln 2
CM ( x ) = T
= +4×
+
.
2
x
x
x
8
7
6
A
5
K
B
4
1
On a C M (3) ≈ 4,746
3
et C M (4) ≈ 4,686
2
x K ≈ 3, 734 et y K ≈ 4 , 679.
1
0
Figure 21
0
0,5
1
1,5
2
2,5
3
3,5
␣⬇3,7
4
4,5
5
Corrigé Séquence 8 – MA01
217
© Cned - Académie en ligne
a) La fonction CM admet un minimum pour
une valeur α telle que 3 ≤ α ≤ 4 (voir figure
21).
Le tableur nous montre que le sens de variation
de CM change sur l’intervalle [3,733 ; 3,735].
D’où 3,733 ≤ α ≤ 3,735.
b) Le coût moyen, par centaine d’appareils,
est minimum lorsque l’entreprise produit 373
appareils.
Le tableur nous permet de dire que le coût moyen minimal, par centaine
d’appareils, est égal à environ 4 679 euros.
On sait que C
Le réel α est solution de l’équation x 2 +
d’où α 2 +
α
8
ma (α ) = α + 2α + 1 et CM (α ) = 2 + 4 ×
ln(2α + 1) 7, 5 − 8 ln 2
+
.
α
α
16 x
− 8 ln( 2x + 1) − 15 + 16 ln 2 = 0,
2x + 1
16α
− 8 ln(2α + 1) − 15 + 16 ln 2 = 0.
2α + 1
On en déduit 8 ln(2α + 1) = α 2 +
d’où, en divisant par 2 α , 4 ×
On obtient alors CM (α ) =
16α
− 15 + 16 ln 2,
2α + 1
ln(2α + 1) α
8
7, 5 − 8 ln 2
= +
−
.
2 2α + 1
α
α
α α
8
7, 5 − 8 ln 2 7, 5 − 8 ln 2
8
+ +
−
+
=α +
.
2 2 2α + 1
α
α
2α + 1
On a montré que CM (α ) = C ma (α ).
8α
.
On peut écrire CT (α ) = α × CM (α ), d’où CT (α ) = α 2 +
2α + 1
218
© Cned - Académie en ligne
Corrigé Séquence 8 – MA01
C orrigé Séquence 9
Corrigé de l’exercice du chapitre 1
Exercice
1 1
= ≈ 0, 333 et n = 100. Les conditions
6 3

1
1 
d’utilisation de l’intervalle  p −
;p+
 sont vérifiées et on trouve environ
n
n


Le dé est bien équilibré donc p = 2 ×
0, 233 ; 0, 434  .
p = 0, 51 et n = 25, les conditions sont vérifiées et on trouve environ
0, 31; 0, 71 , ce qui signifie, en multipliant par 25, entre 8 et 17 garçons parmi
25 nouveau-nés.
Utilisation
Prise de décision
Dans chacun des deux cas suivants, les conditions d’utilisation de l’intervalle de
fluctuation vu en seconde sont vérifiées.
Pour n = 100 et p = 0, 5, l’intervalle de fluctuation est  0, 4 ; 0, 6  . La


fréquence observée f = 0, 56 appartient à cet intervalle, on décide que la pièce
est équilibrée.
Pour
n = 1000 et p = 0, 5, l’intervalle de fluctuation est environ
0, 4683 ; 0, 5317  . La fréquence observée f = 0, 560 n’appartient pas à cet
intervalle, on décide que la pièce n’est pas équilibrée.
Remarque
Dans les deux cas la fréquence est la même, mais la décision est différente.
Dans le deuxième cas, la taille de l’échantillon est plus grande ce qui diminue
l’amplitude de l’intervalle de fluctuation.
Corrigé Séquence 9 – MA01
219
© Cned - Académie en ligne
Corrigé de l’activité du chapitre 2
Activité 1

1
1 
Dans le premier cas, l’intervalle I = 0, 25 −
; 0, 25 +

50
50 

est
à
peu
près
l’intervalle
0,108 ; 0, 392
et
l’intervalle

0, 25 × 0, 75
0, 25 × 0, 75 
J = 0, 25 − 1, 96
; 0, 25 + 1, 96
 est à peu près l’inter50
50


valle [ 0,129 ; 0,371].
Remarque
On prend des valeurs approchées des bornes des intervalles. Il faut arrondir
la borne inférieure d’un intervalle par défaut et la borne supérieure par excès.
Dans ce premier cas, toutes les fréquences appartiennent à I et trois n’appartiennent
pas à J. Ainsi, 100% des fréquences appartiennent à I et 94% appartiennent à J.
Dans le deuxième cas, l’intervalle I est l’intervalle 0,15 ; 0, 35  et l’intervalle
J est à peu près l’intervalle 0,165 ; 0, 335  . On observe 99% des fréquences
dans I et 95% des fréquences dans J.
Dans le troisième cas, l’intervalle I est à peu près l’intervalle 0,179 ; 0, 321
et l’intervalle J est à peu près l’intervalle [ 0,189 ; 0,311]. On observe qu’une
fréquence n’est pas dans I et que six ne sont pas dans J. On observe que 99,5%
des fréquences sont dans I et 97% des fréquences sont dans J.
Dans les trois cas l’intervalle I contient l’intervalle J.
Dans les trois cas, on observe qu’au moins 95% environ des fréquences sont dans
J et plus de 95% dans I. C’est une propriété générale qui sera vue dans le cours.
220
© Cned - Académie en ligne
Corrigé Séquence 9 – MA01
Corrigés des exercices
d’apprentissage du chapitre 2
Exercice 1
n = 400, p = 0, 06, np = 24 et n (1− p ) = 376, donc les conditions sont remplies
pour utiliser l’intervalle de fluctuation asymptotique du cours.

p (1− p )
p (1− p ) 
 p − 1, 96
 est environ égal à [ 0,036 ; 0,084 ].
; p + 1, 96

n
n 
50
La fréquence est f =
= 0,125. Cette fréquence est en dehors de l’intervalle de
400
fluctuation donc on conclut que les femmes ayant eu un travail pénible pendant
leur grossesse sont plus susceptibles d’avoir un enfant prématuré que les autres.
Exercice 2
f =
21
≈ 0, 204.
103
n = 103, p = 0,12, np = 12, 36 et n (1− p ) = 90, 64 donc les conditions sont
remplies pour utiliser l’intervalle de fluctuation asymptotique du cours.

p (1− p )
p (1− p ) 
 p − 1, 96
 est environ égal à [ 0,057 ; 0,183].
; p + 1, 96

n
n 
La fréquence observée f se situe en dehors de l’intervalle de fluctuation. Non,
on ne peut pas dire que ce club est représentatif de la proportion de gauchers
dans le monde puisque, dans ce club, la fréquence des gauchers n’est pas dans
l’intervalle de fluctuation comme le sont 95% des fréquences des gauchers pour
des groupes de 103 personnes choisies au hasard.
Exercice 3
Pour p = 0, 02, les conditions pour n deviennent : n ≥ 30, n ≥
n≥
5
. La plus petite valeur de n qui convient est donc n = 250.
0, 98
L’amplitude de l’intervalle de fluctuation est égale à
2
n
5
et
0, 02
≤ 0,1 est équivalente à
2
n
. L’inéquation
2
≤ n , la plus petite valeur de n qui convient
0,1
est n = 400.
Corrigé Séquence 9 – MA01
221
© Cned - Académie en ligne
Corrigés des exercices
d’apprentissage du chapitre 3
Exercice 4
On se place dans le cas où l’échantillon contient au moins 30 éléments et où la
fréquence f observée est telle que nf ≥ 5 et n (1− f ) ≥ 5.
f = 0, 23 et n = 100,
nf = 23 et n (1− f ) = 77, donc les conditions
d’utilisation de l’intervalle de confiance au niveau 0,95 sont remplies et on

1
1 
obtient f −
;f +
 = 0,13 ; 0, 33  .
n
n


Après avoir fait des modifications, la fréquence obtenue dans un nouvel
échantillon est égale à 0,09. Les conditions sont de nouveau remplies et on
obtient un nouvel intervalle de confiance :  −0, 01; 0,19  . On retient [0 ; 0,19 ].
Les deux intervalles ne sont pas disjoints, il est donc possible, au niveau de
confiance de 95%, que les modifications n’aient pas apporté d’amélioration.
Exercice 5
La fréquence des avis favorables est égale à 0,53, donc nf = 53 et n (1− f ) = 47.
Les conditions sont remplies et un intervalle de confiance au niveau 0,95, est

1
1 
;f +
donné par f −
 ; on obtient 0, 43 ; 0, 63  . Il est donc possible
n
n

que la proportion de la population favorable à la proposition de la mairie soit
inférieure à 50%, on ne peut donc pas dire que la majorité de la population est
favorable au projet.
La fréquence obtenue est la même, mais maintenant n = 500. Les conditions
sont toujours remplies, le nouvel intervalle de confiance est [ 0,485 ; 0,575] et ne
permet toujours pas de conclure quant à la position de la majorité de la population.
Pour estimer, au seuil de confiance 0,95 que la majorité de la population
est favorable à cet emplacement, on doit choisir n de telle façon que la borne

1
1 
inférieure de l’intervalle 0, 53 −
; 0, 53 +
 soit supérieure à 0,5.
n
n

1
1
1
> 0, 5 ⇔
< 0, 03 ⇔ × 104 < n. La plus petite valeur de n
On a 0, 53 −
9
n
n
qui convient est 1112.
222
© Cned - Académie en ligne
Corrigé Séquence 9 – MA01
Corrigés des exercices
de synthèse de la séquence 9
Exercice I
Les comportements des clients étant indépendants les uns des autres, la
variable aléatoire X n suit la loi binomiale de paramètres n et p.
On a n > 300 et p = 0, 85 donc on a np ≥ 5 et n (1− p ) ≥ 5, et on peut utiliser

p (1− p )
p (1− p ) 
.
; p + 1,96
l’intervalle I n du cours : I n = p − 1,96

n
n 

0,1275
0,1275 
I n = 0,85 − 1,96
; 0,85 + 1,96
.

n
n 

Si In ⊂ 0 ;

300 
 X n 300 
X

≤
≥ P  n ∈I n  .
 , alors P 

 n
 n

n 
n 
Comme I n est un intervalle de fluctuation asymptotique au seuil 95 %, on
X
300 
obtient que P  n ≤
est supérieur à une valeur proche de 0,95. Or
n 
 n
X
300 
P ( X n > 300 ) = 1− P ( X n ≤ 300 ) = 1− P  n ≤
, donc P ( X n > 300 ) est
n 
 n
inférieur à une valeur proche de 0,05.

a) Si In ⊂ 0 ;

300 
0,1275 300
≤
et donc
 , alors 0, 85 + 1, 96
n 
n
n
0, 85n + 1, 96 0,1275 n − 300 ≤ 0.
b) La fonction f définie sur [1; + ∞[ par f ( x ) = 0, 85x + 1, 96 0,1275 x − 300
est croissante sur [1; + ∞[ car c’est la somme de deux fonctions croissantes sur
[1; + ∞[
( x 0,85x − 300 et x 1,96
)
0,1275 x .
La calculatrice montre que f ( 337) ≈ −0, 702 et f ( 338 ) ≈ 0,166, donc le plus grand
entier n0 pour lequel la fonction f prend une valeur négative est n0 = 337.

0,1275
0,1275 
; 0,85 + 1,96
c) On obtient alors I 337 = 0,85 − 1,96
 qui,
337
337 

en prenant des valeurs approchées des bornes, est inclus dans 0, 811; 0, 889  .
Corrigé Séquence 9 – MA01
223
© Cned - Académie en ligne
 300 
300
≈ 0, 891 on a bien l’inclusion I 337 ⊂ 0 ;
.
337
 337 
On peut conclure que la compagnie aérienne peut vendre 337 billets pour un
avion n’ayant que 300 places avec une probabilité d’environ 95 % que le nombre
de passagers ne dépasse pas 300.
Comme
Avec p = 0, 9, la fonction f est définie par f ( x ) = 0, 9 x + 1, 96 × 0, 3 x − 300,
 300 
le nombre entier n0 vaut 321 et on a bien I 321 ⊂ 0 ;
.
 321 
Avec p = 0, 95, la fonction f est définie par
f ( x ) = 0,95x + 1,96 × 0,0475 x − 300, le nombre entier n0 vaut 307 et on
 300 
a bien I 307 ⊂ 0 ;
.
 307 
On observe que, sur les trois cas, c’est lorsque la probabilité p est la plus grande
que le nombre n0 est le plus petit, ce qui était prévisible.
Il reste à la compagnie aérienne à calculer des indemnités pour les passagers
refusés à l’embarquement…
Exercice II
18
= 0, 225. Comme n = 80, nf = 18 et
80

1
1 
n (1− f ) = 62, on considère que l’intervalle f −
;f +
 est un intervalle
n
n

de confiance au niveau 0,95 de la proportion p. Cet intervalle vaut à peu près
La fréquence observée est f =
0,113 ; 0, 337  .
La précision est donnée par l’amplitude de l’intervalle de confiance qui vaut
2
2
≤ n ce qui donne n ≥ 1600.
0, 05
n
n

1
1 
On cherche n tel que  0, 225 −
; 0, 225 +
 ⊂ 0 ; 0, 25  c’est-à-dire tel
n
n

1
1
que 0 ≤ 0, 225 −
et 0, 225 +
≤ 0, 25.
n
n
1
1
et n ≥
. Cette
Ces deux conditions sont équivalentes à n ≥
0, 225
0, 025
dernière inéquation équivaut à n ≥ 1600, l’autre inéquation étant alors vérifiée.
. On veut
2
≤ 0, 05 c’est-à-dire
Pour n = 1600 les deux conditions sont vérifiées, l’intervalle de confiance au

1
1 
niveau 0,95 est alors 0, 225 −
; 0, 225 +
 soit 0, 2 ; 0, 25  .
1600
1600 

224
© Cned - Académie en ligne
Corrigé Séquence 9 – MA01
Téléchargement